Normal & Abnormal Growth and Development/Child Health/Paediatrics Flashcards by Lubi Mupwaya (2024)

1

Q

Which one of the following is the most common cause of painful rectal bleeding in children?

A. Intussusception.
B. Gastroenteritis.
C. Anal fissure.
D. Meckel’s diverticulum.
E. Hemorrhoids.

Gastroenterology

A

Correct Answer Is C.

Anal fissure is the most common cause of painful rectal bleeding in childhood. It is usually caused by passage of hard stool associated with constipation. The common presentation is bright blood on the surface of stool, on the nappy or toilet paper.

(Option A) Intussusception is associated with abdominal pain and rectal bleeding, not usually with pain on defecation and not as common as anal fissure.

(Option B) Gastroenteritis and juvenile polyps are the most common causes of rectal bleeding in children aged 2 to 5 years, but these conditions are not associated with painful defecation.

(Option D) Meckel’s diverticulum can cause blood in the stool and abdominal pain but the pain is not related to defecation. Moreover, it is far less common than anal fissure.

(Option E) Hemorrhoids neither are associated with painful defecation, nor as common in childhood.

How well did you know this?

1

Not at all

2

3

4

5

Perfectly

2

Q

Concerned parents of a 7-year-old boy have brought him to your clinic after he passed a large bloody stool this morning. No pain or discomfort is reported. Which one of the following can be the most likely cause?

A. Meckel’s diverticulum.
B. Duodenal ulcer.
C. Colon polyp.
D. Diverticulitis of the colon.
E. Intussusception.

Gastroenterology

A

Correct Answer Is C.

A large bloody stool, especially if painless, in a 7-year-old child is very likely to be caused by a juvenile colonic polyp.

Juvenile polyps are benign hamartomas, occurring between the ages 2 and 8 years, with a peak at 3 to 4 years.

Patients usually present with painless rectal bleeding; however, a few children may have lower abdominal pain from traction of the polyp. More than 60% of patients have a palpable polyp on rectal examination.

(Option A) Meckel’s diverticulum can result in massive rectal bleeding, often associated with central abdominal pain.
However, the presentation is expected earlier in life and is also less common than juvenile polyps.

(Option B) Duodenal ulcer is most likely to present with melena or hematemesis rather than large rectal bleeding.

(Options D and E) Diverticular disease is not common among children of this age group, neither is intussusception.
Moreover, intussusception is often painful.

How well did you know this?

1

Not at all

2

3

4

5

Perfectly

3

Q

A 5-year-old boy is brought to your clinic with generalized petechial rash and bruises all over his body. He had an episode of upper respiratory tract infection (URTI) 3 weeks ago. On examination, he is otherwise healthy. His blood tests is only significant for a platelet count of 74,000/mm3. Red blood cells and white cells are within normal range.

Which one of the following is the most appropriate management option in this child?

A. Strict bed rest.
B. Intravenous immunoglobulin (IVIG).
C. Steroids.
D. Antibiotics.
E. Plasma exchange.

Haematology

A

Correct Answer Is A.
Isolated thrombocytopenia in an otherwise healthy child following an URTI is highly suggestive of immune (idiopathic) thrombocytopenic purpura (ITP). ITP in children is a benign disease of unknown etiology.

ITP can be acute (~90%) or chronic (~10%). Both forms have similar presentation. Twenty percent of those with acute ITP will go on to have the chronic form, defined as persistent thrombocytopenia more than 6 months. However, some authors have suggested that 12 months is a more appropriate cut-off for defining chronic ITP.

The diagnosis is made based on manifestations of thrombocytopenia in the absence of abnormal findings, in
particular no pallor, lymphadenopathy or hepatosplenomegaly. Confirmation relies on exclusion of other causes of thrombocytopenia, acute leukemia in particular.

The management of children with ITP is controversial. There are generally two different approaches: (1) watchful
waiting and (2) pharmacologic intervention. None has been proved superior to the other.
Most patients with a platelet count > 20 x 10
9/L and some of those with a platelet count < 20 x 10 9/L can be managed in outpatient setting with no specific treatment if the following criteria are met:
* The diagnosis is certain, and there is no pallor, hepatosplenomegaly or lymphadenopathy, and there is only
isolated thrombocytopenia without anemia, leukopenia or blood film changes on laboratory studies
* There is no active bleeding other than bruising and petechiae in isolation. There should be no mucosal,
gastrointestinal or renal tract bleeding
* The child is otherwise well
* Social circ*mstances allow confidence about the degree of parental supervision and relative safety of the home environment, particularly for younger children.
* Parental reassurance and education in the emergency department can be provided.
* Follow up is guaranteed within a few days.

(Option B) Intravenous immunoglobulin (IVIG) is not routinely used and is reserved for patients with severe lifethreatening hemorrhage.

(Option C) This child has platelet count of >20,000/mm3, has no hemorrhage other than bruises and petechial rash and is otherwise healthy. He does not need pharmacological treatment. However, if pharmacological intervention is
planned, oral steroids are first-line.

(Option D) Antibiotics have no role in management of ITP.
(Option E) Plasma exchange is not an option for treatment of ITP

How well did you know this?

1

Not at all

2

3

4

5

Perfectly

4

Q

A mother brings her 7-year-old son 6 days after eruption of a generalized rash that, based on the history and clinical
findings, is found to be measles. She asks if her son is required to be excluded from school. She also mentions that according to the class teacher, all students in the class have complete vaccination against measles.

Which one of the following would be the most appropriate advice regarding exclusion from school for this boy?

A. No exclusion is required.
B. Exclusion for 4 days.
C. Exclusion until the rash completely disappears.
D. Exclusion is required only if there is an immunocompromised child in the class room.
E. Exclusion for 6 days

Infectious Disease, Vaccination, Exclusion

A

Correct Answer Is A.

Measles is highly contagious, and certain measures are necessary to prevent its spread, especially in childcare settings. Here are the guidelines for excluding contacts of a child with measles:

  1. Immunized Contacts:
    • Immunized individuals do not require exclusion, as they are protected against measles.
  2. Non-Immunized Contacts:
    • Non-immunized children who have been in contact with a child with measles should be excluded from school.
    • They should remain excluded until 14 days after the infected child developed the rash, unless they receive the MMR vaccine within 72 hours of exposure.
  • Index Case: John developed a rash 3 days ago.
  • Contacts:
    • Ana: Had measles before and is immune now. No exclusion needed.
    • Josh: Up-to-date with vaccinations. No exclusion needed.
    • Alan: Not immunized. Needs exclusion for 11 days from today (3 days since rash onset + 11 days = 14 days total).
    • Mary: Not immunized. Needs exclusion for 11 days from today (same calculation as Alan).
  • Immunized: No exclusion necessary.
  • Non-Immunized: Excluded from school for 14 days after exposure unless vaccinated within 72 hours.

These guidelines help prevent the spread of measles among children in childcare settings by ensuring appropriate exclusion of non-immunized individuals.

Measles is highly infective and all suspected, probable and confirmed cases should be excluded from work, school, and early childhood education and care services. Patients with established measles should be excluded until 4 days after the onset of rash. This child has had eruption of his rash 7 days ago; therefore, he is not infective anymore and exclusion is not required.

TOPIC REVIEW

Since measles is highly infective, it is very important that persons who has been in contact with a person with measles (index case) are traced and excluded from contacts to others accordingly. It is even more crucial in childcare setting. The following are recommendations regarding exclusion of contacts of known case of measles in
childcare setting:
* Immunized contacts of a child with measles do not require exclusion.
* Non-immunized children, who have been in contact with a child with measles should be excluded from school until 14 days after the day the infected child developed the rash, unless they are immunized with MMR within
72 hours of contact with the infected person.

Example: John develops a rash 3 days ago. There are four other children in his class: Alan, Josh, Mary, and Ana. These
children are all 7 years old. Ana contracted measles at the age of 3, and Josh has had all his childhood vaccines to date.
Alan and Mary have not received any vaccines because their parents are against immunization. So exclusion from
school for these children would be:

  • John (the index case): until tomorrow which is 4 days after the onset of the rash (3 days ago).
  • Ana (contact): no exclusion is required because she had measles and is immune now.
  • Josh (contact): no exclusion is required because he is up-to-date with his vaccination.
  • Alan (contact): he is not immune so he should be excluded from school for 11 days from today because John developed the rash 3 days ago.
  • Mary (contact): she is not immune so she should be excluded from school for 11 days from today because John developed the rash 3 days ago.

How well did you know this?

1

Not at all

2

3

4

5

Perfectly

5

Q

A 5-year-old boy is presented to your GP clinic by his parents with intermittent abdominal pain and vomiting usually lasting for 12 hours. The last painful episode was felt in the left flank. Physical examination is quite inconclusive.

You order a urine analysis and culture, the results of which are negative. Which one of the following investigations is most likely to reveal the underlying cause of this presentation?

A. Plain abdominal films.
B. Small intestine barium meal and follow-through.
C. Serum creatinine and BUN.
D. Serum electrolytes.
E. Ultrasound scan during a painful episode.

Urology

A

Correct Answer Is E.

Intermittent abdominal pain in children has a myriad of causes. In approximately 40%, no apparent cause is found after extensive investigations. Such pains are referred to as functional abdominal pain.

In older children, periodic abdominal pain with vomiting is a common symptom. These symptoms are probably caused by intermittent kinking of the UPJ. If obstruction does not resolve, severe hydronephrosis and pelvic distention can follow that presents with flank pain or even tenderness. Although rarely seen in developed countries, massive dilation of the renal pelvis and kidney can fill the entire flank and abdomen.

In younger adults, the typical presentation is episodic flank pain following diuresis.
Ultrasonography is the initial diagnostic choice for most cases of hydronephrosis and UPJ obstruction. Since the condition is present intermittently, sonography during a painful attack is most likely to delineate the diagnosis.

(Option A and B) Abdominal X-ray films and barium and follow through are indicated if intussusception is the suspected diagnosis. Intussusception can present with intermittent abdominal pain; however, most cases present
within the first 2 years of life (2 months -2 years). This child is 5 years old. Moreover, flank pain is unlikely to be a presentation of intussusception.

(Options C and D) Electrolytes, BUN, and creatinine might be indicated for evaluation of kidney function but they are
of little, if any, diagnostic value.

How well did you know this?

1

Not at all

2

3

4

5

Perfectly

6

Q

A 4-year-old is boy is brought to your attention with complaints of intermittent abdominal pain. Each episode lasted for a maximum of 2 to 3 hours before it completely subsided. He has been quite asymptomatic in between the episodes. This time, however, the pain has lasted for 12 hours. On examination, he is afebrile but a mass is palpated in the right upper quadrant (RUQ). The mass is not tender. Which one of the following could be the most likely
diagnosis?

A. Hepatoblastoma.
B. Neuroblastoma.
C. Wilms tumor.
D. Pyelonephritis.
E. Pancreatic tumor.

A

Correct Answer Is C.

Key Points:
- Age: Typically diagnosed at 2-3 years.
- Symptoms: Usually presents with an abdominal mass/swelling, often without other symptoms. May have abdominal pain, hematuria, fever, and hypertension.
- Physical Exam: Smooth, firm, non-tender abdominal mass that usually does not cross the midline.
- Genetics: Associated with WT1 gene on chromosome 11; mutation in ~20% of cases.
- Sporadic vs. Familial: Most cases are sporadic; few have a family history.

Differential Diagnosis:
- Hepatoblastoma: Rare liver malignancy in children; usually asymptomatic but can present with anorexia and severe osteopenia.
- Neuroblastoma: Common solid extracranial tumor in children; usually diagnosed before age 2. Abdominal mass is hard, irregular, and non-tender, often crossing the midline.
- Pyelonephritis: Associated with costovertebral angle tenderness and fever, not a palpable mass.
- Pancreatic Tumors: Extremely rare in children.

Summary:
Wilms tumor is the most likely diagnosis for a child presenting with an abdominal mass, typically without other symptoms. It is most commonly seen in children aged 2-3 years and has distinct physical and genetic characteristics.

The scenario is consistent with Wilms tumor (nephroblastoma) as the most likely diagnosis. Wilms tumor is the most common intraabdominal tumor of childhood that often is diagnosed at 2 to 3 years of age.

Most cases are sporadic, and only a few percent have a family history. Wilms tumor suppressor gene, WT1, is located on chromosome 11 and regulates normal kidney development. In approximately 20% of cases with Wilms tumor, there is a mutation of this gene.

Most children with Wilms tumor presents with abdominal mass or swelling without other signs and symptoms. Other symptoms, if present, can include abdominal pain (between 25-40% according to different studies), hematuria (12-25%), fever, and hypertension (25%). Hypertension is the result of the tumor compressing the renal artery, renal hypoperfusion, and activation of renin-angiotensin-aldosterone system.
The characteristic finding on examination is a smooth firm non-tender palpable abdominal mass that usually does not cross the midline.

(Option A) Hepatoblastoma is rare hepatic malignancy in children. Patients with hepatoblastoma are usually asymptomatic. The disease is advanced at diagnosis in approximately 40% of patients, and 20% have pulmonary metastases. Children with advanced disease may have anorexia. Severe osteopenia is present in most patients and regresses with resection of the tumor; osteopenia is often asymptomatic. Rarely, patients in whom the tumor has
ruptured present with symptoms consistent with acute abdomen. Occasionally, patients present with severe anemia resulting from tumor rupture and hemorrhage.

(Option B) Neuroblastoma, which is almost exclusively a disease of children, is the third most common childhood cancer after leukemia and brain tumors, and is the most common solid extracranial tumor in children. Neuroblastoma can also present with an abdominal mass, but the condition is expected to be diagnosed earlier compared to Wilms tumor, usually before the age of 2 years. Two-thirds (~65%) of neuroblastomas arise within the abdomen, of these 2/3 originate from adrenal glands. The abdominal mass seen in neuroblastoma is hard, irregular and non-tender and can extend beyond the midline. Other symptoms may include loss of appetite and weight loss, malaise, protrusion of one or both eyes. Other symptoms may be caused by compression effect the of tumor or metastases on the adjacent structures.

(Option D) Pyelonephritis can be associated with costovertebral angle tenderness but not a palpable mass. Fever is often a feature. Most importantly, it does not manifest as long standing intermittent abdominal pain with full resolution.

(Option E) Pancreatic tumors are extremely rare in children.

How well did you know this?

1

Not at all

2

3

4

5

Perfectly

7

Q

A 6-year-old girl is brought to your practice by her parents with a 4-day history of bilateral knee and ankle pain. On examination, she has a temperature of 37.6 C. Her joints are tender but there is no swelling, inflammation, or synovial thickening. The full range of motion of the joints is preserved. Palpable purpuric rashes are noted on her legs and buttocks with some ecchymoses around her ankles. Full blood count, blood urea, electrolytes and liver function tests are normal. ESR is 10mm/hr. Which one of the following is a known complication of this girl’s condition?

A. Pleurisy.
B. Fasciitis.
C. Nephritis.
D. Thrombocytopenia.
E. Diabetes mellitus.

A

Correct Answer Is C.

The clinical picture is highly suggestive of Henӧch-Schonlein purpura (HSP).

HSP, also known as anaphylactoid purpura, is an acute immune-mediated vasculitis of unknown cause. It is characterized by arthralgia (not arthritis), and non-thrombocytopenic purpura (the purpura is vascular due to leukoclastic activity) with typical distribution over legs and buttock.

Other clinical manifestations HSP include colicky abdominal pain, melena, swelling of the joints especially the ankles, and nephritis. The arthralgia and abdominal pain may persist for 2 to 4 weeks.

Nephritis of HSP, which is histologically the same as the IgA nephropathy, may occur in a small percentage of patients. For this reason, all patients with HSP should have a urinalysis. If the patient is found to have hematuria, long-term follow-up is required.

How well did you know this?

1

Not at all

2

3

4

5

Perfectly

8

Q

A 4-year-old child is brought to your practice with complaint of intermittent right upper quadrant (RUQ) and right flank pain for 2 years. These painful episodes frequently have been associated with vomiting and lasted for 3 hours. He has been well in between the episodes. This time, the pain has lasted for 12 hours. On examination, a mass is palpated in the RUQ. The mass is tender to palpation. Which one of the following is the most likely diagnosis?

A. Wilms tumor.
B. Neuroblastoma.
C. Ureteropelvic junction obstruction.
D. Vesicoureteral reflux (VUR).
E. Pyelonephritis.

A

Correct Answer Is C.

Intermittent abdominal pain in children is a frequent reason of seeking medical attention by concerned parents. The causes of intermittent abdominal pain are diverse, ranging from benign conditions such as functional abdominal pain to grave diseases such as childhood malignancies.

As usual, a thorough history and physical examination is the cornerstone of approach to such children. Associated symptoms can also help narrow down the differential diagnoses.

Of the options, ureteropelvic junction (UPJ) obstruction is the only likely diagnosis justifying the clinical scenario. UPJ obstruction is by far the most common cause of pediatric hydronephrosis, occurring in 1 per 1000-2000
newborns. Widespread use of antenatal ultrasonography and the advent of modern imaging techniques have resulted in earlier and more common diagnosis of the condition.
In older children, periodic abdominal pain with vomiting is a common symptom. These symptoms are probably caused by intermittent kinking of the UPJ. If obstruction does not resolve, severe hydronephrosis and pelvic distention can follow, presenting with flank pain or even tenderness. Although rarely seen in developed countries, massive dilation of the renal pelvis and kidney can fill the entire flank and abdomen.

In younger adults the typical presentation is episodic flank pain following diuresis.

(Option A) Wilms tumor is the most common intra-abdominal malignant tumor of childhood that often presents with
a smooth firm abdominal mass that usually do not cross the midline. Abdominal pain, either constant and vague or intermittent, is another finding. Hematuria and hypertension are other possible manifestations. Unlike the mass in the scenario, the abdominal mass found in Wilms tumor is non-tender, making Wilms tumor a less likely diagnosis.

(Option B) Neuroblastoma is the most common extracranial malignant tumor of childhood. The initial presentation is
most commonly within the first 2 years of life. The tumor is intraabdominal in two-thirds of cases. Of these cases, two-thirds originating from adrenal glands. It can also present with a palpable abdominal mass that is non-tender.
However, the age of the child and the tenderness of the mass are against neuroblastoma as the diagnosis.

(Options D and E) Based on physical finding of a palpable mass, pyelonephritis or vesicoureteral reflux are very unlikely to be the diagnosis as, these do not cause a palpable abdominal mass.

How well did you know this?

1

Not at all

2

3

4

5

Perfectly

9

Q

Mother of a 6-month-old boy brings him for evaluation after she felt a hard mass in the left hypochondrium while she was bathing him. The mass is non-tender. Which one of the following could be the most likely diagnosis?

A. Wilms tumor.
B. Neuroblastoma.
C. Polycystic kidney disease.
D. Ureteropelvic junction obstruction.
E. Hepatoblastoma.

A

Correct Answer Is B.
In approach to a child with an abdominal mass two possibilities should always be considered: (1) Wilms tumor
(nephroblastoma) and (2) neuroblastoma.

Wilms tumor is the most common intra-abdominal malignant tumor of childhood that often presents with a smooth firm abdominal mass that usually do not cross the midline. Abdominal pain either constant and vague or intermittent is another finding. Hematuria and hypertension are other possible manifestations. The median age of diagnosis is 3.5 years. Although not impossible, it is less likely to present within the first year of life.
Neuroblastoma is the most common extracranial malignant tumor of childhood with the initial presentation most commonly being within the first 2 years of life. The tumor is intraabdominal in two-thirds of cases, of these twothirds originating from adrenal glands. It can also present with a palpable abdominal mass that is non-tender.
Given the age, neuroblastoma is more likely than Wilms tumor to be diagnosis. Wilms tumor and less frequently neuroblastoma are often found when the child is being bathed or hugged.

(Option A) Given the age of the baby, Wilms tumor is a less likely dagnosis compared to neuroblastoma.

(Option C) An enlarged cystic kidney may be palpated as an abdominal mass in patients with polycystic kidney disease, but the mass is not hard.

(Option D) Ureteropelvic junction obstruction is not associated with a hard mass. In fact, if a mass is present, it is the significantly dilated kidney which is not hard, but can be tender.

(Option E) Hepatoblastoma is a rare hepatic malignancy in children. It does not give rise to a hard mass on the left side.

How well did you know this?

1

Not at all

2

3

4

5

Perfectly

10

Q

A previously healthy 18-month boy is brought to the Emergency Department with wheezing and cough starting 24 hours ago. On examination, he has a respiratory rate of 35 breaths per minute. Bilateral wheeze is evident on chest auscultation, as well as mildly diminished breath sounds of the left hemithorax. There is family history of asthma, as well as allergy in history. Which one of the following is the most appropriate next step in management of this child?

A. Nebulized salbutamol 4-hourly.
B. Amoxicillin.
C. Admission and administration of oxygen.
D. Chest physiotherapy.
E. Chest X-ray (CXR).

A

C. Admission and administration of oxygen.

Wheezing in children have a multitude of causes. The age of the child, physical findings, and the pattern and associations can help narrow down the differential diagnoses.

For instance, wheezing associated with feeding points towards reflux disease as the most likely cause, while wheeze and cough may suggest asthma. Positional wheeze, for example when the child is supine, is suggestive of laryngomalacia, whereas, wheeze in the presence of fever may be due to infections. Sudden onset unilateral
wheezing is a strong clue to foreign body aspiration.

The age of onset is another important clue to the diagnosis. Early onset wheeze can reflect a congenital problem.

Bronchiolitis is a very common cause of wheezing in children younger than 12 months, and asthma is the most
common cause of episodic wheezing in children younger than 5 years. Despite this, a definite diagnosis of asthma is avoided at this age group and other terms such as wheeze syndrom, viral wheeze, etc. are used instead.

In this child, wheezing and cough is of sudden onset, suggesting aspiration; however, bilaterality of the wheeze on exam makes this diagnosis very unlikely. The presence of cough and the strong family history of asthma and allergy put asthma at the top of the differential diagnoses list. This child though has also diminished breath sounds of the right lung that may suggest collection either in the lung or pleural space. This child needs to be admitted and through investigation carried out; however, making sure of adequate oxygenation regardless of the diagnosis comes first.

(Option A) Based on the provisional diagnosis of asthma, a trial of salbutamol is required both diagnostically and therapeutically because adequate response to bronchodilators is in favor of a diagnosis of asthma. This should be performed after oxygenation has started. Salbutamol is given by nebulizer in this age group. An adequate response to therapy almost clinches the diagnosis with high certainty.

(Option B) Antibiotics such as amoxicillin may be necessary later on if investigations suggest bacterial infections such as pneumonia.

(Option D) Chest physiotherapy is of little, if any, value at this point.

(Option E) CXR is indicated in children who present with unexplained wheezing that is unresponsive to bronchodilators or with recurrent wheezing. It is not indicated in this child who has presented with first-time wheeze that is very likely to be due to asthma. It, however, might be indicated if the wheeze is proved to be due to other causes than asthma, e.g., infections, foreign body, etc.

How well did you know this?

1

Not at all

2

3

4

5

Perfectly

11

Q

Which one of the following is the most common cause of failure to breastfeed?

A. Exhaustion of the mother.
B. Reduced frequency of breastfeeding.
C. A reduction in the time the baby feeds at each session.
D. Maternal dehydration.
E. Nipple problems.

A

Correct Answer Is B.

The benefits of breastfeeding for infants and mothers are well proven. Recommendations to breastfeed exclusively for 6 months (EBF6) have been widely adopted by relevant organizations in Australia.

Advantages of breastfeeding for the child include increased resistance to diseases, lower rates of diseases such as gastroenteritis, recurrent otitis media, and pneumonia. Additionally, there are lower rates of sudden infant death syndrome (SIDS), colitis, hypertension, obesity, hyperlipidemia, atopic disease and diabetes, and a higher IQ.
For the mother, there is a delay in ovulation, decreased risks of breast and ovarian cancer and the bonding effects of breastfeeding.
For many women, difficulties in breastfeeding result in early termination of breastfeeding before the recommended period of 6 months.

By far, inadequate milk intake or the perception of inadequate milk production is the most common reason for early
termination of breastfeeding. Inadequate milk intake may be due to failure of the infant to extract milk or insufficient milk production.

Inappropriate early feeding routines are the most common cause of insufficient milk intake. They include infrequent feeding, poor latch-on, maternal-infant separation, and the use of supplemental formula.

Other less common causes include oral-motor or neurologic abnormalities of the baby and poor emptying of the breast.

Of the given options, reduced frequency of breastfeeding is the most common cause to decreased milk production.

With infrequent feeds, milk production reduces resulting in both inadequate milk production by the mother and intake
by the baby.

(Option A) Maternal exhaustion can contribute to decreased frequency of breastfeeding and eventually failure of
lactation.

(Option C) Reduced time of latching during each feed does not have such significant negative impact on milk
production as has reduced frequency of feeds.

(Option D) Maternal dehydration can be associated with decreased milk production but is not as common a cause as
is infrequent feeding.

(Option E) Nipple problems can prevent from appropriate and adequate breastfeeding; however, infrequent feeds
remain the most common cause of failure to breastfeed.

How well did you know this?

1

Not at all

2

3

4

5

Perfectly

12

Q

A 10-year-old girl is being assessed in your clinic with complaints of labial fusion. She has the past history of vulvovaginitis which has been successfully treated. The child reports no voiding difficulties. Which one of the following is the most appropriate next step in management?

A. Reassurance.
B. Surgical separation of adhesions.
C. Encourage good hygiene.
D. Antibiotics.
E. Manual separation of adhesions.

A

Correct Answer Is A.
Labial adhesion (labial agglutination) occurs when the medial edges of the labia minora becomes adherent, often in the background history of vulvovaginitis. This is probably due to combination of thin vagin*l mucosa (the normal prepubescent state) and minor irritation (such as in vulvovaginitis).

This is a normal variant and will resolve spontaneously in late childhood. Provided that the child is able to void easily, no treatment other than reassurance is required.

(Option B) Surgical separation of the adhesions, followed by application of Vaseline and/or estrogen cream can be considered for children with urinary symptoms. This method, however, is not generally recommended because of the high recurrence rates.

(Option C) Encouraging good hygiene, althoug a good advice, is not effective in treatment of the condition.

(Option D) Antibitoics have no role in management of labial adhesions.

(Option E) Manual separation of adhesions can be distressing for the child and is associated with a high risk of recurrence and not recommended.

How well did you know this?

1

Not at all

2

3

4

5

Perfectly

13

Q

A 9-year-old boy is brought to your attention by his parents because of poor exercise tolerance. He is a member of the school basketball team but recently, it has been hard for him to play because he becomes short of breath and tired soon after he starts playing. On examination, his height is at 97 percentile and weight at 3 percentile. Which one of the following is the most important step in management?

A. Karyotyping.
B. Bone scan.
C. Cardiac ultrasound.
D. Growth hormone test.
E. CT scan.

A

Correct Answer Is C.

Marfan syndrome (MFS) should always be considered as a probable diagnosis in children with tall stature and low
weight, MFS is an autosomal dominant connective tissue disorder with involvement of the cardiovascular, skeletal and
ocular systems, as well as the skin, lungs and dura. The prevalence is at least 1 in 5000. In 90-93% of cases, MFS is caused by mutations in FBN1. In a minority, MFS is caused by mutations in a second gene called TGFBR2. MFS caused by this mutation is termed MFS2. Based on current statistics, MFS2 accounts for up to 10% of cases with MFS.

Cardinal manifestations include aortic aneurysm and dissection, ocular lens dislocation and long bone overgrowth.
Cardiac disease is the leading cause of morbidity and mortality in patients with MFS. When the syndrome is suspected based on clinical grounds, an echocardiogram (cardiac ultrasound scan) should follow as the most
appropriate next step in management for evaluation of aortic root dilation (the most important) and mitral valve prolapse – conditions frequently seen in patients with MFS. The former is more common and significant clinically.

Serial echocardiographic surveillance is indicated for all affected individuals. Frequency should be tailored to each
individual by their cardiologist. Based on current evidence, the use of beta blockers remains the first-line treatment (except where contra-indicated e.g., in asthmatic patients) in aortic dilatation in MFS even in young children if a diagnosis of MFS is clear, or if there is a known FBN1 mutation in a young child without clinical features of MFS but with affected first degree relative(s) with a known mutation and aortic root dilatation. If beta blockers are ineffective or contraindicated, verapamil or ACE inhibitors would be the appropriate second-line treatment options.

How well did you know this?

1

Not at all

2

3

4

5

Perfectly

14

Q

A 14-year-old boy has clinically apparent Marfan syndrome. Which of the following cardiac murmurs would you expect to hear on cardiac auscultation?

A. Midsystolic ejection murmur at the second right intercostal space.
B. Decrescendo high-pitched diastolic murmur at the left sternal edge.
C. Low-pitched rumbling diastolic murmur at the apex.
D. Pansystolic murmur at the left sternal edge with no radiation.
E. Continuous machinery murmur at the second left intercostal space.

Cardiology, Murmurs

A

Correct Answer Is B.

Aortic Root Disease in Marfan Syndrome (MFS)

  • Common Cardiac Issues: Aneurysmal dilation, aortic regurgitation (AR), and aortic dissection.
  • Impact: Leading causes of mortality and morbidity in MFS.

Aortic Dilation:
- Found in ~50% of children and 60-80% of adults with MFS.
- Often associated with AR.

Aortic Regurgitation (AR):
- Murmur: Decrescendo murmur during diastole, high-pitched, loudest at the left sternal border.
- Severity: Duration of the murmur correlates with severity, not loudness.

Other Murmurs in MFS with AR:
- Austin-Flint Murmur: Mid-diastolic rumbling murmur at the apex, caused by severe AR.
- Functional Systolic Flow Murmur: Due to increased stroke volume.

Mitral Valve Prolapse in MFS:
- Auscultation Findings: Mid-to-late systolic click, sometimes followed by a high-pitched mid-to-late systolic murmur at the apex.

Differential Murmurs:
- Option A: Midsystolic ejection murmur at the second right intercostal space (aortic stenosis, hypertrophic obstructive cardiomyopathy).
- Option C: Austin-Flint murmur (mid-diastolic rumbling at the apex) in severe AR.
- Option D: Pansystolic murmur at the left sternal edge (ventricular septal defect).
- Option E: Continuous machinery murmur at the second left intercostal space (patent ductus arteriosus, PDA).

  • AR Murmur: Decrescendo, high-pitched, diastolic, loudest at the left sternal border.
  • Austin-Flint Murmur: Mid-diastolic rumbling at the apex in severe AR.
  • Mitral Valve Prolapse: Mid-to-late systolic click and possible murmur at the apex.

This should help in remembering the key cardiac manifestations and murmurs associated with Marfan syndrome and aortic root disease.

Aortic root disease is the most common cardiac manifestations in Marfan syndrome (MFS). This results in aneurysmal dilation, aortic regurgitation and aortic dissection. Such conditions are the most common causes of mortality and morbidity in patients with MFS.

Dilation of the aorta is found in approximately 50% of children and 60-80% of adults with MFS. Aortic regurgitation (AR) is often present. On cardiac exam, murmurs caused by chronic AR are most likely to be heard.

The murmur of AR is a decrescendo murmur occurring during diastole usually as a high-pitched sound that is loudest at the left sternal border. The duration of the murmur correlates with the severity of AR; loudness of the
murmur does not.

Other possible murmurs in patients with MFS and AR include:
Austin-Flint murmur - During diastole, blood jet backs through the incompetent aortic valve and strikes anterior leaflet of the mitral valve and results in premature disclosure of this valve. This produces a middiastolic rumbling murmur best heard over the apex. Austin-Flint murmur is heard in patients with severe AR.

A functional systolic flow murmur may also be present due to increased stroke volume.

NOTE - Patients with MFS may also have mitral valve prolapse. The classic auscultation finding in mitral valve prolapse is a mid-to-late systolic click, which is present due to the leaflets prolapsing into the left atrium resulting in tensing of the mitral valve apparatus. This click may or may not be followed by a high-pitched, mid-to-late systolic
murmur at the cardiac apex.

(Option A) Midsystolic ejection murmur at the second right intercostal space is the characteristic finding in aortic
stenosis and hypertrophic obstructive cardiomyopathy (HOCM).

(Option C) A low-pitched rumbling mid-diastolic murmur over the apex (Austin-Flint murmur) is also possible in patients with MFS with severe AR. this is, however, a less common finding.

(Option D) Pansystolic murmur at the left sternal edge with no radiation is characteristic for ventricular septal defect.

(Option E) A continuous machinery murmur at the second left intercostal space is characteristic of patent ductus arteriosus (PDA).

How well did you know this?

1

Not at all

2

3

4

5

Perfectly

15

Q

A 5-week-old female infant is brought to your office by her parents because of ‘feeding problems’. Parents tell you
that their child has been vomiting after many of her feeds for the past week or so. They mention that this is a new problem, and she was previously quite healthy. For the first time, the baby’s vomit was projectile and forceful last night. On examination, a mass is palpated in the epigastric area. Which one of the following, can the most likely
metabolic profile in this infant?

A. Increased pH, increased K, increased Cl, increased pCO2, and increased HCO3.
B. Decreased pH, decreased K, decreased Cl, increased pCO2, and decreased HCO3.
C. Increased pH, increased K, decreased Cl, increased pCO2, and decreased HCO3.
D. Increased pH, decreased K, decreased Cl, increased pCO2, and increased HCO3.
E. Increased pH, decreased K, decreased Cl, decreased pCO2, and decreased HCO3

A

Correct Answer Is D.

The scenario describes a classic presentation of infantile hypertrophic pyloric stenosis (IHPS). IHPS is more common in males than females (4:1 to 6:1).

IHPS typically presents with immediate post-prandial non-bilious projectile vomiting in a 3- to 6-week old baby. After vomit, the baby demands to be re-fed immediately (hungry vomiter).

On physical examination, the baby is dehydrated with a palpable ‘olive-like’ mass at lateral edge of the rectus abdominus muscle in the right upper quadrant of the abdomen. The mass is most easily felt immediately after
vomiting.

Gastric fluid is rich in hydrochloric acid and potassium. Loss of gastric fluid by emesis results is hypochloremic metabolic alkalosis. Initially the potassium is normal, but as emesis persists hypokalemia develops. Metabolic alkalosis is associated with increased HCO3. To compensate the metabolic alkalosis, respiratory acidosis occurs that is associated with increased arterial pressure of CO2 (pCO2).

Given these, the expected metabolic picture in a patient with prolonged emesis will be increased PH (metabolic alkalosis), decreased serum potassium and chloride, increased HCO3 and increased PCO2.

How well did you know this?

1

Not at all

2

3

4

5

Perfectly

16

Q

Kyle, 5 months old, is brought to your practice by his mother for evaluation. He has had a runny nose and a fever or 40°C for the past 48 hours, but otherwise quite active and healthy. Today, Kyle’s temperature dropped to normal but he developed a maculopapular rash on his trunk. He is still in good health. Which one of the following is the most likely diagnosis?

A. Measles.
B. Chicken pox.
C. Erythema infectiosum.
D. Rubella.
E. Roseola infantum.

A

Correct Answer Is E.

The scenario is typical for roseola infantum as the diagnosis. Roseola infantum, also known as exanthem subitum, sixth disease, and three-day fever, is a clinical syndrome most frequently caused by human herpes virus 6. Other causes include human herpes virus 7 (HHV-7), enteroviruses (coxackievirus A and B, echovirus), adenovirus, and parainfluenza virus type 1.

It is a clinical syndrome of infants (mostly 6-18 months) characterized by 3 to 5 days of high fever of commonly up to 40°C (or even higher). The fever then resolves abruptly, followed by development of a rash.
.
Typical rash of roseola infantum is a macular or maculopapular rash, starting on the neck and trunk and spreading to the face and extremities. Uncommonly, the rash is vesicular. It is unusual for the rash to be pruritic. The rash typically persists for 1 to 2 days, but occasionally may come and go within 2 to 4 hours. In children receiving antibiotics due to the high fever, the onset of rash can be confused with drug allergy. Diagnosis is almost always clinical. The disease has a benign course and resolves spontaneously. Febrile convulsions may occur.

Lymphadenopathy and erythematous tympanic membrane are common. During the febrile phase, the disease can be confused with otitis media. The combination of high fever and bulging fontanelle occurs in as many as 26% of infants and can be mistaken for meningitis.

How well did you know this?

1

Not at all

2

3

4

5

Perfectly

17

Q

Which one of the following is suggestive of hypertrophic pyloric stenosis?

A. Vomiting after 1 hour of feeding.
B. Vomiting immediately after feeding.
C. Vomiting after few minutes of feeding.
D. Bilious vomiting.
E. Vomiting after 2 hours of feeding.

A

Correct Answer Is C.
Hypertrophic pyloric stenosis (HPS) is caused by progressive thickening of the circular muscle of the pylorus. This leads to gastric outlet narrowing. The condition usually presents between 2-4 and 6-9 weeks of age.

Predisposing factors to HPS are:
* Male gender
* First born
* Caucasian
* Parental history of HPS (higher if mother affected)

HPS typically presets initially with non-bloody, nonbilious vomiting at 2-9 (typically 4-8) weeks of age. Vomiting may initially be infrequent, but over several days it becomes more predictable, occurring at nearly every feeding.
The vomiting occurs within the first hour after meals, but the typical time is within the first few minutes (up to 30 minutes). Vomiting intensity increases until pathognomonic projectile vomiting develops.

Slight hematemesis of either bright-red flecks or a coffee-ground appearance is sometimes observed. Patients are usually not ill-looking or febrile. The baby in the early stage of the disease remains hungry and sucks vigorously after episodes of vomiting (hungry vomiter).

Prolonged delay in diagnosis can lead to dehydration, poor weight gain, malnutrition, metabolic alterations, and lethargy. Parents often report trying several different baby formulas because they (or their physicians) assume vomiting is due to intolerance.

How well did you know this?

1

Not at all

2

3

4

5

Perfectly

18

Q

A 13-year-old boy presents to his general practitioner, accompanied by his mother, complaining of poorly localized pain in his knee for the past 6 weeks. His mother has observed a limp for the same period of time. He walks with his foot externally rotated. Examination of the knee is normal. Which one of the following is the most likely diagnosis?

A. Osteochondritis dissecans of the knee.
B. Perthes disease of the hip.
C. Undiagnosed congenital dislocation of the hip.
D. Slipped capital femoral epiphysis (SCFE).
E. Osgood-Schlatter osteochondritis of the knee.

A

Correct Answer Is D.

The clinical features described, as well as the age of the patient suggests slipped capital femoral epiphysis (SCFE) as the most likely diagnosis.

SCFE is more commonly seen in adolescents of 10 to 15 years of age. The classic case would be an oversized prepubertal boy. The condition is bilateral in 20% of cases.

SCFE presents with the following:
* Limp and irritability of hip on movement
* Knee pain – referred from the affected hip
* On flexion of the hip, it rotates externally.
* Hip is often in external rotation on walking.
* Most movements restricted, especially internal rotation.

The first symptom is hip stiffness that subsides with rest. Later on, limping and hip pain radiating down the anteromedial thigh to knee follows. Early hip examination neither detects pain nor movement limitation. In more advanced stages, hip movements become painful and there is decreased flexion, abduction and internal rotation.
The affected leg is externally rotated on walking.

The most significant aspect of the SCFE is the great number of patients who develop avascular necrosis of the femoral head despite expert treatment. Therefore, diagnosis of the condition before major slipping occurs is important. This necessitates early investigation and referral.

Any adolescent with a limp or knee pain should have X-rays (AP and frog view) of both hips. Otherwise, this important condition will be overlooked.

The most important management principles include:
* Cease weight-bearing and refer urgently.
* If acute slip, gentle reduction via traction is better than manipulation for prevention of later avascular necrosis.
* Once reduced, pinning is performed.

(Option A) Osteochondritis dissecans is characterized by separation of an osteochondral fragment from the articular
surface. The underlying bone from which the fragment separates has normal vascularity. Knee pain and swelling are clinical features of the disease. Hip is not affected.

(Option B)** Legg-Calvé-Perthes disease**, also called Perthes disease, is a temporary condition in the hip joint characterized by decreased blood supply to the femoral head and consequent avascular necrosis. As a result, femoral head collapses and the area becomes inflamed and irritated. Legg-Calvé-Perthes disease causes the hip joint to become painful and stiff. Affected children are usually between 4 and 10 years old, physically active and small for their age.

(Option C) It is very unlikely for congenital dislocation of the hip to remain asymptomatic until this age. The recent onset of the problem makes this diagnosis unlikely.

(Option E) Osgood-Schlatter disease is seen in preadolescent children and is characterized by pain localized to the tibial tubercle and occasionally the patellar tendon. The pathophysiology of the disease is by repetitive traction effect of patellar tendon on an immature tibial tubercle. There is often tenderness over the tibial tubercle on examination. Hip movements are not painful or restricted.

How well did you know this?

1

Not at all

2

3

4

5

Perfectly

19

Q

A 4-year-old boy is brought to your practice by his parents because of what they think to be vertigo. The boy suddenly starts feeling that the world is spinning around him. Each episodes lasts for about one to two minutes, while the child is fully conscious. This has happened once a month for the past three months. His past medical history isunremarkable. He has had no headache, nausea, or vomiting before, during, or after the attacks. Physical examination including ear exam is normal. Which one of the following is the most appropriate next step in management?

A. Reassure the parents as this is very likely to be benign paroxysmal positional vertigo (BPPV).
B. EEG and CT scan of the head.
C. Audiology and ENT referral.
D. MRI of the head.
E. Psychiatric assessment.

A

Correct Answer Is B.
Although dizziness and vertigo can be confused by patients, the spinning sensation described by the child is most likely true vertigo rather than dizziness. Unlike in adults, vertigo in children often has a sinister cause such as brain tumors (e.g., medulloblastoma) or temporal lobe epilepsy.
In every child with vertigo, epilepsy and CNS tumors should be excluded as the most appropriate next step in management, and early referral for specialist review must be considered.

In addition to epilepsy and CNS tumors, the following can also cause vertigo in children:
* CNS infections
* Trauma, especially to the temporal area
* Middle ear infections
* Prescription drugs
* Alcohol and other illicit drugs
* BPPV (rare)
* Labyrinthitis

(Option A) BPPV in children is considered a migraine variant of childhood and presents quite similar to this scenario.
However, BPPV is rare in children and is a diagnosis of exclusion, only made once other causes have been excluded. While CNS tumors are more common and serious diagnoses, reassuring the parents based on a hasty diagnosis of BPPV is inappropriate.

NOTE – BPPV in childhood often precedes migraine in adulthood.

(Option C) Generally, neurology referral (not ENT referral) is an appropriate and safe option to select because CNS tumors and epilepsy are the major differential diagnosis in such presentation in a child. Audiology assessment and ENT referral are appropriate once CNS-related causes have been safely excluded.

(Option D) MRI may be considered later if a CNS tumor is suspected during initial evaluation with CT scan.

(Option E) This child does not seem to have a psychiatric problem, and psychiatric assessment is not an appropriate
initial management option.

How well did you know this?

1

Not at all

2

3

4

5

Perfectly

20

Q

A 3-year-old girl is brought to your practice by her mother with complaint of multiple painful small oral ulcers. The child refuses to eat because of the pain. On examination, the child is found to have a temperature of 38°C. Similar ulcers are noted on his hand and feet. Which one of the following is the causative organism?

A. Herpes simplex virus (HSV).
B. Group B streptococcus (GBS).
C. Coxsackie A virus.
D. Adenovirus.
E. Coronavirus.

A

C. Coxsackie A Virus

The clinical picture of ulcers in mouth and on hands and feet is highly suggestive of ‘hand, foot and mouth (HFM) disease’. This often self-limiting mild viral infection is causes by Coxsackie A virus. The disease has an incubation period of 3 to 5 days. The illness begins with a prodromal syndrome of mild fever, headache and malaise, followed by
appearance of the rash after 1 to 2 days. The rash starts as an erythematous macular rash, progressing to gray vesicles with surrounding erythema. These vesicles form shallow ulcers on buccal mucosa, gums and tongue.

Hands and feet, especially the lateral borders, are affected as well. It is not uncommon to see the rash on buttocks and genitalia as well.
The rash resolves in 3 to 5 days with no scarring. The child is infectious until blisters disappear, but the virus can be secreted into saliva and feces.

How well did you know this?

1

Not at all

2

3

4

5

Perfectly

21

Q

A 10-year-old boy is referred to your clinic from his school for assessment of probable hearing deficit because he has difficulty in hearing resulting in dropped school performance. You carry out a Rinne test that is positive in both ears. A Weber test shows no lateralization; however, he has symmetrical decreased hearing threshold. Which one of
the following options could be the most likely diagnosis?

A. Presbycusis.
B. Otosclerosis.
C. Congenital sensorineural deafness.
D. Chronic perforation of the tympanic membrane.
E. Chronic secretory otitis media with effusion.

A

C. Congenital sensorineural deafness.

In clinical practice it is common to see patients complaining of hearing loss. It is useful to be able to have a simple test that can distinguish where the site of the cause of the hearing loss is. This can help guide the need for further examination, investigation and management. Rinne’s test and Weber’s test are done to differentiate between a conductive (middle and outer ear causes) and a sensorineural deafness (caused by damage to the cochlea or to the
8th nerve – or its central connections). These tests are always done together. The Rinne test is done first.
The function of the external ear is to collect sounds vibrations form the air and focus these onto the tympanic membrane. These vibrations are then transmitted through the middle ear cavity by the ossicular chain (Malleus, Incus and Stapes). The stapes transmits these vibrations to the cochlea through the oval window (fenestra ovalis).
Sound can also be transmitted through the bones of the skull to the cochlea. The hair cells in the cochlea convert the physical vibrations into action potentials that are transmitted via the nerves in the vestibulo-cochlear (auditory) nerve to the brainstem for further processing.

The Rinne and Weber tests are used for quick screening of conductive versus sensorineural hearing loss. The Rinne test is performed for evaluation of hearing loss in one ear (unilateral hearing loss) and compares perception of sound transmitted by air conduction to those transmitted by bone conduction through the mastoid. Therefore, the presence of conductive hearing loss is screened for.

For the Rinne test, a vibrating tuning fork (typically 512 Hz) is placed initially on the mastoid process behind each ear until sound is no longer heard, signaled by the patient. The fork is then immediately positioned just outside the ear with the patient asked to report when the sound caused by the vibration is no longer heard. A normal or positive Rinne test is when the sound heard outside the ear (air conduction or AC) is louder than the initial sound heard when
the tuning fork end is placed on mastoid process behind the ear (bone conduction or BC). Therefore, AC > BC; which is how it is reported clinically for a normal or positive Rinne result. In conductive hearing loss, bone conduction is better than air or BC > AC, a negative Rinne.

In the Weber test, a vibrating tuning fork (often 256Hz or 512Hz) is placed in the middle of the forehead, or above the upper lip under the nose over the teeth, or on top of the head in an equal distant from the patient’s ears. The patient is then asked to report in which ear the sound is louder. A Weber test result is normal (positive) if the patient reports the sound equally on both sides. Lateralization (hearing the sound better in one ear) is due to either a sensorineural or conductive problem. In a patient with sensorineural hearing loss, the normal ear hears the sound better than the defective one. In a patient with conductive hearing loss, the sound is lateralized to the affected side.

NOTE – Patients with symmetrical hearing loss have also a positive (normal) Weber test result because both ears are affected equally and no lateralization occurs despite the fact that there is hearing loss.

The results of these two tests are compare to localize and characterize the nature of any detected hearing loss.These test are, however, screening tests and are not replacements for formal audiometry.
In this boy, a positive (normal) Rinne test for both ears excludes conductive impairment. The Weber test is also positive for both ears indicating that there is no lateralization. Based on these findings, chronic perforation of the tympanic membrane (option D), chronic secretory otitis media (option E), and otosclerosis (option B) are excluded
from the differential diagnoses. This boy should have a hearing problem affecting both ears equally.

NOTE - Otosclerosis is a disease characterized by fusion of the stapes to incus resulting in conductive hearing loss. This disease is more common among young adult. This boy has normal Rinne test and this condition is unlikely to be the cause of his hearing loss.

Presbycusis (option A) is the most common cause of sensorineural hearing loss in aging individual. A 10-year-old boy is unlikely to have presbycusis.
Given the exam findings and the age of this child, congenital sensorineural hearing loss either in isolation or associated with a syndromic condition could be the most likely diagnosis among others.

How well did you know this?

1

Not at all

2

3

4

5

Perfectly

22

Q

A 15-year-old boy is brought by his parents for his behavioral problems. According to the parents, he has had marked aggressiveness with several fights and arguments with the school staff, anger outbursts, and being uneasy to control. He seemingly is not obedient at home and defies his parents and the home rules. Parents say that they have fights over every simple issue at home. They, however, deny any act of vandalism, cruelty towards animals or people,
or drug use. He has declined school performance. On examination, he has a normal attention span and does not appear hyperactive. If pharmacological intervention is required, which one of the following is most likely to benefit him?

A. Carbamazepine.
B. Sodium valproate.
C. SSRIs.
D. Risperidone.
E. Olanzapine.

A

Correct Answer Is D. Risperidone.

The clinical picture and the age of the child are mostly consistent with oppositional defiant disorder (ODD) as the diagnosis.

ODD is a disruptive behavior disorder in children and teenagers characterized by patterns of unruly and
argumentative behavior and attitudes toward authority figures. This pattern is often considered by parents simply as stubbornness, emotionalism, and strong will. However, in ODD the behavior is significantly more extreme than what is considered normal. Interestingly, ODD occurs much more frequently than the type of childhood stubbornness,
whininess, and rebellion that often occurs at different stages of a child’s development.

ODD manifests with a constant extremely negative, defiant, and hostile behavior leading to disruption of the social, school, and home life for at least 6 months. Symptoms may appear as early as the late preschool years.

Children with ODD often direct their anger and resentment toward their parents, teachers, and other authority figures; however, they can have such problems with their peers as well. They are often uncooperative, vindictive, and easily annoyed. They usually defy the rules, have anger outburst, blame others for their mistakes, seek revenge, and disturb
others on purpose. ODD symptoms may be directed at one person or many people, and may occur only at home, at school, or may occur in a number of settings.

Since the child is unlikely to understand that he has a problem, seeking treatment is often from the parents’ side. A careful history is essential because many other childhood conditions may have some shared features with ODD.

Also, ODD may co-exist with other psychiatric conditions such as anxiety disorder, ADHD, learning disorders and language disorders. It is important to differentiate whether the abnormality in the behavioral pattern is due to ODD or simply a response to a temporary situation.

The international Classification of Diseases 10
th Revision (ICD-10) classifies** ODD as a mild form of conduct disorder**. It has been estimated that up to 60% of patients with ODD will develop conduct disorder.

Treatment is necessary at earlier stages to prevent it from developing into a more serious conduct disorder (most important), mental health disorder, or criminal behavior. Treatment entails a combination of behavioral therapy, family therapy, and at occasions, medications.

In more than 50% of patients with ADHD, ODD is also part of the clinical picture. There is strong evidence suggesting that ODD and ADHD overlap and many medications that are used to treat ADHA may also be efficacious in the treatment of ODD too. A few studies have reported the positive effects of psychostimulants or atomoxetine in the treatment of ODD associated with ADHD. Patients with ODD and conduct disorder with severe aggression may well respond to risperidone, with or without psychostimulants.
Mood regulators, alpha2 agonists, and antidepressants may also have a second-line role in the treatment of ODD and its comorbidities.

Of the options, risperidone is the only option that can be used if pharmacotherapy is considered.

Mood stabilizers such as sodium valproate (option B) or olanzapine (option A), and antidepressants such as SSRIs (option C) are second-line options.

There is no evidence supporting carbamazepine (option E) as a pharmacological treatment for ODD.

23

Q

A 7-year-old boy presents with a generalized rash illustrated in the following photograph. The rash developed after 24 hours of mild malaise and fever. When he can go back to school?

A. After the resolution of the blisters.
B. After 2 days.
C. After 5 days.
D. No exclusion is required.
E. After clearance of the rash.

Normal & Abnormal Growth and Development/Child Health/Paediatrics Flashcards by Lubi Mupwaya (1)

A

Correct Answer: A. After the resolution of blisters.

The photograph shows blisters of varying stages, macules and papules. Of the blisters, some are intact, some unroofed and some dried. This, along with the history, is highly diagnostic for chicken pox.

Children with chickenpox should be excluded from school or other daycare setting until all blisters are dried out. This usually takes 5 days from the onset of the rash but may be less in previously immunized children.

Dried blisters take more time to clear, during which the child is not infectious. Hence, waiting until the complete clearance of the rash is not necessary.

How well did you know this?

1

Not at all

2

3

4

5

Perfectly

24

Q

The parents of a 7-year-old girl have brought her to you for assessment because she frequently soils her underwear. In the past 2 months, they have received calls from her school about her problems there. She is easily irritated and becomes angry and aggressive with her classmates and other children at school. At home, they found her soiled underwear hidden. She becomes angry, cries, and fights with her older brother when he calls her ‘smelly’. This happens several times a week. Which one of the following is the most likely diagnosis?

A. Conduct disorder.
B. Oppositional defiant disorder.
C. Regression.
D. Delayed developmental milestones.
E. Depression.

A

Correct Answer Is B, Oppositional Defiant Disorder.

The scenario suggests encopresis. However, full diagnostic criteria for such diagnosis is uncertain (frequency and duration of symptoms are lacking in the scenario).

Encopresis is the voluntary or involuntary passage of formed, semi-formed, or liquid stool into a place other than the toilet for more than one time per month in a child older than 4 years of age for at least 3 months. If the child has never been continent, the condition is termed primary encopresis, whereas secondary encopresis is fecal incontinence in a previously continent child. Encopresis is reported in 1-4% of school-aged children. Encopresis is
more common in boys than girls.

There are two different types of encopresis:
With constipation and overflow Incontinence: children with the constipation and overflow incontinence type have less than 3 bowel movements per week. Due to constipation, only part of the total available stool is emptied during each of these movements. Parts of the remaining stool leak out, often during the child’s daily activities. Once the constipation is resolved, the encopresis usually no longer exists.

Without constipation and overflow Incontinence: in this type, there is no constipation, and the child’s feces have normal consistency. Unlike in cases associated with constipation and overflow, soiling of this type is intermittent.

Feces may be emitted in a prominent location (e.g., as an act of defiance) or maybe an unintentional consequence of anal self-stimulation (e.g., a variety of masturbation). Encopresis without constipation and overflow incontinence is less common than the first type of encopresis and is often associated with oppositional defiant disorder and conduct disorder. It has been estimated that 3% of children with psychiatric issues may have encopresis.
With aggressiveness, anger outbursts, and being easily annoyed and irritated, oppositional defiant disorder is the most likely underlying cause of encopresis and the diagnosis. This girl has no history of violence, a serious breach of law, or vandalism to suggest conduct disorder (option A).

(Option C ) Regression is a defense mechanism characterized by the reversion of an individual’s personality to an earlier stage of development and adopting more childish mannerisms. In children, regression presents with returning to behaviors that they have already grown out of. Examples are wanting a bottle or pacifier, temper tantrums, whining, aggression, thumb-sucking, and baby talk. Enuresis (bet wetting) or encopresis may occur. Regression is
often triggered by a stressful life event such as bereavement, parental separation, or sexual abuse. In the absence of such history, regression is less likely as the diagnosis. Furthermore, the behavioral pattern of this child is more consistent with oppositional defiant disorder.

(Option D) Encopresis is unlikely to have been caused by delayed development in a girl who has otherwise fulfilled other developmental milestones.

(Option E) There is no strong link between depression and encopresis. Moreover, there are no other symptoms such as low mood, sadness, or altered sleep pattern and appetite to suggest depression as a diagnosis.

How well did you know this?

1

Not at all

2

3

4

5

Perfectly

25

Q

A 24-month female child is brought for assessment of development. Which one of the following is expected to develop at this age?

A. Knowing two pronouns.
B. Naming four colors.
C. Knowing her age.
D. Knowing her family name.
E. Speaking in full sentences.

A

Correct Answer Is A.

Of the options knowing pronouns and prepositions is a language developmental milestone that could have been achieved at this age. Other options are often achieved after the age of** 3** years (36 months)

(Option B) Naming some colors is the ability achieved between** 3 and 5 years** of age. It is too soon for this child for this.
(Option C) knowing the age is an ability achieved often after 5 years of age.
(Option D) Knowing the family name is an expected ability around the age of 4 years. Assessment should be considered if a child cannot say their family name by the age of 5 years.
(Option E) Speaking in full sentences using many words is a language development milestone normally achieved between the ages **3 and 5 **years. It would not be expected in a 24-month-old child.

How well did you know this?

1

Not at all

2

3

4

5

Perfectly

26

Q

Parents of a 14-month-old boy has brought him for evaluation because they think their baby lags behind language development milestones because he does not speak any words and only babbles. He is the outcome of an uneventful pregnancy and was born through an uncomplicated vagin*l delivery. He has not had any medical problems to date except few episodes of upper respiratory tract infections. He started rolling over at the age of 4 months, can sit without support from the age of 8 months and has started unaided walking recently. He transfers objects from one hand to the other and plays and enjoys peak-a-boo. Which one of the following this child have?

A. Language delay.
B. Social delay.
C. Normal development.
D. Fine motor delay.
E. Gross motor delay.

A

Correct Answer Is C.
Developmental milestones are categorized under the following developmental areas:
* Physical
* Social
* Emotional
* Cognitive
* Language

This child has been able to roll over since the age of 4 months (normal: 0-4 months), can sit without support from the age of 8 months (normal: 8-12 months), can transfer objects from one hand to the other (normal: 8-12 months) and has started walking without support (normal: 1-2 years). In terms of physical development, he is fully developed and does not have any fine motor (option D) or gross motor (option E) developmental delay. He plays peak-a-boo (normal: 4-8 months). This is an example of cognitive developmental milestone often achieved between 4-8 months of age.
There is no information regarding social developmental milestones in the question but parents do not seem to be concerned about it. This exclude social delay (option B) as his problem as parents are first to suspect something wrong in their baby and they are often right for that matter.
In fact the parent’s concern is only language delay.

A child aged 1 to 2 years is expected to:
* Say first name
* Say many words (mostly naming words)
* Begin to use one- to two-word sentences such as ‘want milk’

This child is 14 months and have enough time to fulfill the above milestones. He has been developing normal so far and has not lagged behind. Language delay would be a concern if the child still babbles beyond 24 months of age.

How well did you know this?

1

Not at all

2

3

4

5

Perfectly

27

Q

You order an ultrasound scan for a 9-year-old girl, who has been brought for treatment of urinary tract infection for the second time this year. On the ultrasound, the right kidney is reported smaller than the left beyond the normal discrepancy. Which one of the following is the most appropriate investigation to consider for assessment of renal status?

A. DMSA.
B. DTPA scintigraphy.
C. Urea and electrolytes.
D. Abdominal CT scan.
E. Urine culture.

A

Correct Answer Is A, DMSA.

The dimercaptosuccinic acid scintigraphy (DMSA) scan is the gold standard for diagnosis of kidney scarring, which can have been resulted from recurrent urinary tract infections.

DMSA is indicated in the following conditions:
1. 1. Clinical suspicion of renal injury
1. Reduced renal function
1. Suspicion of VUR
1. Suspicion of obstructive uropathy on ultrasound in older toilet-trained children

For this child with recurrent episodes of UTI and a smaller-than-normal kidney, DMSA should be considered as the most diagnostic modality for assessment of possible renal scarring.

(Option B) DTPA scintigraphy is a type of radioisotope renography using DTPA and **Tc99 **as radio-labeled material.
This test also provides information regarding renal function and/or scarring. Compared with DMSA, this test is faster and associated with less radiation but DMSA remains the gold standard criterion for assessment of renal scarring.

(Option C) Urea and creatinine abnormalities are common among patients with urinary problems. Elevated levels of urea and creatinine indicated renal impairment but gives no clue regarding the kidney function per se. Urea and creatinine may raise in patients with pre-renal renal failure or post-renal renal failure in the initial course of which the
kidneys are normal or near normal. On the other hand, normal values do not exclude the presence of abnormalities in a single kidney. Many patients may have normal ranges in the presence of severe damage to one kidney while the other is functional and compensating.

(Option D) Abdominal CT scan will visualize the kidneys, adrenal glands and adjacent structure. It is not capable of assessment of renal function.

(Option E) Urine culture will show the presence or absence of infection in kidneys but does not provide any clue regarding current kidney function.

How well did you know this?

1

Not at all

2

3

4

5

Perfectly

28

Q

A five-year-old boy is brought for evaluation by his parents concerning unusual behavior. He loves Popeye cartoon and always remembers when it is shown on TV. At that time he turns on the TV. He becomes distraught and irritated if he misses the cartoon. He has a goldfish and spends several hours staring at it with curiosity and fascination. His
favorite toy is wooden building blocks that he uses to make towers, break them and build them again. He did not say his first word until the age of 2.5 years and could not use a communicative phrase at the age of 3 years. He does not play with his peers and is not interested in making friends. During the examination, he adamantly avoids eye contact.

Which one of the following is the most likely diagnosis?

A. Asperger syndrome.
B. Autism.
C. Mental retardation.
D. Delayed developmental milestones.
E. Obsessive-compulsive disorder.

A

Correct Answer Is B, Autism.

The persistent ritualism and preoccupations with activities (watching the Popeye cartoon) and objects (the building blocks and the goldfish), delayed speech language development, poor communication with friends and avoiding eye contact make Autistic disorder the most likely diagnosis in this child.
Autistic disorder is pervasive developmental disorder (PDD) affecting at least 4 children in 10000 with a male to female ration of 4:1.

Characteristic features of autistic disorder include the following:
1- Onset during infancy and early childhood
2- An impairment of social interactions shown by at least two of the following:
* lack of awareness of the feelings of others
* absent or abnormal comfort seeking in response to distress
* lack of imitation
* absent or abnormal social play
* impaired ability to socialize, including avoiding eye contact

3- Impairment in communication as shown by at least one of the following:
lack of babbling, gesture, mime or spoken language
absent or abnormal non-verbal communication
abnormalities in the form or content of speech
poor ability to initiate or sustain conversation
abnormal speech production

4- **Restricted or repetitive **activities, interests and imaginative development, shown in at least one of the following:
* stereotyped body movements
* persistent and unusual preoccupations and rituals with objects or activities
* severe distress over changes in routine or environment
* an absence of imaginative and symbolic play

5- Behavioral problems:
* tantrums
* hyperactivity
* destructiveness
* risk-taking activity

In a nutshell diagnosis of autistic disorder requires the presence of the 3 following core features by the age of 3 years:
1. Qualitative impairment of** social interaction**
2. Qualitative impairment of communication
3. Restricted, repetitive and stereotyped patterns of activities, behavior and interest

A majority of children with autistic have intellectual disability. Approximately 30% of autistic children have normal intellectual development.

(Option A) Asperger syndrome, also called high-functioning autism, shares many features with autistic disorder. However, children with Asperger syndrome have normal language development and intellectual ability. With language problems in this child, Asperger syndrome is an unlikely diagnosis.
(Option C) Although intellectual disability is common in children with Autism, this is a condition not a diagnosis.
(Option D) Children with Autism have language and cognitive problems but again these are specific conditions not a diagnosis. On the other hand, language development is impaired not delayed. ‘Delayed’ implies that such milestones may be reached at a later stage which will not happen in autistic children.
(Option E) Routines and ritualistic behavioral patterns are features seen in obsessive-compulsive disorder (OCD); however, delayed language, stereotypic behavior, and poor communications skills present in this child make OCD an unlikely diagnosis.

How well did you know this?

1

Not at all

2

3

4

5

Perfectly

29

Q

A 4-year-old girl is brought to you by her kindergarten teacher. She states that the child has not been well since this morning, vomited once, and complained of central abdominal pain. On examination, the child looks unwell and has a temperature of 39°C. Which one of the following would be most expected in this child?

A. Numerous pus cells on urine microscopy.
B. Inflamed ear drums.
C. Neck stiffness.
D. Inflamed tonsils.
E. Localized tenderness over the right iliac fossa.

A

Correct Answer Is A.

It is not uncommon for young children to have urinary tract infection (UTI) without classic urinary symptoms of urgency, frequency and/or dysuria. In fact, the younger the child, the more likely the symptoms are vague and nonspecific. In case the cause of such presentation is a UTI, abnormally high counts of white blood cells (puss cells) are the most common and expected finding on urinalysis.

(Option B) Inflamed ear drums are seen in otitis media. Otitis media is not associated with abdominal pain; therefore, not a likely diagnosis with this constellation of symptoms.

(Option C) Neck stiffness could be elicited in meningitis. Fever and unwellness are expected features, but abdominal pain is less likely.

(Option D) Tonsillitis can make a child ill and febrile, but does not cause abdominal pain.

(Option E) Central abdominal pain, vomiting and localized tenderness over the right iliac fossa can be caused by appendicitis, but it is uncommon for appendicitis to cause a fever as high as 39°C, unless it is complicated due to perforation. Since the symptoms has commenced this morning, perforation of an appendicitis would not be expected. It is unusual for an appendicitis to perforate this early.

How well did you know this?

1

Not at all

2

3

4

5

Perfectly

30

Q

A 7-year-old boy weighs 30 kg and has a body mass index of 19.5 kg/m2. Which one of the following is correct regarding his weight and BMI (use the following chart for calculation)?

A. He is underweight.
B. He is overweight.
C. He has normal weight.
D. He is obese.
E. BMI is not applicable in children.

Normal & Abnormal Growth and Development/Child Health/Paediatrics Flashcards by Lubi Mupwaya (2)

A

Correct Answer Is D.

Since the body composition changes with normal growth and stage of puberty, BMI in children is not a fixed measure like it is in adults. BMI interpretation in children 2-18 years of age must be plotted on the sex-specific BMI percentile charts. A 17-year-old boy with a BMI of 20 kg/m2 is within the normal range while a 6-year-old boy with the same BMI is obese.
To check the weight status of a child, his/her BMI should be calculated by dividing the weight by the square of the height in meter. The BMI then should be plotted on the sex-specific BMI percentile to see in which zone it is. This child’s BMI lies in the ‘obesity’ zone (see the chart below).

If used with the sex-specific percentile chart, BMI can be used for children with the same efficacy as for adults.

Normal & Abnormal Growth and Development/Child Health/Paediatrics Flashcards by Lubi Mupwaya (3)

How well did you know this?

1

Not at all

2

3

4

5

Perfectly

31

Q

Rachel, 5 years old, is brought to your GP clinic by her mother because of pain and redness in the back of her left ear.

According to the mother, Rachel developed fever and left ear ache one week ago, for which he was seen by another GP and started on paracetamol and amoxicillin. She was feeling better for the first 4 days but spiked a fever again and developed a painful red swelling behind her left ear. On examination, she has a warm, red, and tender swelling behind the left ear shown in the accompanying photograph. Which one of the following could be the most likely pathogenic factor for this presentation?

A. Staphylococcus aureus.
B. Streptococcus pneumoniae.
C. Hemophilus influenzae.
D. Moraxella catarrhalis.
E. Streptococcus viridans.

Normal & Abnormal Growth and Development/Child Health/Paediatrics Flashcards by Lubi Mupwaya (4)

A

Correct Answer Is B.

With the history of unilateral earache and fever one week ago, and development of a red and tender swelling behind the affected ear, acute mastoiditis complicating acute otitis media (AOM) is the most likely diagnosis.

Acute mastoiditis is rare; however, it is the most common suppurative complication of AOM. Acute mastoiditis may result in intracranial complications; therefore, it should be taken very seriously. Treatment of acute mastoiditis starts with intravenous antibiotics (Flucloxacillin plus a third-generation cephalosporin). Immediate involvement of an ENT consultant is of paramount importance as some cases may even require surgical intervention.

Acute mastoiditis is diagnoses based on postauricular (behind the ear) inflammatory signs such as erythema, edema, tenderness, and fluctuance. Other clinical findings include edema of auricle and/or external canal and associated signs of AOM. Although acute mastoiditis follows AOM, it can be the first presentation of AOM in some cases.

Common pathogens for acute mastoiditis are the same for AOM, with streptococcus pneumoniae being the most common one and the cause in over 50% of cases. Hemophilus influenza (option C) is the second most common causative organism responsible for such presentation.
Staphylococcus aureus (option A), Moraxella catarrhalis (option D), and streptococcus viridans are less common pathogenic organisms in AOM and acute mastoiditis.

How well did you know this?

1

Not at all

2

3

4

5

Perfectly

32

Q

A 5-year-old child is brought to your practice by his parents with complaints of fever and ear pain. On examination, he is in mild distress and has a fever of 38.3°C. Otoscopic examination reveals a red bulging tympanic membrane on the right side with decreased mobility. Which one of the following is** more likely to have caused** this presentation?

A. Hemophilus influenza.
B. Streptococcus pneumoniae.
C. Pseudomonas aeruginosa.
D. RSV virus.
E. Moraxella catarrhalis.

A

Correct Answer Is B.
A red bulging tympanic membrane with decreased mobility is the classic finding in acute otitis media (AOM).
In children older than 6 weeks, AOM is caused by bacteria in majority of cases. Streptococcus pneumoniae, Hemophilus influenzae, Moraxella catarrhalis, and Streptococcus pyogenes are responsible for the majority of episodes of AOM in persons older than 6 weeks. Other bacteria implicated in AOM include Staphylococcus aureus,
streptococcus viridans, and Pseudomonas aeruginosa.

Of these, Streptococcus pneumoniae is the most common etiologic agent responsible for AOM and for invasive bacterial infections in children of all age groups, followed by Hemophilus influenzae as the second most common etiologic factor.

RSV virus is the etiologic factor, mostly in neonates. Infection with this virus is associated with pneumonia,
bronchiolitis and otitis media in this age group; however, streptococcus pneumoniae remains the most common etiologic factor for AOM even in this age group.

How well did you know this?

1

Not at all

2

3

4

5

Perfectly

33

Q

Parents of an 11-year-old boy have brought him to your clinic for assessment because they believe he is obese. On examination, his weight and height are on 90 and 50 percentiles for sex- and age-matched growth charts respectively. Which one of the following is the most appropriate investigation to consider for him?

A. GH.
B. FSH and LH.
C. TSH.
D. Bone age.
E. Reassure he has normal growth and investigation is required.

A

Correct Answer Is C

For children and adolescents aged 2–18 years, growth is monitored based on age, height, and weight, using sexspecific Body Mass Index (BMI) percentile charts. BMI is not a fixed measure in this age group but varies with normal growth, stage of puberty, and sex. Either the United States Centers for Disease Prevention and Control (US-CDC) or WHO BMI percentile charts may be used, with the same chart used over time to allow for consistent monitoring of
growth. The US-CDC categorizes overweight as between the 85th and 95th percentile and obesity as above the 95th percentile. The WHO categorizes overweight as between the 85th and 97th BMI percentiles and obesity as above the 97th percentile. These categories are not diagnostic but contribute to the overall clinical impression of the child or adolescent being measured.

This boy, based on either chart, is overweight (>85th). The likelihood that childhood overweight and obesity will persist into adulthood increases with the age of the child and with the presence of parental obesity. One of the strongest predictors of a child’s weight is the weight status of his/her parents. For overweight or obese children, initial assessment by history and clinical assessment should determine current health problems and risks for future disease.

History taking includes developmental history, physical and mental health (including family history of obesity), and current health behaviors.

Clinical assessment includes pubertal stage, possible causes for overweightness or obesity (e.g., hypothyroidism), and indicators of comorbidities (e.g., raised blood pressure, joint pain, gastrointestinal symptoms, insulin resistance, intertrigo, dental health etc.)

It is also very important that secondary causes for obesity are considered and excluded. Of the options, TSH is an appropriate investigation for exclusion of hypothyroidism as a potential cause of overweightness.

(Options A and B) There is no clinical finding to suggest FSH/LH or GH as an initial assessment. Such clinical findings may include but not limited to short stature (necessitating GH evaluation) and delayed puberty (for TSH, FSH and LH).

(Option D) Bone age is often used as the initial assessment for short stature that is not the problem here.

(Option E) This boy is overweight and reassurance cannot be given because there might be an underlying cause present and also the child is at increased risk of obesity-relates health problems in the future.

How well did you know this?

1

Not at all

2

3

4

5

Perfectly

34

Q

Which of the following organism is the most common cause of urinary tract infections in children?

A. Proteus mirabilis.
B. E.coli.
C. Staphycoccis aureus.
D. Enterobacter.
E. Pseudomonas.

A

B. E.coli

In children, between 75 to 90% of urinary tract infections are caused by E.coli. Other organisms may include Klebsiella pneumonia, proteus mirabilis, staphylococci species, pseudomonas and enterobacters.

  • Medscape - Pediatric Urinary Tract Infection
  • RCH - Urinary tract infection

How well did you know this?

1

Not at all

2

3

4

5

Perfectly

35

Q

A 15-year-old girl presents with an upper respiratory tract infection (URTI). On examination, she has a temperature of 38.1°C and an erythematous pharynx with no exudate. She is otherwise healthy. A urine dipstick is positive for blood for which you order a urinalysis (UA). UA is reported back significant for red blood cells, 1+ proteinuria, no WBCs, and negative for nitrite. Which one of the following would be the next best step in management?

A. Repeat UA after the URTI resolves.
B. Urine culture.
C. 24-hour urine exam.
D. Ultrasonography.
E. Start her on corticosteroids.

A

A. Repeat UA after the URTI resolves

Asymptomatic hematuria in children, in the setting of a febrile illness other than urinary tract infection, should be assessed after the febrile illness subsides; provided that the illness is not due to urinary tract infection (UTI). With UTI, the next step would be a urine culture. This patient neither has urinary symptoms, nor laboratory findings consistent with UTI; therefore, no urine culture is indicated.

Asymptomatic microscopic hematuria should always be confirmed with 2 positive urine exams out of 3 in 2-3 weeks before any further assessment is considered. However, in the presence of symptoms such as hypertension, edema, or renal failure, prompt action is mandated.

How well did you know this?

1

Not at all

2

3

4

5

Perfectly

36

Q

A 5-year-old boy is brought to the Emergency Department by ambulance after he had a generalized tonic-clonic seizure at home 20 minutes ago. Upon arrival to the hospital, he was assessed immediately. He is not having a seizure now but is lethargic and confused. His blood pressure is 105/65 mmHg, pulse rate 76 bpm, respiratory rate 16 breaths/min, and temperature 37°C. His mucous membranes are not dry, skin turgor is normal, and capillary refill time is 2 seconds. The rest of the examination, including cardiovascular, respiratory and neurological exam is completely normal. Pathology results are as following:
* FBE: Normal
* Random blood sugar: 8.3 mmol/L (4 - 11.1 mmol/L)
* Sodium: 120 mmol/L (135-145 mmol/L)
* Potassium: 4.1 mmol/L (3.5 – 5.5 mmol/L)
* Bicarbonate: 24 mmol/L (22-32 mmol/L)
* Creatinine: 80 μmol/L (60-110 μmol/L)
* Urea: 4.5 mmol/L (2.5-7.1 mmol/L)
* Calcium: 3.1 mmol/L (2.2-2.7 mmol/L)

Which one of the following could be the most likely cause to this presentation?

A. Acute renal failure.
B. Addison disease.
C. Congestive heart failure.
D. Hyponatremia due to dehydration.
E. SIADH syndrome.

A

E. SIADH

Seizure can be caused by a variety of causes including:
* Metabolic derangements such as hyponatremia, hypernatremia, hypocalcemia, and hypoglycemia
* Structural abnormalities such as space occupying lesions in the brain
* CNS Infections such as meningitis
* Medication intoxication or withdrawal
* Epilepsy

There are a few clues in the history, physical examination, and laboratory findings that point towards hyponatremia as the most likely explanation for this presentation. Hyponatremia (and hypernatremia) present with CNS manifestations including lethargy, headache, alteration in consciousness, and in worse cases, seizure, coma, or even death. He also has hypercalcemia. But it is it is uncommon for hypercalcemia to cause seizures while hypocalcemia is notorious for that.

All the given options can potentiate and cause hyponatremia; hence, such presentation. However, this child has no signs of dehydration, and this excludes hyponatremia due to dehydration (option D) as a cause. Also, there is no history of congestive heart failure option C) nor any clinical findings such as abnormal cardiovascular exam findings, volume overload, etc. to support such diagnosis.

Acute renal failure (option A) can also cause hyponatremia but the normal urea and creatinine rules out such diagnosis.

Addison disease (option B) can be another explanation for hyponatremia in general but not as a likely diagnosis in this scenario. In Addison disease, adrenal insufficiency and lack of aldosterone result in hyponatremia and hyperkalemia at the same time. This child has a normal potassium. Furthermore, hypotension is one of the main clinical findings in patients with Addison disease. This child has normal blood pressure. Collectively, these make Addison disease a very remote possibility.

Of the options, SIADH is most likely to have resulted in such presentation. SIADH (syndrome of inappropriate anti- diuretic hormone) is caused by excess amount of anti-diuretic hormone (ADH) from the hypothalamus-pituitary unit. ADH affects kidneys to concentrates urine and retain water. In SIADH, excess ADH results in a very concentrated urine, increased body water, and hyponatremia through dilutional effect.

Depending on the rate at which hyponatremia occurs, patients with SIADH can be asymptomatic or have classic symptoms of hyponatremia including the following:
* Nausea and vomiting
* Headache
* Problems with balance that may result in falls
* Mental changes, such as confusion, memory problems, strange behavior Seizures or coma, in severe cases

Common causes of SIADH are:
* Medicines, such as certain type 2 diabetes medications, anti-epileptic medications, antidepressants, heart and blood pressure drugs, cancer drugs, drugs used for anesthesia
* Surgery under general anesthesia
* Disorders of the brain, such as injury, infections, stroke
* Brain surgery in the region of the hypothalamus
* Lung disease, such as pneumonia, tuberculosis, cancer, chronic infections
* Cancer of the lung, small intestine, pancreas, brain, leukemia

  • RACGP – AFP – The suspect: SIADH

How well did you know this?

1

Not at all

2

3

4

5

Perfectly

37

Q

Concerned parents of a 6-year-old girl has brought her to your GP clinic as they have noticed scattered growth of pubic hair in their daughter recently. On examination, she has sparse pubic hair growth and Tanner I breast. Her height and weight are on 75% and 50% percentiles respectively. Which one of the following is the most appropriate next step in management?

A. Ultrasounds scan of the pelvis.
B. Assessment of the bone age.
C. FSH and LH.
D. Review in 3 months.
E. 17-hydroxyprogesterone and estradiol.

A

D. Review in 3 months

The scenario represents development of pubic hair before the age of 8 years in girls (or 9 in boys) in the absence other sexual characteristics which are thelarche (breast development) and menarche (periods), which is referred to as premature isolated adrenarche. Adrenarche is the maturation of the adrenal zona reticularis in both boys and girls, resulting in the development of pubic hair, axillary hair, and adult apocrine body odor. Premature isolated adrenarche is considered a benign variant of normal development if the weight and height are normal for the age.

No investigations are required for premature isolated adrenarche as FSH and LH (option C), 17-hydroxyprogesterone (option E) and estradiol , and testosterone all normal. The only laboratory finding might be slight elevation of dehydroepiandrosterone sulfate (DHEAS). It is recommended that children with premature isolated adrenarche are followed every 3-6 months for surveillance of puberty development and linear progression and velocity of growth.

Bone age assessment (option B) is considered in constitutional growth delay (CGD) alongside exclusion of other causes of delayed puberty/growth to confirm the diagnosis or where the child is small for age. This child has normal weight and height for age.

Ultrasound of the pelvis (option A) is the study of choice for girls whose periods are overdue despite development of other secondary sexual characteristics, normal body measures, and absence of features suggesting chromosomal abnormalities such as Turner syndrome.

NOTE - puberty starts with thelarche. A Tanner II breast before the age of 13 years suggests estrogen exposure and commencement of puberty in girls. This is equivalent to testicular enlargement in boys. A Tanner II or less after 13 years is abnormal and indicated absence of breast development.

  • American Family Physicians: Disorders of Puberty: An Approach to Diagnosis and Management
  • RACGP – AFP – Growth disorders in children

How well did you know this?

1

Not at all

2

3

4

5

Perfectly

38

Q

Concerned parents of a 5-year-old boy have brough him to the Emergency department because he has been having palpitations and dizziness since this morning. He is otherwise healthy with no significant medical history, and this is the first time he is experiencing it. An ECG as well as blood tests are arranged. On examination, he has a weak thready pulse which is so rapid that cannot be counted correctly. His blood pressure is 98/60 mmHg. He seems restless but does not seem to be breathless. Extremities are of normal temperature and color. A 12-lead ECG is obtained and is shown in the following photograph. Which one of the following is the most appropriate next step in management of him?

A. Immersion of face in cold water.
B. Intravenous adenosine.
C. Intravenous amiodarone.
D. Synchronized cardioversion.
E. Intravenous esmolol.

Normal & Abnormal Growth and Development/Child Health/Paediatrics Flashcards by Lubi Mupwaya (5)

A

C. Intravenous amiodarone

The ECG shows wide-QRS complex tachycardia (WCT) at a rate of approximately 300 bmp. Most WCTs in children are in fact supraventricular tachycardias (SVT) with aberrancy. Compared to SVT with aberrancy, ventricular tachycardia (VT) is rare in children; however, due to seriousness of the condition, Australian Advanced Paediatric Life Support (APLS) and Royal Children Hospital (RCH) guidelines recommend that WCTs are considered and treated as VT until proven otherwise. This approach is different from recommendations by American Heart Association (AHA) that advise to treat WCTs in a hemodynamically stable children as SVT first by a trial of intravenous adenosine (option B) before making a diagnosis of VT.

APLS recommends intravenous amiodarone under close monitoring for treatment of symptomatic VT in children with stable hemodynamics, and synchronized DC cardioversion in those with hemodynamic instability/ shock if there is pulse and defibrillation in the absence of a pulse.

This child is hemodynamically stable; therefore, intravenous amiodarone will be the next best step in management.

OPTION A : Vagal maneuvers such as immersion of the face in cold water , application of ice pack to the face or Valsalva maneuvers are initial treatment for patients with narrow-QRS complex tachycardia i.e., SVT. Vagal maneuvers suppress the AV node and might worsen the condition in VT.

OPTION E : Intravenous beta blockers such as esmolol are not generally recommended for treatment of VT or SVT in children.

  • Advanced Paediatric Life support (APLS) – Algorithms – Ventricular Tachycardias
  • Medscape – Pediatric Ventricular Tachycardia Overview of Ventricular Arrhythmias

How well did you know this?

1

Not at all

2

3

4

5

Perfectly

39

Q

Emad is 6 years old and is being rushed to the Emergency Department by an ambulance after he experienced light- headedness, shortness of breath, chest tightness, and anxiousness at home 15 minutes ago. When you visit him, he is lying on a stretched and is receiving oxygen by non-re-breathable face mask. He is mumbling and does not seem to be oriented to time and place however he seems to recognize his parents. Examination reveals a very rapid barely perceptible pulse and blood pressure of 80/45 mmHg. Cardiac monitoring shows the following rhythm. Which one of the following is the most appropriate immediate management in this situation?

A. Defibrillation.
B. Synchronized DC cardioversion.
C. Intravenous amiodarone.
D. Intravenous lignocaine.
E. Intravenous adenosine.

Normal & Abnormal Growth and Development/Child Health/Paediatrics Flashcards by Lubi Mupwaya (6)

A

B. Synchronized DC cardioversion

Emad’s ECG shows tachycardia of approximately 300 bpm with regular wide QRS complexes (wide-QRS complex tachycardia [WCT]). In children, ventricular tachycardia (VT) is very rare, and most WTCs are in fact supraventricular tachycardia with conduction aberrancy (SVT with aberrancy). However, due to seriousness of VT compared to SVT, guidelines such Advanced Paediatric Life Support (APLS) as those by the Royal Children Hospital (RCH) recommend that WCTs should be considered and treated as VT until prove otherwise.

As such, Emad’s ECG should be considered and treated as VT. Treatment of VT with a pulse is determined by the presence or absence of shock/hemodynamic instability (hypotension, hypoperfusion, altered mental status)

For every patient presenting with hemodynamic instability caused by a cardiac tachyarrhythmia, DC cardioversion should always be considered as the most appropriate treatment option (defibrillation for ventricular fibrillation and pulseless VT; synchronized for the rest.)

Emad has an ECG with WCT in addition to hypotension and hypoperfusion evident by altered mental status (disorientation to time and place) and cold pale sweaty extremities. Of the options, synchronized DC cardioversion is the next step in treatment to consider for him.

Defibrillation (unsynchronized) (option A) is used as first line for treatment of ventricular fibrillation (VF) and pulseless VT.

If Emad was not hemodynamically unstable, intravenous amiodarone (option C) would be considered first. Amiodarone is the medication of choice for children presenting with symptomatic wide QRS tachycardia who are not in shock.

Intravenous amiodarone (class III antiarrhythmic) is almost always the medication of choice for treatment of symptomatic VT in stable infants and young children. The medication has a broad efficacy and is readily available. However, major adverse reactions have always been concerning.

Lignocaine (lidocaine) (option D) is a class I (sodium channel blocker) antiarrhythmic medication. It is no longer recommended for treatment of VT in children.

NOTE - American Heart Association recommends a trial of intravenous adenosine in symptomatic yet stable children with WCT because most of such WTCs are in fact SVT with aberrancy. This way, the child is not unnecessarily exposed to adverse effects of amiodarone or other antiarrhythmics. The Australian Advanced Paediatric Life Support (APLS) and Royal Children Hospital (RCH) guidelines on the other hand endorse considering and treating WCTs as VT rather than SVT until proven otherwise. In any case, **Emad is unstable, and adenosine has no role in his management **based on either of those sets of guidelines.

  • APSL – Ventricular tachycardia algorithm

How well did you know this?

1

Not at all

2

3

4

5

Perfectly

40

Q

Parents of a 4-year-old girl has brough her to the Emergency Department after she started feeling short of breath and dizzy half an hour ago at home. Parents deny any other previous similar episodes or any significant recent or past medical conditions. On examination, a very rapid thready and hardly perceptible pulse is felt. She has a blood pressure of 98/65 mmHg and respiratory rate of 35 breaths per minute. He is afebrile and has normal peripheral perfusion. You start oxygen via non-rebreathable face mask, put him on cardiac monitoring, and stablish an intravenous line. Cardiac monitoring shows the following rhythm. Which one of the following is the most appropriate next step in management?

A. Put an icepack on his face.
B. Immerse his head in cold water.
C. Intravenous adenosine.
D. Intravenous verapamil.
E. Synchronized DC cardioversion.

Normal & Abnormal Growth and Development/Child Health/Paediatrics Flashcards by Lubi Mupwaya (7)

A

A. Put an icepack on his face

The photograph is typical for supraventricular tachycardia (SVT) at a rate of approximately 170 bpm. SVT is an umbrella term used to describe tachycardias (atrial and/or ventricular rates above 100 bpm at rest) that originates from the His bundle or above. These SVTs include inappropriate sinus tachycardia, atrial tachycardia (AT) including focal and multifocal AT, macro-reentrant AT (including typical atrial flutter), junctional tachycardia, atrioventricular reentrant tachycardia (AVRT), atrioventricular nodal reentrant tachycardia (AVNRT), and various forms of accessory pathway-mediated reentrant tachycardias e.g., Wolff-Parkinson, White syndrome (WPW).

In younger children, SVT is often caused by AVRT, including WPW while in adolescents it is caused by AVNRT. Other causes are very rapid sinus tachycardia, atrial flutter, ectopic atrial tachycardia, and junctional ectopic tachycardia.

SVT typically has a fixed rate, usually >220 bpm. Onset and offset are abrupt, and p-waves are either not visible or seen after the QRS complexes. Conditions such as sepsis, pain, dehydration, anxiety, and fever can contribute to development of SVT and should always be considered and addressed in management of children with a tachyarrhythmia

NOTE - SVT typically causes narrow complex tachycardia; however, in SVT with aberrancy, QRS complexes could be wide, resembling ventricular tachycardia (VT). Although the majority (95%) of wide-QRS complex tachycardias in children are SVT with aberrancy, sustained wide-QRS complex tachycardia should always be considered and treated as VT until proven otherwise (RCH recommendation).

Symptomatic SVT often presents with palpitations, shortness of breath, dizziness, and decreased exercise tolerance. If protracted, signs and symptoms of congestive heart failure can develop.

Management of children (and adults) with SVT depends on the presentation. For all patients, oxygen, securing intravenous access, cardiac monitoring, and an a 12-lead ECG should are the initial steps to take. The rest of the management is determined by the presence or absence of indicators of hemodynamic instability which are hypotension, hypoperfusion, or hypoperfusion-related altered mental status.

In the absence of hemodynamic instability, vagal stimulation maneuvers are tried first:

  1. Neonates and infants (<6 months):
    Immersion of the face in ice water for 5 seconds to elicit diving reflex (not to be use for those with hemodynamic instability/shock). While attached to cardiac monitor and arms are wrapped in a towel, immerse the whole face in ice water for five seconds. It is unnecessary to occlude the nostrils. This technique is safe and 90% effective in terminating SVT.
  2. Older infants/ toddlers:
    Apply an ice-cold facecloth or bag filled with ice to the face for 15-30 seconds (elicits the ‘dive reflex’). It is less effective than facial immersion but better tolerated in older infants.
  3. School-aged children:

Valsalva technique : with the child in the supine position, ask them to blow on their thumb or a 10 mL syringe, after full inspiration, for 10-15 seconds. There should be no air escape and the child should be seen to strain. In modified Valsalva, the same manoeuvre is performed in a semi-recumbent position. Then at the end of the strain, the child is immediately repositioned into a supine position with passive leg raise to 45 degrees.
This child has symptomatic SVT but is hemodynamically stable. For her, vagal maneuvers are likely to end the SVT and should be tried first. At this age group, putting an ice pack or ice-cold towel/cloth on the face for 15-30 seconds is the preferred method.

Immersion of face (option B) in ice water for five seconds is more effective than placing ice pack on the face. However, it is less likely to be tolerated by children older than 6 months and is not recommended for this age group as the preferred method despite higher efficacy.

Intravenous adenosine (option C) is the preferred medication for terminating SVT in children and adults, but it should be considered if vagal maneuvers fail to work.

Despite the increasing use of the calcium-channel blocker verapamil (option D) for treatment of SVT in adults, it is not routinely used for such purpose in children. It is also contraindicated in children less than 12 months because it can cause irreversible hypotension and fatal ventricular dysfunction in this age group.

Synchronized DC cardioversion (option E) would be the correct answer if the child was in hemodynamic instability/shock. However, some guidelines including Advances Paediatric Life Support (APLS) recommend vagal maneuvers are tried while preparation for DC cardioversion is made, provided it does not cause any delay in delivering the latter.

NOTE - American Heart Association (AHA) recommends that based on rarity of ventricular tachycardias (VT) in children, wide-complex QRS tachycardias with a pulse be considered and treated as supraventricular tachycardia (SVT) with aberrancy if the QRS complex are regular and a pulse is present. Based on recommendations by the AHA (and also UpToDate) a trial of adenosine in such cases should be considered first for children who are hemodynamically stable, both as therapeutic and diagnostic measures (cessation of tachycardia in response to adenosine favors SVT with aberrancy rather than VT). With no response to adenosine, the tachycardia should be treated as VT (e.g., with IV amiodarone).

  • RCH – Supraventricular tachycardia (SVT)
  • APLS – Supraventricular tachycardia algorithm

How well did you know this?

1

Not at all

2

3

4

5

Perfectly

41

Q

Sarah has brought Jesse, her 4-year-old son, Jesse, because of bedwetting. Jesse is her first child and is the outcome of an uncomplicated pregnancy with good health so far. He mingles with his peers and has met all milestones otherwise. He is very attached to his Teddy bear and will not sleep without it. He also has some imaginary friends to whom he talks sometimes. He is dry during the daytime but wets the bed a few times per month. Physical examination is normal. Which one of the following is the most likely diagnosis?

A. Autism.
B. ADHD.
C. Psychosis.
D. Normal development.
E. Anxiety.

A

D. Normal development

Bedwetting happens when the bladder empties involuntarily during sleep, most children will gain daytime urinary continence by the age of 3 years, and night continence by the age of 6 years. Usually, no treatment is required for night-time bed wetting by the age of 6 years as there is a high rate of spontaneous resolution. Also, attachment to toys or objects such as the Teddy bear in this scenario is completely normal up to the age of 7 years and is a healthy form of play for children.

Jesse is 4 years old and otherwise normal. Sarah should be advised that for now, it can be normal to wet the bed a few nights in a month and that his talking to imaginary friends and attachment to his toys are not concerning at this age and there is no need to be alarmed.

OPTION A : Autism presents with communication problems such as finding friends and playing with peers and repetitive ritualistic behaviors, none of which are present in Jesse.

OPTION B : ADHD presents with poor concentration and impulse control. The main clinical presentation of ADHD includes:

  • Inattention: difficulty concentrating, forgetting instructions, moving from one task to the other without completing the previous one
  • Impulsivity: acting without thinking, talking over the top of others, losing control of emotions easily, and being accident prone
  • Overactivity: constant fidgeting and restlessness

None of the above is present in Jesse.

OPTION C : Having imaginary friends and talking to or playing with them is quite normal in children and part of their social development; therefore, a diagnosis of psychosis cannot be based on that unless other features are psychosis are present e.g., hallucinations, delusions, thought process problems, apathy, etc.

OPTION E : Anxiety presents with affective symptoms such as subjective discomfort, erratic concentration, or hypervigilance or somatic manifestations such as tachycardia, hyperventilation, nausea, vomiting, urinary frequency, or so on. Jesse doe does not seem to have any signs or symptoms suggestive of anxiety.

  • RCH – Enuresis: Bed wetting and Monosymptomatic Enuresis

How well did you know this?

1

Not at all

2

3

4

5

Perfectly

42

Q

A 35-year-old woman gives birth to her second male baby through an uneventful vagin*l delivery. The baby is preterm
at 34 weeks and weighs 3100 gr. Apgar’s scores at the first and 5th minutes are 6 and 9, respectively. Almost one hour after birth, he develops tachypnea and grunting as well as intercostal recession. He is placed under the oxygen hood. In the next 4 hours, his condition remarkably improves. Which one of the following could be the most likely diagnosis?

A. Meconium aspiration.
B. Birth asphyxia.
C. Transient tachypnea of the newborn.
D. Respiratory distress syndrome.
E. Tension pneumothorax.

A

C. Transient tachypnea of the newborn

The scenario represents early-onset respiratory distress in a newborn with a near normal and normal APGAR score of 6 and 9 at 1 and 5 minutes. These scores exclude birth asphyxia , which is defined as failure to establish breathing at birth. This baby started breathing right after birth but developed respiratory problems shortly after.

Of the other options, transient tachypnea of the newborn (TTN) and respiratory distress syndrome (RDS) seem more likely compared to others.

Transient tachypnea of the newborn (TTN) is a benign, self-limiting condition that can present in infants of any gestational age shortly after birth. The pathophysiology is a delay in the clearance of fetal lung fluid after birth, which leads to ineffective gas exchange, respiratory distress, and tachypnea. RDS on the other hand is caused by surfactant deficiency in preterm newborns.

Both TTN and RDS present with the following:
1. Onset within the first few minutes to hours after birth.
2. Physical exam findings usually include signs of respiratory distress:
-Tachypnea (respiratory rate greater than 60 per minute) Nasal flaring
-Grunting
-Intercostal/subcostal/suprasternal retractions

The distinctive feature between these two is the clinical course. While TTN improves with supportive treatment usually within 6 hours of onset, RDS worsens over 48-72 hours before it improves. The fact that this newborn has made a remarkable recovery in 4 hours makes TTN more likely of a diagnosis compared to RDS.

Meconium aspiration syndrome is a common cause of early-onset neonatal respiratory distress in near- term, term or post-term neonates. One important clue to such diagnosis is the presence of meconium on the neonate’s body (meconium-stained baby) or in the amniotic fluid. Prematurity and absence of any clues on the presence of meconium in the scenario are less likely.

OPTION D : Tension pneumothorax presents with acute onset respiratory distress. Diagnostic clues include tachycardia, tachypnea, hypotension, asymmetrical respiratory movements, absence of breath sounds, and hyper- resonance of the affected side and engorged neck and forehead veins. As none is mentioned in the scenario, it is less likely of a diagnosis.

  • Star Pearls – Transient Tachypnea of the Newborn
  • Safe Care Victoria – Respiratory Distress Syndrome (RDS) in Neonates
  • Safe Care Victoria – Meconium Aspiration Syndrome
  • RCH – Recognition of the seriously unwell neonate and young infant
  • AAFP - Newborn Respiratory Distress

How well did you know this?

1

Not at all

2

3

4

5

Perfectly

43

Q

A 9-year-old girl was brought to your practice two weeks ago with a fever. Your assessment established an upper respiratory tract infection (URTI) as the diagnosis; however, you were also concerned about 3+ blood and 1+ proteinuria on a dipstick urine exam; therefore, you advised a follow-up in two weeks. Today, she is back for it and dipstick urine shows 2+ blood and no proteinuria. You arrange for a formal urinalysis which comes back negative for glomerular red blood cells and casts. Which one of the following is the most important next step in management?

A. Urine culture.
B. DMSA.
C. Ultrasound.
D. IVP.
E. ASOT.

A

A. Urine culture

Based on current guidelines by the Royal Children’s Hospital (RCH), asymptomatic isolated hematuria in a child with a febrile disease other than urinary tract infection should be followed up once the febrile disease settles. Based on this guideline, this child has undergone another urine exam in 2 weeks which still shows isolated hematuria of non- glomerular origin.

For this child, a urine culture/sensitivity to exclude urinary tract infection is the most appropriate next step in management.

OPTION B : DMSA scan (DMSA, or dimercaptosuccinic acid, bound to technetium 99m) isotope is circulated through the body. 95% of the radiopharmaceutical is bound to the renal cortex and the remaining 5% is usually excreted into the urine. Modern gamma cameras capable of performing single-photon emission tomography (SPECT), are able to reconstruct 3-dimensional images of the kidneys. In children, the accepted indications for a DMSA scan include renal scars as a result of an insult (usually recurrent urinary tract infections) or reflux-associated nephropathy.

OPTION C : Ultrasound has a place in the diagnosis of renal morphology and assessing renal damage, especially in the setting of hematuria, edema, and hypertension where underlying nephritis is suspected.

OPTION D : IVP (Intravenous Pyelography) is used to check urine flow if there is concern regarding kidney, ureter, or bladder blockage, or to check the function and appearance of the urinary tract after surgery to prevent blockages from causing permanent damage to the kidneys.

OPTION E : ASOT (Anti-streptolysin O Titer) is used in the case of post-streptococcal GN in a school-aged child with macroscopic hematuria, edema due to fluid retention, and possibly a headache due to hypertension. Investigation shows elevation of urea and creatinine, normocytic anemia, elevated streptococcal markers (ASOT/anti-DNase B), and a depressed complement (C3) level.

TOPIC REVIEW

Pediatric hematuria can be divided into the following categories :

  • Microscopic hematuria - (>10 RBC/microlitre) can only be detected by urinalysis as urine color remains normal
  • Macroscopic hematuria - visible blood in urine without microscopy, and is more likely to come from the bladder or urethra rather than the kidney
  • Persistent microscopic hematuria – three positive separate samples for blood, each taken at least one week apart, without prior exercise nor during menstruation
  • Isolated asymptomatic microscopic hematuria - the presence of microscopic hematuria without clinical symptoms or any other abnormalities in the urine such as proteinuria

Assessment of pediatric hematuria should include :
* Previous history of hematuria
* Symptoms of urinary infection such as dysuria, frequency, pain, fever
* Systemic symptoms such as fatigue, edema, rash, arthralgia, or coryza
* Recent surgery or trauma including non-accidental injury
* Family history of hematuria
* History of underlying bleeding disorder or immunodeficiency
* Medication history
* Food intake (beetroot and berries can color urine to pink or red)
* Recent exercise, especially if strenuous

Causes of hematurai in the pediatric pupulation :
A . Common
-Urinary tract infection
-Perineal irritation
-Trauma
-Meatal stenosis with ulceration

B. Uncommon
-Glomerulonephritis
-Renal calculi
-Coagulation abnormalities
-Tumors

Investigation of macroscopic hematuria
* Urine Dipstick urinalysis
* Mid-stream urine Microscopy, culture, and sensitivity (MSU MCS)
* Calcium/creatinine ratio
* Blood Electrolytes including creatinine, full blood count, Antistreptiterin titer (ASOT)/anti-DNase B Complement (C3)
* Renal ultrasound

  • RCH - Haematuria

How well did you know this?

1

Not at all

2

3

4

5

Perfectly

44

Q

Tom, 5 years old, is in the emergency department with a swollen face, ankles, and a distended abdomen. Dipstick urine is positive for blood (trace) and protein (+++). He is hemodynamically stable. Which one of the following is the most appropriate next investigation to consider for him?

A. Urine culture.
B. Urinary Protein to Creatinine Ratio.
C. Urine cytology.
D. Blood chemistry.
E. Renal ultrasound.

A

B. Urinary Protein to Creatinine Ratio

With proteinuria and edema, Tom must be fully assessed for renal disease and probably nephrotic syndrome.

Proteinuria is likely to present either as a symptomatic disease (i.e., nephrotic syndrome), an incidental finding during an assessment of either renal or nonrenal symptoms or detected on urinary screening in asymptomatic patients.

Nephrotic syndrome is defined by the occurrence of generalized edema, heavy proteinuria (+++ or ++++ on dipstick), hypoalbuminemia, and hypercholesterolemia. Nephrotic syndrome occurs most commonly in young children, with peak incidence for the initial episode at 2 years of age.

Children with steroid-responsive nephrotic syndrome usually have no nephritic features such as hematuria, hypertension, or raised serum creatinine. However, microscopic hematuria is occasionally seen (15–30%).

Most children will turn out to have minimal change disease (80%) or focal segmental GN (5–10%), while mesangial proliferative glomerulonephritis (GN), membranoproliferative GN, and membranous GN are all rare.

In as many as 30–50% of children, proteinuria is transient and resolves over 1–2 weeks. Transient proteinuria is common during febrile illnesses and can occur with strenuous exercise, emotional stress, and following seizures or abdominal surgery.

Isolated proteinuria resolves spontaneously after the cessation of the causal factor and an extensive workup is not necessary. Isolated proteinuria is common in children while persistent proteinuria is much less common.

Evaluation of proteinuria should begin with a careful history and thorough physical examination, urine microscopic examination, and determination of the amount of protein excretion rate (PER). The PER has been traditionally measured using 24-hour urine collections. However, the collection of 24-h urine is often cumbersome, and spot urinary protein-to-creatinine ratio (PCR), expressed in g/g or mg/mg, has become a simple and attractive yet reliable alternative. A spot urine PCR has been found to have a significant linear correlation with a 24-h urine PCR. because the PCR compares urinary protein concentration with urinary creatinine concentration, urinary dilution or concentration does not influence this value.

In young children, accurately timed collections are difficult to obtain and the protein/creatinine ratio (PCR) on an untimed urine specimen has been the accepted standard for many years. A random urine specimen is acceptable.

OPTION A : Urine microscopy and culture (Urine MCS) is considered if the patient is febrile

OPTION C : Urine cytology is considered when the glomerular or non-glomerular origin or under debate and the presence of urinary casts provides a clue

OPTION D and E : Blood chemistry and renal ultrasound are considered the next steps in order to look for an underlying disease but not the next step.
* RCH -Proteinuria
* RCH - Nephrotic syndrome

45

Q

The parents of John, 6 years of age, have brought him to your GP practice due to being bullied in school. When you start talking to him by asking how he is feeling today, he says anxiously “I I I…wan ‘n ‘n ttt to ttoo go o home”. He is otherwise normal with no remarkable physical findings. According to the parents, he has met developmental milestones otherwise. Which one of the following is the most important initial step in management?

A. Referring the child to a speech therapist.
B. Referring the patient to a pediatric psychiatrist.
C. Telling the child to speak slowly.
D. Enforcing reward and punishment strategy.
E. Arranging for school observation.

A

A. Referring the child to a speech therapist

Stuttering or stammering is a speech disorder characterized by interruptions to speech such as hesitating, repeating sounds and words, or prolonging sounds. The cause is unknown, but genetics may be a factor. Anxiety and stress do not cause stuttering but can make it worse.

About 1 in 100 Australians stutter. The condition can affect children, adolescents, and adults. It usually starts in childhood, between the ages of around 2 and 4 years, although it can also start later. Stuttering can start overnight, or it can build up over time.

Up to 1 in 12 three-year-old children stutter, but approximately three-quarters of them will recover without any treatment – although it might take a few years. Persisting stutter beyond adolescence is unlikely to spontaneously recover.

It is important that any child who stutters is promptly referred to a speech therapist/pathologist for appropriate assessment. The earlier a child starts treatment, the better the prognosis. The best evidence from clinical trials for treating children who stutter is the Lidcombe Program of Early Stuttering Intervention. This is a behavior modification treatment. The main principles involve praising a child when words are spoken clearly, and occasionally noting when stuttering has occurred.

OPTION B : A psychiatrist referral is not the appropriate initial step if the child appears to have no developmental delays otherwise.

OPTION C : Asking the child to speak slowly is not the appropriate response or treatment.

OPTION D : A reward and punishment strategy is also not appropriate, because punishing the child may induce further stress and cause even more stuttering (D is incorrect).

OPTION E : School observation is not indicated at this stage since the child has already presented to your clinic with the condition.

  • The Sydney Children’s Hospital Network
  • Better Health Victoria

46

Q

A mother brings her 10-month old boy to the Emergency Department with fever, nausea and vomiting. She mentions that she has used fewer nappies in the past 48 hours because he has been less wet. She also mentions that his urine has an offensive odor. A urine analysis is perfomed that is positive for nitrite and leukocytes consistent with urinary tract infection (UTI). This child has never had an UTI before. Which one of the following is the next best step in management?

A. Antibiotics and ultrasound of kidneys, ureter and bladder.
B. Antibiotics, intravenous fluids and ultrasound scan of the kidneys, ureter and bladder.
C. Antibiotics, oral fluids and discharge home once the child improves.
D. Refer to a pediatrician.
E. Antibiotics.

A

B. Antibiotics, intravenous fluids and ultrasound scan of the kidneys, ureter and bladder

The scenario describes a baby who has developed dehydration (decreased urine output) as a result of vomiting in the background of UTI. This child needs rehydration as the most appropriate initial management.

Dehydration is an indication for hospital admission and fluid resuscitation. Antibiotics should also be started for treatment of UTI.

In the following conditions, ultrasound scan of the kidney, ureter and bladder is always indicated:
* Concurrent bacteremia
* Atypical UTI organisms: i.e., Staphylococcus aureus, Pseudomonas
* UTI <3 years old
* No/inadequate response to 48 hrs of IV antibiotics
* Abdominal mass
* Abnormal voiding (e.g., dribbling)
* Recurrent UTI
* First febrile UTI and no prompt follow up assured
* Renal impairment
* Significant electrolyte derangement
* No antenatal renal tract imaging in second to third trimester

Since this child is younger than 12 months age, he needs an ulrasound as well. It is best to perform the ultrasound within the first 3 days of presentation. If the child is younger than 1 year, either ultrasound or micturating cystoureterogram or both can be used to exclude vesicoureteric reflux (VUR). If the child is older than 1 year, ultrasound is adequate. Early detection of VUR and control of recurrent urinary tract infections could prevent the development of reflux nephropathy and its complications including hypertension and chronic renal failure.

NOTE - According to RCH guidelines, ultrasonography is indicated in the following situation (different from international guidelines and even National Guidelines by the Therapeutic Guidelines or RACGP):

  • Children with atypical UTI, those not responding to treatment within 48 hours, and boys <3 months of age should have a renal ultrasound to exclude renal obstruction.
  • Children <6 months should have a renal ultrasound within 6 weeks of diagnosis. It should be performed during the illness if the UTI is atypical or not responding to antibiotics within 48 hours.
  • Older children do not require an ultrasound after the first UTI, but should have a renal ultrasound for recurrent UTI.
  • RCH - Urinary tract infection
  • Medscape - Pediatric Urinary Tract Infection
  • Therapeutic Guidelines – Antibiotics; available on http://tg.org.au

47

Q

Lucas, 16 years old, is brought for medical attention after he developed a widespread rash yesterday. He has asthma and a history of eczema but is otherwise fit and healthy. The rash started on his face and spread downwards to involve his torso. He feels unwell and has flu-like symptoms and a fever of 38.2°C. Which of the following is the most likely diagnosis?

A. Impetigo.
B. Stevens-Johnson syndrome.
C. Allergic contact dermatitis.
D. Erysipelas.
E. Eczema herpeticum.

Normal & Abnormal Growth and Development/Child Health/Paediatrics Flashcards by Lubi Mupwaya (8)

A

E. Eczema herpeticum

The photograph shows multiple facial lesions on an erythematous base, which are fluid-filled and some with central dimpling. The characteristics of the rash and importantly the presence of eczema in history make eczema herpeticum the most likely diagnosis in this scenario.

Eczema herpeticum is a disseminated viral infection characterized by fever and clusters of itchy blisters or punched- out erosions. It is most often seen as a complication of eczema/atopic dermatitis. The lesions resemble the chickenpox/varicella rash.

Most cases of eczema herpeticum are due to type 1 or 2. Eczema herpeticum usually arises during the first episode of infection with herpes simplex (primary herpes). Signs appear 5–12 days after contact with an infected individual, who may or may not have visible cold sores. Eczema herpeticum may also complicate recurrent herpes. However, repeated episodes of eczema herpeticum are unusual.

Eczema herpeticum can affect males and females of all ages but is more commonly seen in infants and children with atopic dermatitis. Patients with atopic dermatitis appear to have reduced immunity to herpes infection. Their underlying dermatitis can be mild to severe, active, or inactive.

Eczema herpeticum starts with clusters of itchy and painful blisters. It may affect any site but is most often seen on the face and neck. Blisters can occur in normal skin or sites actively or previously affected by atopic dermatitis or another skin disease. New patches form and spread over 7–10 days and may rarely be widely disseminated throughout the body. The patient is unwell, with a fever and swollen local lymph nodes.

The following are the characteristics of the rash in eczema herpeticum:
* The blisters are monomorphic, that is, they all appear similar to each other.
* They may be filled with clear yellow fluid or thick purulent material.
* They are often blood-stained i.e., red, purple, or black.
* New blisters have central dimples (umbilication).
* They may weep or bleed.
* Older blisters crust over and form sores (erosions)
* Lesions heal over 2–6 weeks.
* In severe cases where the skin has been destroyed by infection, small white scars may persist long-term.

Severe eczema herpeticum may affect multiple organs, including the eyes, brain, lungs, and liver. It can rarely be fatal.

Secondary bacterial infection with staphylococci or streptococci may lead to impetigo (option A) , erysipelas (option D) , or cellulitis; however, this patient does not have any clinical features suggestive of any of the mentioned
conditions.

Contact dermatitis (option C) is localized at and limited to the site of the contact with the culprit allergen. Moreover, systemic symptoms are not a feature. Stevens-Johnson syndrome (option B) is a rare serious disease of the skin and mucous membranes usually as a reaction to a medication. It starts with flu-like symptoms and is followed by a painful rash that spreads and blisters and results in sloughing off of the skin and mucous membranes. The absence of a triggering medication and different rash characteristics make such a diagnosis unlikely.

  • DermNet - Eczema herpeticum

48

Q

A 12-year-old child has history of recurrent urinary tract infections since the age of 3 years. Which one of the following is the investigation of choice to exclude renal scarring?

A. Renal biopsy.
B. Renal ultrasound.
C. Serum urea and creatinine.
D. DMSA scan.
E. CT scan of the abdomen.

A

D. DMSA scan

Recurrent urinary tract infections (more than 2 times during childhood) can result in renal scarring, hypertension, and end-stage renal disease. The underlying etiology is vesicoureteral reflux (VUR) -retrograde passage of urine from the bladder into the upper urinary tract.

Dimercaptosuccinic acid scintigraphy (DMSA) scan is the gold standard for diagnosis of kidney scarring and assessment of renal functions.

DMSA is indicated in the following conditions:
* Clinical suspicion of renal injury
* Reduced renal function
* Suspicion of VUR
* Suspicion of obstructive uropathy on ultrasound in older toilet-trained children

For this child with recurrent episodes of UTI, a DMSA should be considered as the most diagnostic modality for assessment of renal function and possible renal scaring.

OPTION A : Biopsy provides clues to histopathological abnormalities in kidney associated with certain type of renal diseases such as glomerulonephritis but not the overall kidney function. Moreover, it is unnecessarily invasive.

OPTION B : Ultrasound is capable of assessing the presence of fluid collections, bladder volume and the size, shape and location of kidneys. However, it gives no clue regarding kidney function or scarring. Ultrasound is indicated for assessment of children with UTI in the following situations:

  • Concurrent bacteremia
  • Atypical UTI organisms: i.e., Staphylococcus aureus, Pseudomonas UTI <3 years old
  • Non/inadequate response to 48hrs of IV antibiotics
  • Abdominal mass
  • Abnormal voiding
  • Recurrent UTI
  • First febrile UTI and no prompt follow up assured
  • Renal impairment
  • Significant electrolyte derangement
  • No antenatal renal tract imaging in second to third trimester

OPTION C : Urea and creatinine abnormalities are common among patients with urinary system problems. Elevated levels of urea and creatinine indicated renal impairment but gives no clue for each indivudual kideny or at times kidneys in general because urea and creatinine may be normal in early stages of pre-renal or post-renal renal failure.

On the other hand, normal values do not exclude the presence of abnormalities in only one kidney as the other kidney can compensate. Many patients may have normal ranges in the presence of severe damage to one kidney while the other is functional and compensating.

OPTION E : Abdominal CT scan will visualize the kidneys, adrenal glands and adjacent structure but is not capable of assessment of renal function.

  • RCH - Urinary tract infection
  • Medscape - Pediatric Urinary Tract Infection

49

Q

An eight-month-old male infant is brought to your office with history of vomiting and diarrhea. Urine dipstick examination suggests urinary tract infection. The child is not dehydrated but seems slightly ill. Which one of the following is the next best step in management?

A. Give antibiotics.
B. Give antibiotics and refer for ultrasound.
C. Investigate for a sexually transmitted infection.
D. Notify child protection services.
E. Oral rehydration and discharging home.

A

B. Give antibiotics and refer for ultrasound

Urinary tract infection (UTI) in children may arise from urogenital abnormalities such as vesicoureteric reflux. For this reason, in addition to treatment with antibiotics, an ultrasound scan of the kidney, ureter and bladder should be considered in all children younger than 3 years of age (2-36 months in most guidelines) with the first episode of UTI.

Also, ultrasound is also indicated for children with UTI if:
* Concurrent bacteremia
* Atypical UTI organisms: i.e., Staphylococcus aureus, Pseudomonas
* Non/inadequate response to 48hrs of IV antibiotics
* Abdominal mass
* Abnormal voiding
* Recurrent UTI
* First febrile UTI and no prompt follow-up assured
* Renal impairment
* Significant electrolyte abnormalities
* No antenatal renal tract imaging in second to third trimester

In children less than 1 year, an ultrasound should be performed first. If the ultrasound is normal, voiding cystoureterogram (VCU) would be the next step. This approach is adopted for early detection of urologic structural abnormalities as well as possible vesicoureteric reflux.

In children older than 1 year of age, ultrasound alone is the investigation of choice for excluding vesicoureteric reflux. Approximately 40% of the children with UTI have vesicoureteric reflux.
Vesicoureteric reflux can result in scarring of the kidneys, hypertension and chronic renal failure. Early detection of vesicoureteric reflux helps prevent such complications.

  • RACGP - Paediatric urinary tract infections: Diagnosis and treatment

50

Q

A mother has brought her 2-year-old boy to your GP clinic with complaint of chronic constipation. She mentions that her boy opens bowels every 5 days. She is also concerned about her child’s recent weight loss. Which one of the following piece of information, if in history, will help to make a diagnosis?

A. Delayed passage of meconium.
B. Cystic fibrosis of the mother’s brother in family history.
C. The child is premature at 35 weeks.
D. The child’s weight is on 25th percentile.
E. Diarrhea after introducing cow’s milk.

A

A. Delayed passage of meconium

Of the options, delayed meconium passage if in the history suggests Hirschsprung disease as the most likely cause of this infant’s constipation.

Hirschsprung disease is a congenital anomalyy characterized by absence of ganglia in a segment of colon and paralysis of this aganglionic segment. The condition starts at the anus and progresses up the rectum towards the colon. It rarely affects more than the first 30 cm of the rectum and colon. The disease in more common in males. Classic presentation is with the history of delayed meconium passage after birth and constipation in early infancy.

OPTION B : With an affected brother with CF, the mother may be a carrier. If so, the baby had a 25% chance of inheriting the disease if the father is a carrier, or 50% if affected. The history and clinical presentation is completely inconsistent with CF as the diagnosis. The diagnostic triad of CF includes (1) chronic cough and recurrent chest infections, (2) loose bowel motions and (3) failure to thrive. With constipation and no significant history of chest infections, CF is unlikely.

OPTION C and D : Prematurity and the child’s growth centile are irrelevant to the presenting condition.

OPTION E : Diarrhea after introducing cow’s milk is suggestive of cow’s milk allergy. Symptoms allergy may include a rash (eczema or hives), swelling, abdominal pain or vomiting, diarrhea or breathing difficulties. Constipation is not a common feature of cow’s milk allergy.

TOPIC REVIEW

Hirschsprung disease and its clinical presentation

Hirschsprung disease (HD) results from the absence of enteric ganglions in the rectum and/or colon. Hirschsprung disease is approximately 4 times more common in males than females. Nearly all children with Hirschsprung disease are diagnosed during the first 2 years of life. Approximately 50% of affected children are diagnosed before the age of 1 year. The presenting features of HD vary depending on the length of the aganglionic segment and the severity. Common presenting features include:

  1. Delayed passage of meconium - In healthy term neonates, delay of more than 48 hours is indicative of HD. However, this feature occurs in only about 50% of neonates with HD.
  2. Neonatal bowel obstruction - Distal bowel obstruction is a presenting feature in about 25% of newborns with HD. The classical features are distended abdomen, bilious vomiting, fever, dehydration, lethargy, not passed meconium, and occasionally dilated peristaltic loops visible on the per abdominal examination in the neonate with a normal anus.
  3. Neonatal bowel perforation - 5% percent of children with Hirschsprung’s disease have bowel perforation. Features of perforative peritonitis predominate and in this situation the diagnosis of HD is not clear. Hence, during exploration of such neonates, the colon must be examined and appropriate seromuscular biopsies must be done so as not to miss the diagnosis of HD.
  4. Neonatal enterocolitis - About 30% of neonates with HD develop enterocolitis. Sudden onset of diarrhea with or without blood in the stools in neonates should raise the suspicion of HD. The stool is very foul smelling, explosive and is associated with severe abdominal distension, fever, lethargy, dehydration and occasionally generalized sepsis. Hirschsprung’s enterocolitis is a life threatening condition if toxic megacolon develops. Early diagnosis and treatment is of significant importance.
  5. Constipation- Older infants and children typically present with chronic constipation. This constipation often is refractory to usual treatment protocols and may require daily enema therapy.
  • Journal of Neonatal Surgery
  • MedScape - Pediatric Hirschsprung Disease

51

Q

Six days after an uneventful tonsillectomy on a 7-year-old boy, he is brought back to the emergency department after coughing up blood 30 minutes earlier. Which one of the following would be the next best step in management?

A. Packing.
B. Oral antibiotics.
C. Intravenous antibiotics.
D. Take him back to the operating room for exploration.
E. Give him ice cream and cold fluids.

A

C. Intravenous antibiotics

Hemorrhage after tonsillectomy is the most common complication of the procedure. The bleeding may occur in the early post-operative period or may be as delayed as 5 to 10 days. Late bleeding is normally secondary to infection of the tonsillar fossae. Typically, there is initially minor bleeding, but blood-pouring develops afterwards; therefore any bleeding even trivial must be considered serious in all cases. Post-tonsillectomy hemorrhage is a life-threatening condition, which requires prompt management in all cases (even if only little blood is spat out and bleeding stops again – all these cases should be dealt with caution – particularly the children).

NOTE - The chance of post-tonsillectomy hemorrhage increases with age.

On physical examination, a hint of blood in a corner of one of the tonsillar fossae may be noticed. Small grey or clotted blood is another likely fkinding. Parents of younger children may describe finding blood on the child’s pillowcase, or an episode of hemoptysis or hematemesis. Excessive swallowing may also be an indicator of ongoing bleeding in young children

Management of post-tonsillectomy hemorrhage includes:
* Resuscitation – 2 large bore cannula with bloods
* Reservation of cross-matched packed red cells
* Intravenous antibiotics: with 1.2g Benzyl Penicillin, 6-hourly + 500mg of metronidazole, 12-hourly
* Analgesia without NSAIDs (NSAIDs may increase the chance of bleeding)
* Hydrogen peroxide gargle - 20mls of hydrogen peroxide diluted with water in a ratio of 1:6. This gargle should be used every 4 hours; it should be used when a large clot overlies one of the tonsil beds.

Of the given options, the only one in the management plan of this patient is administration of intravenous antibiotics.

OPTION A : Packing the tonsils is neither practical nor effective.

OPTION B : Antibiotics in oral forms are not recommended for management.

OPTION D : Exploration might be indicated; however, most patients will respond to more conservative managements.

OPTION E : The patient should be kept nil by mouth (NBM) because it is very likely that emergency surgery becomes inevitable; therefore giving him ice cream or cold fluids is an incorrect option.

PRACTIVE POINT

If a clot, as describes above, overlies a tonsil bed, it should be removed to see if there is bleeding behind it. Inform the ENT registrar and the anesthetic team, then prepare yourself with a Magill’s forceps, some gauze, 1:10,000 adrenaline, tongue depressor and a headlight. Use the Magill’s forceps to pull off the clot and watch what happens. If there is no bleeding, you can follow the advice given above and admit the patient with H2O2, IV fluids, and NBM.

If the patient starts bleeding, quickly soak a gauze with 1:10,000 adrenaline. Grip the gauze tightly with the tip of the Magill’s forceps and push it into the fossa and hold it there for as long as your patient can cope with it. The idea is to soak the fossa with adrenaline while also blocking the flow of blood. Lean the patient forward so he/she can spit out the blood and saliva. The direction of the pressure should be directly into wall of the mouth (so laterally rather than any posterior or inferior pressure which will cause the patient to gag). Continue this until either the bleeding stops or help arrives.

52

Q

Tanya has brought her 2-year-old son to your clinic, disgruntled and annoyed by the child’s condition. She tells you that 2 months ago, she started to wean the child onto solid foods and currently his diet includes breastfeeding 3 times a day 5 minutes from each of her breasts and a bowl of cereal and rice every morning. The child has dropped
from the 50th to 25th percentile. The child’s past medical history is remarkable for one episode of admission due to bronchiolitis. Which one of the following is the most likely cause of this baby’s diminished weight gain?

A. Cystic fibrosis.
B. Celiac disease.
C. Low calorie intake.
D. Child abuse.
E. Normal physiologic response to weaning.

A

C. Low calorie intake

With a drop in growth percentile, this child has developed failure to thrive.

Failure to thrive is defined as either of the following:
* Body mass index for age less than the 5th percentile
* Length for age less than the 5th percentile
* Weight deceleration crossing two major percentile lines
* Weight for age less than 50th percentile
* Weight less than 75% of median weight for age
* Weight less than 75% of median weight for length
* Weight velocity less than the 5th percentile

The most common cause of failure to thrive is inadequate caloric intake. With 3 sessions of breast feeding and only one bowl of cereal, this baby is definitely receiving far less than his caloric needs and therefore, low caloric intake would be the most likely cause of his problem.

OPTION A : With only one episode of bronchiolitis, the child is unlikely to have cystic fibrosis. Patients with cystic fibrosis often have respiratory infection recurrences.

OPTION B : Celiac disease can be a possibility, especially considering the fact that this child has been started on cereal and rice, both of which contain gluten, but celiac disease is far less common than inadequate calorie intake. Furthermore, the child with celiac disease has more pronounced symptoms e.g., diarrhea.

OPTION D : Child abuse seems unlikely because firstly there is no comment in the question pointing towards the likelihood of child abuse and secondly, the mother seems concerned about her child’s condition and has sought medical care.

OPTION E : Normal physiologic response to weaning could be considered if the baby was receiving adequate nutrition.

53

Q

A mother has brought her 9-month old boy to the Emergency Department with dyspnea and poor feeding. On examination, his chest is hyperinflated, and subcostal recession is noted. On auscultation, there are widespread fine inspiratory crackles and expiratory wheezes. Which one of the following is the next best step in management?

A. Nebulized adrenaline.
B. 100% oxygen via nasal prongs.
C. Intravenous corticosteroids.
D. Intravenous ceftriaxone.
E. 100% oxygen via facemask.

A

B. 100% oxygen via nasal prongs

This clinical presentation in this 9-month old infant is classic for acute bronchiolitis, a viral illness most commonly caused by respiratory syncytial virus (RSV). The usual age for the disease is between 2 weeks to 9 months of age.

The classic clinical presentation is prodromal symptoms (cough, coryza) followed by wheezy breathing, tachypnea and hyperinflated chest with subcostal recessions. In children with bronchiolitis, oxygen therapy should be used when oxygen saturations are persistently less than 90%. Infants with bronchiolitis will have brief episodes of mild/moderate desaturations to levels less than 90% but these brief desaturations are not a reason to start oxygen therapy.

If required, oxygen 100% should be given via nasal cannula and discontinued when oxygen saturations are persistently greater than or equal to 90%.
Dehydration is also a serious problem in children with severe bronchiolitis and requires rehydration using intravenous fluids or nasogastric tube.

Antibiotics are not indicated, unless a bacterial infection is suspected. There is no evidence to support the use of nebulized adrenaline, bronchodilators or corticosteroids in children with bronchiolitis.

  • RCH - Bronchiolitis

54

Q

Which of the following is regarded as the most important clinical exam finding in a child with acute viral bronchiolitis?

A. Fever.
B. Expiratory wheeze.
C. Nasal discharge.
D. Inspiratory fine crackles.
E. Cough with productive sputum.

A

D. Inspiratory fine crackle

Bronchiolitis is an acute viral illness most commonly caused by respiratory syncytial virus (RSV). Usual age is from 2 weeks to 9 months (up to 12 months). It is the most common cause of lower respiratory tract infection in infants.

It presents initially with prodromal symptoms such irritating cough and rhinorrhea and then becomes worse in the next 3 to 5 days.

Findings on examination include:
* Wheezy breathing which is often distressed
* Bilateral fine inspiratory crackles.
* Tachypnea
* Hyper-inflated chest (barrel-shaped)
* Use of accessory muscles and subcostal recession

Chest X-rays are not necessary for diagnosis, but if obtained bilateral perihilar congestion and fullness will be noticed.

Inspiratory fine crackles are regarded as the most characteristic feature of bronchiolitis.

OPTION A : Children with bronchiolitis may have low-grade fever; however, fever is a very nonspecific finding seen in many other respiratory tract infecions.

OPTION B : Expiratory wheezing may be presents in bronchiolitis, as well as asthma and foreing body aspiration. It is NOT a specific feature for bronchiolitis.

OPTION C : Nasal discharge can be a featue in the prodromal phase of bronchiolitis, common cold, influenza, and allergies. It is not specific for bronchiolitis.

OPTION E : Cough can be a feature of bronchiolitis, but firstly it is not productive, and secondly, it may be present in many other respiratory infections such as pneumonia, bronchitis, and asthma.

  • MJA - Bronchiolitis: assessment and evidence-based management * RCH - Bronchiolitis

55

Q

A 2-year-old boy is brought to the Emergency Department by his parents, who are concerned about his fever and difficulty breathing. The fever has started this afternoon. On examination, he looks ill and pale and is grunting. He has a temperature of 39.6°C and his respiration is shallow with a rate of 38 breaths per minute. Chest auscultation is normal. Which one of following could be the most likely diagnosis?

A. Lobar pneumonia.
B. Acute bronchitis.
C. Atypical pneumonia.
D. Bronchiolitis.
E. Foreign body aspiration.

A

A. Lobar pneumonia

The findings of a high fever associated with pallor and respiratory distress in an infant is strongly suggestive of lobar pneumonia. In this age group, chest auscultation may deceitfully be normal due to shallow rapid breathing and grunting. The chest X-ray, however, will show consolidation. The most common causative organism is this situation would be streptococcus pneumoniae.

OPTION B : Acute bronchitis has a more insidious onset and the child is not often as severely ill as this child.

OPTION C : With atypical pneumonia, a prodromal state and more insidious onset is expected. Moreover, fever is not often that high.

OPTION D : Bronchiolitis is the viral infection of bronchioles, caused by respiratory syncytial virus (RSV) in most cases, and is more commonly seen in infants younger than 12 months. There may be low-grade fever, wheezing, bilateral fine crackles and respiratory distress. There is a prodrome of cough and rhinorrhea, followed by more severe symptoms in the following 3 to 5 days. The high fever and acute onset of symptoms favors the diagnosis of pneumonia rather than bronchiolitis.

OPTION E : Although post-obstructive atelectasis can give rise to pneumonia in children with foreign body aspiration, this usually occurs over a course of weeks. The respiratory symptoms such as unilateral wheezing would be evident.

  • Medscape - Pediatric Pneumonia
  • AAFP- Community-Acquired Pneumonia in Infants and Children
  • RCH - Community acquired pneumonia

56

Q

A 13-year-old boy is brought to your practice with fever and malaise for the past 2 days. On examination, he has a temperature of 38°C. There rest of the exam is inconclusive. In an attempt to find the source of the infection, you order a set of laboratory tests including urinalysis (UA). The UA is positive for 1+ proteinuria and RBCs 30/hpf. No white blood cells are present. The urine is negative for nitrates. Which one of the following would be the most appropriate step in management of this patient in terms of the abnormal UA?

A. Renal ultrasonography.
B. Repeat UA after fever settles.
C. Urine culture.
D. Start him on antibiotics.
E. Renal biopsy.

A

B. Repeat UA after fever settles

It is recommended that asymptomatic hematuria during a febrile illness should be assessed after the patient recovers from the illness, provided that the illness is not due to urinary tract infection (UTI). Asymptomatic microscopic hematuria should always be confirmed with 2 positive urinalysis (UA) out of 3 in 2-3 weeks before any further assessment is considered. With hypertension, edema, or renal failure, however, prompt action is mandated.

57

Q

A mother brings her 2-week-old male neonate with noisy breathing, which becomes worse when the baby is laid on his back. The child is otherwise healthy. Which one of the following is the most likely cause of this presentation?

A. Acute asthma.
B. Laryngomalacia.
C. Croup.
D. Acute bronchiolitis.
E. Acute epiglotitis.

A

B. Laryngomalacia

The clinical history is suggestive of laryngomalacia. Laryngomalacia is congenital softening of the tissues of the larynx above the vocal cords. This is the most common cause of noisy breathing in infancy. In laryngomalacia, the laryngeal structure is malformed and floppy, causing the tissues to fall over the airway opening and partially block it. Laryngomalacia symptoms are usually present at birth, and can become more obvious within the first few weeks of life. Most children outgrow laryngomalacia by 18 to 20 months of age.

Symptoms of laryngomalacia include:
* Noisy breathing - an audible wheeze when a baby inhales. It is often worse when the baby is agitated, feeding, crying or sleeping on his back
* High pitched sound
* Difficulty feeding (in severe cases)
* Poor weight gain (in severe cases)
* Choking while feeding (in severe cases)

OPTION A : Acute asthma makes breathing difficult, and wheeze is audible in any position. Asthma is an extremely rare possibility in a 2-week-old neonate.

OPTION C : Croup (laryngotracheobronchitis) is most commonly caused by para-influenza virus type 1 infection. Croup is characterized by a distinctive seal barking cough and inspiratory stridor. Croup affects mainly children aged 6 month to 3 years. This child is in no respiratory distress, is otherwise healthy, and does not have any cough.

OPTION D : Acute bronchiolitis is an acute viral infection of the lower respiratory tract, typically affecting infants younger than 12 months of age, and is characterized by respiratory distress, wheezing, and inspiratory fine crackles. It is primarily caused by respiratory syncytial virus (RSV). Treatment is supportive with oxygen and hydration. Prognosis is generally excellent; however, some children may develop apnea or respiratory failure.

OPTION E : Epiglottitis is a rapidly progressive bacterial infection of the epiglottis and surrounding tissues that may lead to sudden respiratory obstruction and death. Symptoms include severe sore throat, dysphagia, high fever, drooling, and inspiratory stridor. Treatment includes airway protection and antibiotics. This child does not have any of these symptoms.

NOTE - While expiratory stridor indicates an obstruction in the lower trachea, inspiratory stridor suggests obstruction above the glottis.

  • Medscape - Laryngomalacia Clinical Presentation
  • UpToDate - Congenital anomalies of the larynx
  • AFP - Diagnosis of Stridor in Children

58

Q

A 6-month-old male infant with Tetralogy of Fallot (TF) is brought to your clinic by his mother with history of nasal discharge, low-grade fever, feeding difficulty, wheezy breathing, and cough for 3 days. The infant’s father smokes a pack of cigarettes per day at home. Physical examination is significant for inspiratory crackles and expiratory wheeze. Respiratory rate is 60 breaths per minute and oxygen saturation is 92 % on room air. Which one of the following is the next best step in management?

A. Reassure the mother and educate about minimal handling and frequent feeds.
B. Admit the baby to the hospital and consider symptomatic care with supplemental oxygen, minimal handling and use of intravenous fluids.
C. Start corticosteroid therapy immediately.
D. Start nebulized ribavirin and monitor the response.
E. Start the child on nebulized saline 3% immediately.

A

B. Admit the baby to the hospital and consider symptomatic care with supplemental oxygen, minimal handling and use of intravenous fluids

This infant has clinical diagnosis of acute viral bronchiolitis. The diagnosis of bronchiolitis is clinical and based on typical history of increased respiratory rate, cough, expiratory wheezing and bilateral fine crackles, preceded by a prodromal syndrome of low-grade fever and rhinorrhea.

For children with increased risk of progression to severe bronchiolitis, special consideration and probably admission to the hospital and discussion with a senior doctor is required. Such infants are those with:

  • History of significant apnea before assessment
  • Known structural cardiac anomaly, especially large left-to-right shunt (e.g., ventricular septal defect)
  • Known pre-existing lung disease (e.g., cystic fibrosis)
  • Age less than 6 weeks
  • Significant prematurity (< 32 weeks) and/or chronic neonatal lung disease
  • Severe degree of respiratory distress or apnea
  • Significant dehydration
  • Hypoxemia (Spo2 < 93%)
  • Re-presentation to the emergency department within 24 hours
  • Uncertain diagnosis.

In the following situations, hospital admission is definitely indicated:
* Difficulty in feeding
* Increased work of breathing - use of accessory respiratory muscles
* Respiratory rate>50 breaths per minute
* Oxygen saturation less than 93% on room air.
* Apnea
* Dehydration
* Patient or parental exhaustion

This child has SpO2 of 92% and a breathing rate of 60 breaths per minute. He also has as an underlying congenital heart disease. For him, hospital admission is the first thing to consider.

The main issues in management are parental reassurance, education about minimal handling, and frequent feeds. The child should receive intravenous fluids because the illness itself, as well as tachypnea makes the child dehydrated.

OPTION A : Although mentioned measures are appropriate, these should be taken in the hospital after admission. This option does not include hospital admission and is not correct.

OPTION C : Corticosteroids have not shown any benefit in management of bronchiolitis.

OPTION D : Nebulized ribavirin may be used in immunocompromised patients but only provides marginal benefit in respiratory syncytial virus infection. It is expensive, potentially toxic, and rarely indicated. Its use is not recommended without specialist consultation.

OPTION E : There is some evidence that the use of inhaled 3 % hypertonic saline may reduce the length of stay in hospital and improve clinical severity score; however, it is not considered first-line treatment and its use remains controversial.

  • MJA- Bronchiolitis: assessment and evidence-based management
  • RCH - Bronchiolitis

59

Q

A 4-year-old child is brought to your practice with a 2-week history of harsh cough, fever, runny nose and dyspnea. He is short of breath on examination. Which one of the following would be the next best step in management?

A. Nasopharyngeal aspiration for pertussis.
B. Pertussis serology.
C. Chest X-ray.
D. Pulmonary function tests.
E. Blood culture.

A

C. Chest X-ray

With dyspnea, this child is likely to have a pulmonary pathology. Considering this, obtaining a chest X-ray is the most appropriate initial step in management, as an attempt to assess the lungs for the cause of this presentation.

OPTION A and B : Nasopharyngeal aspiration in children (nasopharyngeal swab in adults) for culture or serology is to confirm the diagnosis of pertussis. However, the findings in this scenario are against pertussis, because (1) patients with pertussis are not febrile and (2) lower respiratory tract is not involved. Unless during the bouts of cough, shorteness of breath is not a common feature in pertussis.

OPTION D : Pulmonary function test are not indicated here; at least not as an initial step.

OPTION E : Blood culture can be considered as part of workup later on if indicated. An example is when chest X-ray suggests pneumonia and measures to isolate the causative organism is required. At this stage, however, obtaining a chest X-ray would be the most appropriate initial step.

  • RACGP - AFP - Pertussis presentation, investigation and management
  • Australian Prescriber - Managing pertussis in adults
  • Medscape - Pertussis

60

Q

A mother brings her 8-year-old daughter for consultation because the girl avoids school and goes to games and parlors. Her parents were advised by her teachers to seek medical help. When you talk to the girl, she becomes tearful and looks withdrawn. Which one of the following is most suspected from the history?

A. Separation anxiety.
B. Truancy.
C. Depression.
D. Conduct disorder.
E. ADHD.

A

B. Truancy

Truancy is any intentional unauthorized or illegal absence from compulsory schooling. Children who run away from school to do other things (truancy) usually have different psychosocial issues when compared to those who do not want to go to school and have the preference of staying at home.

Truancy may be an attempt to draw attention or to make an impression on peers. It also can be caused by anger at school or problems at home. Truancy may simply be due to learning problems. Truant children sometimes go on to break laws more seriously when they are older.

A feature of truancy is that the child tries to hide his/her action from the parents. On the other hand, the parents of such children often are not very interested in their children educational state. They are more commonly of low socioeconomic status.

Schools usually expect parents to inform the school in advance if their child is absent through the student diary, or by phoning the school on the day of absence. It is a good way of ensuring child’s safety. After 3 days of unexplained absence, schools are required to contact the parent (s) and report the absence.

OPTION A : This child does not fulfil the criteria for separation anxiety. Children with separation anxiety:
* Want to stay at home with parents
* Get upset about going to school and may have stomach aches or headaches, or do not feel well without a physical cause
* Do not have any serious behavior problems
* Do not try to hide their wish regarding not going to school from their parents
* Are more likely to be the youngest member of a family

OPTION C : Children with depressive disorders feel sad, lack interest in activities they previously enjoyed, criticize themselves, and are pessimistic or hopeless about the future. They may also be irritable and aggressive. They may be indecisive, and have problems concentrating. They tend to lack energy and to have sleeping problems.

OPTION D : The typical behavior of those with conduct disorder involves bullying, frequent physical fights, deliberate destruction of other people’s property, breaking into houses or cars, staying out until late night despite parental prohibitions, running away from home, or frequent truancy from school. In this scenario, there is no misconduct in the history to make this diagnosis likely.

OPTION E: ADHD is characterized by lack of concentration and hyperactivity at least at two distinct environments (e.g. school and home) which is different from this child’s problem.

  • RACGP - School Refusal

61

Q

Which one of the following can be an alarming sign in a 2-month old infant?

A. Inability to smile at people.
B. Inability to hold neck.
C. Inability to explore things by holding and looking at them and putting them in mouth.
D. Does not seem to recognize mother.
E. Does not seem interested in activities around them.

A

A. Inability to smile at people

Normally, social smile develops at 6 weeks of age; therefore its absence in a 2-month-old baby could be concerning.

OPTION A : The ability to hold the neck is obtained at around 3 months.

OPTION B : Ability to explore things by holding, looking at them and putting them in mouth is achieved by 6 months of age.

OPTION D : A skill to recognize mother’s face is achieved at the age of 3 months.

OPTION E : The child starts to show interest in the activities around them at about 9 months of age.

  • http://childrenandfamilies.nt.gov.au/library/scrip
  • http://files.acecqa.gov.au/files/ACECQA/2014/devel * http://www.parenting.sa.gov.au/pegs/peg77.pdf

62

Q

A male neonate born at 36 weeks gestation is noted to have many clinical features suggestive of Down syndrome. Six hours after birth, he starts vomiting. The vomit contains caseated milk and green material suggestive of bile. On examination, the abdomen is mildly distended. Which one of the following is the most likely diagnosis?

A. Hirschprung disease.
B. Duodenal atresia.
C. Necrotizing enterocolitis.
D. Large bowel volvulus.
E. Meconium plug syndrome.

A

B. Duodenal atresia

Children with Down syndrome have an increased risk of gastrointestinal tract anomalies, occurring in approximately 5%.
These anomalies include:
* Duodenal atresia or stenosis, sometimes associated with annular pancreas - the most characteristic lesion, occurring in 2.5%
* Imperforate anus
* Esophageal atresia with tracheoesophageal fistula
* Hirschsprung disease (<1%)

Of the options, early onset of bilious vomiting is explicable by duodenal atresia as the most likely diagnosis.

OPTION A : Although Hirschsprung disease can present in neonatal period, especially in children with Down syndrome, typical presentation occurs after 4 to 5 days after birth with constipation, progressive abdominal distention. Delayed passage of meconium is an early characteristic feature in history. Vomiting is not common.

OPTION C : Necrotizing enterocolitis is almost only seen in sick premature infants who are on maximum support during the acute stage of their complicated illness.

OPTION D : In neonatal period, malrotation with midgut volvulus may occur. Volvulus presents with abdominal pain that may be associated with bilious or nonbilious vomiting; however, Down syndrome, very early onset of vomiting, and more importantly vomiting immediately after breastfeeding in more consistent with duodenal atresia.

OPTION E : Meconium plug syndrome tends to present later (2 to 3 days after birth), and is not associated with Down syndrome.

  • RACGP - AJGP - Common neonatal presentations to the primary care physician * Medscape - Pediatric Duodenal Atresia

63

Q

Lucy has presented to your office concerned about her 3-years-old daughter. Lucy has past medical history of asthma and eczema and heard that these conditions can run in families. She wants to know about the risk of her daughter developing asthma or eczema as well. In consulting her, which one of the following would be the most appropriate advice you can give to minimize the risk allergic reactions in the child?

A. Avoid monosodium glutamate in foods.
B. Avoid contact with sheepskin clothes.
C. Minimizing household dust mites.
D. No specific advice is needed.
E. Giving the child antihistamines in case allergy develops.

A

C. Minimizing household dust mites

Household dust mites are the most common allergens that can cause allergy and asthma in genetically susceptible individuals. About 90% of children with atopic symptoms and asthma demonstrate positive skin-prick responses to dust mite extract.

In general, reducing dust mites is the most appropriate advice here. Avoiding contact with sheepskin bedding and clothes is a part of measures taken to reduce dust mites.

Anti-histamines and avoiding mono-sodium glutamate in food is not recommended for minimizing the risk of atopy in children.

Current evidence suggests that exposure to dust mite allergen can be minimized by:
* Encasing mattresses, pillows and blankets/doonas in dustproof covers
* Polishing floors and replacing carpet
* Washing bed clothes in hot water (>55°C) weekly
* Not using sheepskin bedding

  • ACAAI - Dust Allergy

64

Q

Emily, 18 months old, is brought to your clinic by her parents for a routine health visit. They are new to then town and this is the first time you visit her. She is the outcome of an uneventful pregnancy and has had no major medical problem so far. On examination, you hear a cardiac murmur. Which one of the following clinical findings suggests that the murmur is an innocent murmur?

A. The murmur is diastolic.
B. The murmur is pansystolic.
C. The murmur is associated with a thrill.
D. The murmur increases when the child is positioned supine.
E. The murmur is associated with reduced exercise tolerance.

A

**D. The murmur increases when the child is positioned supine. **

Physiologic (innocent) murmurs are common in healthy infants, children, and adolescents.

Innocent murmurs have the following characteristics (7 ‘S’s):
1. Sensitive – changes with child’s position or with respiration
2. Short duration (not holosystolic)
3. Single – no associated clicks or gallops
4. Small – murmur limited to a small area and non-radiating
5. Soft – low amplitude (up to 2/6 or very rarely to 3/6)
6. Sweet – no harsh sounding (one innocent murmur may be harsher than the other, but they are not generally harsh sounding like pathologic murmurs), no thrills
7. Systolic – occurs during and is limited to systole

With the presence of either of the following, the murmur is pathological:
* Grade 3/6 or higher murmurs
* Harsh quality
* Abnormal S2
* the presence of systolic click
* Symptoms related to cardiac conditions e.g., shortness of breath, chest pain, poor feeding, failure to thrive, etc.
* Increased intensity when the venous return decreases (e.g., when the patient stands)

Also with any of the following every murmur should be considered pathologic until proven otherwise:
* Family history of sudden cardiac death or congenital heart disease
* In utero exposure to certain medications or alcohol
* Maternal diabetes mellitus
* History of rheumatic fever
* History of Kawasaki disease

Physiologic (innocent murmurs) always change in intensity (increase) with maneuvers that increase venous flow to the heart (squatting, lying down, leg raising, etc.) and are decreased in intensity when there is a reduction in venous return (increased intra-abdominal pressure by e.g. Valsalva maneuvers). It should be noted, however, that pathologic murmurs can be affected by different maneuvers as well. An innocent murmur changes with position, but not every murmur that changes with position is innocent.

There only 3 murmurs that increase in intensity when the venous return to the heart is decreased:
1. The murmur caused by hypertrophic obstructive cardiomyopathy (HOCM) – pathologic
2. The murmur caused by mitral valve prolapse – pathologic
3. The venous hum – venous hum is a roaring sound caused by flow of the blood from the superior vena cava best heard on the neck above the clavicle, mostly on the right side.

OPTION A and B : Innocent murmurs are never diastolic. Even holosystolic murmurs are frequently found to be pathologic on further evaluation.

OPTION C : A thrill is never physiologic and an underlying cardiac pathology is always present.

OPTION E : An innocent murmur is always asymptomatic; therefore, symptoms such as decreased exercise tolerance, failure to thrive, poor feeding, etc. are against such diagnosis.

OPTION E : Reduced exercise tolerance is highly suggestive of a cardiac disease, either congenital or acquired.

TOPIC REVIEW

Types of physiological (innocent) murmurs Still’s murmur:

  1. Still’s murmur
    Still’s murmurs are low-pitched sounds best heard at the lower left sternal border. They are musical or have relatively pure tone or may be squeaky. They most commonly occur between 3 years of age and adolescence. They are related to flow; therefore, they change with position.
  2. Pulmonary flow murmur
    These murmurs are harsher and best heard over the upper left sternal border. They are flow-dependent and change in intensity with changing positions. They disappear or decrease with maneuvers that increase the intraabdominal pressure resulting in decreased venous return. These murmurs originate from the right ventricular outflow tract and radiate along the pulmonary arteries and may be best heard in the back and axilla bilaterally.
  3. Pulmonary flow murmur
    Pulmonary flow murmurs can occur at any age, but they are common particularly in adolescents or in children with pectus excavatum. They are prominent in high-flow situations, such as when a child has a fever or is anemic.

In infants, these sounds may be most prominent in the back or axilla because turbulence occurs when the blood flows from the larger main pulmonary artery to the smaller and less-developed distal pulmonary arteries. In fetal life, the main pulmonary artery transports about 90% of the blood to the ductus arteriosus and only about 10% to the distal pulmonary arteries. The main pulmonary artery is thus large, whereas the distal pulmonary arteries are relatively smaller and come off at more acute angles than they do later as the child’s chest grows. An analogy from nature would be the noise that is created as a large stream narrows into smaller streams.

  1. Systemic flow murmurs (supraclavicular systemic bruits)
    These are harsh high-pitched murmurs caused by normal blood flow into the aorta and into the head and neck vessels and are heard best high up in the chest and above the clavicles. There is no ejection click. It has been said that because of these sounds “all children have carotid bruits”; however, the sounds are different in quality from true carotid bruits and are not associated with carotid outflow pathology.
  2. Venous hums
    Venous hums are low-pitched continuous murmurs produced by blood returning from the great veins to the heart. They are heard best with the bell of the stethoscope. By changing the position of the patient’s head or by pressing in the area of the major neck veins, the flow may be changed and these murmurs will change or disappear. Having the child look down or to the side while listening, will often make these murmurs or sounds disappear.

NOTE - Still’s murmur and venous hum are low-pitched and best heard by the bell of the stethoscope, whereas pulmonary flow murmurs and systemic flow murmurs are high-pitched and best detected by the diaphragm of the stethoscope.

  • AAFP - Evaluation and Management of Heart Murmurs in Children
  • GP Notebook - Pediatric murmur

65

Q

A one-year-old female child is presented to you for a health checkup. She has normal growth parameters. She can sit unsupported, stand with assistance, and say ‘mama’. On examination, a 2/6 systolic murmur is heard over the apex. The rest of the exam is completely normal. Which one of the following would be the most appropriate next step?

A. Order ECG and chest X-ray.
B. Refer her to pediatric cardiologist.
C. Reassure the parents as this is an innocent murmur.
D. Refer her for emergency echocardiography.
E. Tell the parents that she can have a congenital heart disease.

A

B. Refer her to pediatric cardiologist

Heart murmurs are common in healthy infants, children, and adolescents. Despite the fact that most heart murmurs are not pathologic, a murmur may be the only manifestation of a serious underlying heart disease.

With either of the following present, the murmur should be considered pathological unless proven otherwise:
* Family history of sudden cardiac death or congenital heart disease In utero exposure to certain medications or alcohol
* Maternal diabetes mellitus
* History of rheumatic fever
* History of Kawasaki disease

When a murmur is presents on heart auscultation, a thorough physical exam, including the following, should be performed :
* Vital signs
* Peripheral pulses and perfusion
* Exercise tolerance (age-matched)
* Auscultation over the heart valves
* Looking for respiratory or gastrointestinal manifestations of cardiac diseases

An innocent murmur is a physiologic murmur with the following characteristics (7 ‘S’s):
1. Sensitive – changes with child’s position or with respiration
2. Short duration (not holosystolic)
3. Single – no associated clicks or gallops
4. Small – murmur limited to a small area and non-radiating
5. Soft – low amplitude (up to 2/6 or very rarely to 3/6)
6. Sweet – no harsh sounding (some innocent murmurs my sound harsher than the others, but generally physiologic murmurs are not harsh in nature), no thrills
7. Systolic – occurs during and is limited to systole

With the presence of either of the following, the murmur is pathologic:
* Grade 3/6 or higher murmurs
* Harsh quality
* Abnormal S2
* The presence of a systolic click
* Increased intensity with decreased venous return (e.g., when the patient stands)
* The patient has any symptoms that could be related to a cardiac condition (e.g. shortness of breath, chest pain, poor feeding, etc.)

In the following situations, the patient should be referred to a pediatric cardiologist:
* Murmurs in infancy, especially in the first 6 months
* There is a family history of sudden cardiac death or congenital heart disease
* There are cardiac symptoms – shortness of breath, poor feeding, failure to thrive, chest pain, fatigability, etc.
* There are chromosomal abnormalities
* The diagnosis is equivocal

This child is completely healthy and has no associated physical finding suggestive of a cardiac pathology. The murmur itself, on the other hand, has all characteristic features of an innocent physiologic murmur, but since she is in the infancy period, referral to a specialist (preferably a pediatric cardiologist) is the most appropriate option to consider.

OPTION A : Chest X-ray and electrocardiography are of little value in assessment of the underlying cause of a murmur in a child. Chest X-rays were obtained previously as a part of assessment, but not an acceptable practice today.

OPTION C : Although this murmur is likely to be innocent, parents cannot be reassured unless full evaluation by a pediatric cardiologist excludes any pathologic etiology.

OPTION D : Echocardiography is the investigation of choice that may be considered by the pediatric cardiologist for further assessment.

OPTION E : Although parents should be informed of all possibilities, telling them that their baby has heart disease while the murmur is more likely to be innocent and no further assessment has been carried out is not appropriate.

  • AAFP - Evaluation and Management of Heart Murmurs in Children
  • FP Notebook - Pediatric Murmurs
  • GP Notebook - cardiac murmur (innocent)

66

Q

Ten hours after uneventful delivery of a 3200 g male baby, he develops central cyanosis. He is immediately started on supplemental oxygen but there is no improvement. On auscultation, no murmur is heard. Which one of the following is the most likely cause of this presentation?

A. Tetralogy of Fallot.
B. Patent ductus arteriosus.
C. Ventricular septal defect.
D. Transposition of the great arteries.
E. Arterial septal defect.

A

D. Transposition of the great arteries

The cyanotic congenital heart lesions are associated with right to left shunts. Of the given options, patent ductus arteriosus, ventricular septal defect, and arterial septal defect are associated with left to right shunts; therefore, no cyanosis is expected.

The only two options associated with cyanosis are Tetralogy of Fallot (TOF) and Transposition of the Great Arteries (TGA).

In TOF, as the name implies, there are 4 defects:
1. Pulmonary infundibular stenosis
2. Overriding aorta
3. Ventricular septal defect
4. Right ventricular hypertrophy

In TOF, the VSD gives rise to a systolic murmur, but this child has no cardiac murmurs on auscultation. This excludes TOF as a diagnosis.

TGA is the most common cyanotic congenital heart lesion that presents in neonates. In TGA the aorta arises from the right ventricle and the pulmonary artery from the left ventricle. Infants with TGA are usually born at term, with cyanosis developing within hours of birth. The clinical course and manifestations depend on the extent of intercirculatory mixing and the presence of associated anatomic lesions. If there are no associated defects such as ventricular septal defect (VSD) no murmur is heard.

  • Medscape - Transposition of the Great Arteries

67

Q

Regarding patent ductus arteriosus, which one of the following statements is incorrect?

A. It frequently occurs as an isolated phenomenon.
B. Cyanosis is usually present.
C. It causes a pansystolic ‘machinery’ murmur at the left sternal border.
D. There is a wide pulse pressure.
E. Definite treatment is often by surgical closure.

A

B. Cyanosis is usually present

During fetal life, ductus arteriosus, a conduit connecting the pulmonary artery to the proximal descending aorta, is patent (open). This patency is vital to the fetus because it allows most of the blood from the right heart to bypass the fetal fluid-filled lungs and enter the aorta, and from there, flow into the systemic circulation. After birth and when the neonate should take over breathing and oxygenation, it is essential for ductus arteriosus to close for prevention of mixing oxygenated and deoxygenated blood; otherwise, a shunt of oxygenated blood from higher- pressure aorta to the lower-pressure pulmonary artery will occur. With left-to-right shunts, such as in patent ductus arteriosus, cyanosis is not a feature until very late in the course of the disease.

OPTION A : Patent ductus arteriosus (PDA) is often an isolated congenital cardiac problem with no association with other cardiac anomalies.

OPTION B : PDA is associated with left-to-right shunting; therefore, cyanosis would not be a presenting feature; however, if untreated, right-to- left shunting and cyanosis eventually follows.

OPTION C and D : Small lesions are expected to be asymptomatic, but larger ones present with bounding (strong) arterial pulses, wide pulse pressure, the typical systolic machinery murmur, and congestive heart failure in severe or prolonged cases.

OPTION E : In some cases, PDA closure occurs spontaneously within one week. Lesions persisting beyond one week are very unlikely to close on their own. NSAIDs, by inhibiting prostaglandin synthesis play a role in management; nonetheless, closure by surgical intervention is needed for cases with persisting PDA.

  • Medscape - Patent Ductus Arteriosus (PDA)

68

Q

A 4-year-old boy is brought to your practice with a second urinary tract infection (UTI) in the past 4 months. Last time, an ultrasound scan revealed a grade II vesicoureteral reflux (VUR). In addition to treatment of the current infection, which one of the following would be the most important action to take for prevention of further UTIs?

A. Intermittent antibiotics.
B. Practicing good hygiene.
C. Surgical correction of the VUR.
D. Continuous antibiotics.
E. Circumcision.

A

D. Continuous antibiotics

Vesicoureteral reflux (VUR) (retrograde flow or urine from the bladder into the ureter and kidneys) is an anatomic and functional disorder that potentially can result in significant morbidity, both from acute urinary tract infections and from the sequelae of reflux nephropathy.

After entering the bladder through the muscular hiatus of the detrusor muscle, the normal distal ureter passes through a submucosal tunnel before opening into the bladder lumen via the ureteral orifice. If the length of the submucosal tunnel or its muscular backing is inadequate, the valve mechanism is incompetent and causes reflux.

**Voiding cystoureterogram is the criterion standard for both diagnosis and staging of the VUR.
**
The International Classification System for VUR is as follows:
1. Grade I – Reflux into the nondilated ureter
2. Grade II – Reflux into the ureter and renal calyces without dilation
3. Grade III – Reflux with mild to moderate dilation and minimal blunting of the fornices
4. Grade IV – Reflux with moderate ureteral tortuosity and dilation of pelvis and calyces
5. Grade V – Reflux with gross dilation of ureter, pelvis and calyces, loss of papillary impressions, and ureteral tortuosity

The condition often resolves with growth. The lower the grade, the higher the chance of resolution would be.

Prophylaxis against urinary tract infection (UTI) should be started after a child has completed treatment of the first episode of UTI and continued at least until a VCUG has been performed. If no VUR is detected, prophylaxis can be discontinued. If VUR is present, prophylactic antibiotics are continued until:
1. VUR resolves
2. VUR is surgically repaired
3. The child grows old enough and prophylaxis is no longer considered necessary
4. Since a significant number of children will have spontaneous resolution of VUR (50-85% of cases with grade I-III), medical treatment spares this group the morbidity from surgery, while protecting the kidneys from more damage.

Medical treatment consists of antibiotics dosed at 1/4 – 1/3 of the therapeutic dosage and regular follow-up and imaging. A typical routine includes renal ultrasound and VCUG, or nuclear cystography every 12-18 months.

In boys with persistent VUR, who have not had recurrent UTIs, antibiotics are often discontinued as the boys approach puberty. Surgery may be recommended in girls approaching puberty who have persistent VUR.

NOTE - In the past few years, the role of prophylaxis with antibiotics has been challenged. Several studies failed to find any decrease in the incidence of UTIs in children with VUR who take antibiotics. However, these studies had significant flaws, making it difficult to state with certainty that antibiotic prophylaxis is ineffective. Accordingly, the National Institutes of Health (NIH) sponsored a large prospective clinical trial of prophylaxis in an effort to answer this key question. In 2014, the results of the Randomized Intervention for children with Vesicoureteral Reflux (RIVUR) study showed that antibiotic prophylaxis with trimethoprim-sulfamethoxazole was associated with a decrease of approximately 50% in the incidence of recurrent UTIs among children with VUR, in comparison with placebo. Based on these results prophylactic antibiotics for all grades of VUR remains in practice.

This boy with recurrent UTIs needs continuous prophylactic antibiotics.

OPTION A : While continuous prophylactic antibiotics have shown to decrease UTI recurrences, intermittent use of antibiotics has not proven effective.

OPTION B : Although practicing good hygiene is an appropriate advice to give, it does not specifically reduce the risk of recurrent UTIs in children with VUR.

OPTION C : Surgery is considered for sever VURs (grade V), girls approaching puberty whose VUR persist, or patients with recurrent UTIs despite prophylaxis. A grade II reflux is very likely to resolve spontaneously with growth and only needs medical management.

OPTION E : Circumcision of infant males with VUR may be considered based on an increased risk of urinary tract infections in boys who are not circumcised compared to those who are. Although there are insufficient data to evaluate the degree of this increased risk and its duration, parents need to be made aware of this association so that they can decide whether their children are circumcised.

  • UCSF - Vesicoureteral Reflux (VUR)
  • Medscape - Pediatric Vesicoureteral Reflux
  • American Urological Association - Management and Screening of Primary Vesicoureteral Reflux in Children

69

Q

You are called by the nurse on duty to assess a 24-hour old female neonate in the neonatal ward. Physical examination is remarkable for the knees being at unequal levels when the hips and knees are flexed, asymmetric gluteal folds, and limited abduction of the right hip. The history is significant for breech delivery. Which one of the following is the most likely diagnosis?

A. Osgood-Schlatter disease.
B. Slipped capital femoral epiphysis.
C. Developmental dysplasia of the hip.
D. Femoral head fracture.
E. Legg-Calve-Perthes disease.

A

C. Developmental dysplasia of the hip

The exam findings are characteristic of developmental dysplasia of the hip (DDH), also called congenital hip dislocation.

DDH can be unilateral or bilateral. Both genetic and environmental factors contribute to the condition. With positive family history, bilateral involvement is more common. Genetic as well as environmental factors has been implicated for DDH. Environmental factors include intrauterine malposition (especially breech with extended legs) and maternal production of relaxin shortly before delivery (relaxin is a ligament-relaxing hormone)

Female newborns are 6 times more likely to be affected. Every newborn should be screened for the condition because if left untreated it can lead to disabling morbidity.

DDH can present with the following features on examination:

  • Diminished abduction in flexion of the affected hip
  • Apparent inequality of legs with the affected leg being shortened and externally rotated
  • Asymmetrical skin creases of the groin and thigh
    ‘clicking’ on hip movements

In DDH the femoral head of the affected side is subluxed, dislocatable or dislocated. This, if left untreated, leads to early degenerative joint disease of the hips.

Two tests are used to diagnose DDH:

  1. Barlow test – The affected femoral head can be telescoped in and out of the acetabular socket by piston-like ‘telescoping’ movements in the long axis of the flexed and abducted thighs. This maneuver is often done at the same time as Ortolani test. Great caution should be taken to avoid further ligamental injuries.
  2. Ortolani test – The child’s flexed hips are abducted. The thighs are the grasped between the thumbs in front and other fingers behind. The child’s knees are flexed and hip flexed to a right angle. Each thigh is steadily abducted towards the examining surface while the fingers behind apply forward pressure. Positive findings include reduction of the dislocated femoral head into the acetabulum with an audible and palpable ‘jerk’ or ‘clunk’ (not a click). (Positive Ortolani sign)

NOTE - Barlow test is always performed before Ortolani test.

OPTION A : Osgood-Schlatter disease is the overuse apophysitis of the tibial tubercle resulting in localized pain, especially with quadriceps contraction in active young boys.

OPTION B : Slipped capital femoral epiphysis is the separation of the proximal femoral epiphysis through the growth plate, possibly as a result of imbalance between growth hormone and sex hormones, and often seen in early adolescence (the typical patient is an overweight boy of 10-14 years of age).

OPTION D : With no history of trauma, femoral head fracture is unlikely. Furthermore, femoral head fracture presents differently.

OPTION E: Legg-Calve-Perthes disease is an idiopathic avascular necrosis of the femoral head most commonly found in boys of 4 to 10 years of age.

  • Royal Prince Alfred Hospital – Developmental dysplasia of the hip
  • SA Health – Neonatal Hip Screening and Management of Developmental Dysplasia of the Hip * Diagnostic Imaging Pathways – Paediatrics, Hip Developmental Dysplasia

70

Q

A 6-year-old boy is brought to your practice by his mother with complaints of pain in the right knee and limping. He has had flu-like symptoms 10 days ago. On examination, he has a temperature of 38 °C. Hip movements are limited in all directions, but limitation of internal rotation and abduction are more pronounced. An ultrasonography of the right hip is arranged that reveals widening of the right hip joint space. Which one of the following is the most appropriate management?

A. Start him on antibiotics.
B. Start him on analgesics.
C. Immobilize the affected joint.
D. Start him on systemic corticosteroids.
E. Bed rest.

A

E. Bed rest

Hip joint pain and limp in children may have different etiologies. In children aged 4 -10 years, transient synovitis (TS) also known as irritable hip is the most common cause of hip pain. Legg-Calve-Perthes disease is another cause with a predilection for male children. TS is the most common cause of nontraumatic hip pain in children.

`Clinical manifestations of TS include:

  • Arthralgia and arthritis secondary to a transient inflammation of the synovium of the hip
  • Unilateral hip or groin pain is the most common symptom reported; however, some patients may report medial thigh or knee pain
  • There is often no fever, or only a mild fever
  • Some patients with transient synovitis may not report pain and may present with only a limp`

Diagnosis is based on history, physical examinationو and ultrasonography. Hip X-rays are performed to exclude other possible causes. On physical examination, mild restriction of motion, especially abduction and internal rotation is noted; however, one-third of patients with transient synovitis do not have limitation of hip movements. The hip may be painful even with passive movement, or tender to palpation.

The most sensitive test for transient synovitis is the log roll test in which the patient lies supine and the examiner gently rolls the involved limb from side to side. This may detect involuntary muscle guarding of one side when compared to the other side.

Radiographs exclude bony lesions such as occult fractures or osteoid osteoma. Medial joint space may be slightly wider in the affected hip. Radiographs are often used as the initial diagnostic test.

Ultrasonography (US) is the most accurate modality for detection of intracapsular effusion, but does not help in determining the etiology. An effusion is present if US demonstrates capsular distention greater than 2 mm. US cannot exclude osteomyelitis or soft tissue infection. US is used if X-rays are normal despite clinical suspicion of TS.

Management of TS is protection of the hip by bed rest for 7-10 days and use of crutches to avoid weight bearing of the affected joint. Paracetamol or NISAIDs (e.g.و ibuprofen) may help.

High fever is rare, and if present should raise suspicion against more serious conditions such as septic arthritis

OPTION A : TS is not an infectious diseases. Antibiotics have no role in treatment of TS.

OPTION B : NSAIDs and paracetamol may have a role in management, but relative joint rest is the mainstay of therapy in most cases.

OPTION C: While bed rest and use of crutches is advised, strict immobilization of the hip is not recommended.

OPTION D : TS is a benign condition that resolves spontaneously within 1 to 4 weeks. Corticosteroids are unnecessary for treatment.

  • UpToDate- Overview of hip pain in childhood
  • Medscape - Transient Synovitis
  • Therapeutic Guidelines – Rheumatology; available from http://tg.org.au

71

Q

A 4-year-old boy is brought to you with a painful left knee and limping. Recently, the child had a diagnosis of ‘irritable hip’. On examination, internal rotation and abduction of the left hip joint is limited. An X-ray of the hip shows increased joint space. Which one of the following is the next best step in management?

A. Encourage activity.
B. Use crutches and encourage bed rest.
C. Do a follow up X-ray next week.
D. Hip joint aspiration under general anesthesia.
E. Referral to an orthopedic surgeon.

A

E. Referral to an orthopedic surgeon

The clinical and radiological findings are highly suggestive of Legg - Calve - Perthes disease as the most likely diagnosis.

Legg-Calve-Perthes disease is caused by idiopathic avascular necrosis of the femoral epiphysis, possibly due to repeated multiple vascular occlusive episodes that involve the femoral head. It typically presents with hip/knee pain and/or limp of acute or insidious onset, most commonly in Caucasian boys between the ages of 4 and 10 years, with peak incidence at 5 to 7 years of age. In 15% of the patients the condition is bilateral.

In children older than 8 years, the condition is associated with higher incidence of osteoarthritis in adulthood. Younger children (less than 6 years old) have a greater capacity for the acetabulum to undergo remodeling to accommodate irregularities in the shape of the femoral head.

As there is a significant risk of femoral head destruction due to avascular necrosis, immediate referral to orthopedic specialist is essential and the most appropriate next step in management.

OPTION A : Encouraging activity is not an appropriate option because reducing the weight on the hip joint is an essential part of treatment.

OPTION B : Activities should be restricted and the patient should be offered to use crutches at least until the time he is reviewed by an orthopedic surgeon. Although, use of crutches and encouraging bed rest are partially effective, they are not the definite management for such patients and are only temporary measures to prevent worsening of the problem.

OPTION C : Legg-Calve-Perthes requires urgent referral and delayed referral with further investigations is not appropriate. The condition will not improve spontaneously and follow-up X-rays unnecessarily delays appropriate treatment.

OPTION D : Aspiration under general anesthesia is performed to exclude septic arthritis if history suggest such diagnosis. This child is afebrile and his problem has had an insidious development, making septic arthritis less likely.

  • Medscape - Legg-Calve-Perthes Disease
  • UptoDate – Approach to hip pain in childhood

72

Q

A 2-year-old boy is brought to your clinic due to a 3-day history of fever with occasional cough and coryza. On clinical history, viral upper respiratory tract infection is suspected. On physical examination, the patient is noted to have down-slanting of palpebral fissures, eyes that are apart more than usual and low-set ears. The height of the patient is smaller than the standard. Which one of the following is not consistent with this condition?

A. Pulmonary valve stenosis.
B. Myopia and ectopic ocular lens.
C. An autosomal disorder involving mutation in chromosome 11.
D. Webbed neck.
E. Abnormalities in cardiac conduction and rhythm.

A

B. Myopia and ectopic ocular lens

This clinical picture is suggestive of Noonan syndrome. This is an autosomal dominant disorder with mutation of chromosome 11. It has been described as the male Turner’s syndrome, but affects both sexes.

Clinical features of the syndrome include the following:

  • Facial/eye features
  • Triangular face
  • Hypertelorism (increased distance between the eyes)
  • Down-slanting eyes
  • Ptosis
  • Strabismus – seen in 48%
  • Amblyopia – seen in 33%
  • Refractive errors – seen in 61%
  • Low-set ears with thickened helices
  • High nasal bridge
  • Short webbed neck

Chest/back features
* Pectus carinatum/excavatum
* Scoliosis

Cardiac features
* Stenotic/dysplastic pulmonic valve is the characteristic cardiac anomaly
* Hypertrophic cardiomyopathy (obstructive and non-obstructive) – seen in as many as 30%
* Virtually all types of congenital heart defects have been described in patients with Noonan syndrome

Abdominal features
* Hepatosplenomegaly unrelated to cardiac disease – seen in 25% of the patients

Genitourinary features
* Renal anomalies are present in 10% of patients but are not clinically significant
* More than half of male patients have undescended testes

Skeletal features
* Joint laxity is present – seen in more than 50% of the patients
* Less common findings include talipes equinovarus (club foot) that is an excessively turned-in foot and high medial longitudinal arc, radioulnar synostosis (fusion), cervical spine fusion, and joint contractures.

Skin findings
* Lymphedema
* Prominent pads of fingers and toes – seen in 67%
* Follicular keratosis of face and extensor surfaces – seen in 14%
* Multiple lentigines (a lentigo is a small, sharply circ*mscribed, pigmented macule surrounded by normal- appearing skin) – seen in 3%

Neurological findings
* Hypotonia
* Seizure disorders – seen in 13%
* Unexplained peripheral neuropathy – not seen frequently
* Mental retardation

Previously, patients with Noonan syndrome were thought to have a form of Turner syndrome, as clinical picture is similar to that of Turner’s.

Myopia and ectopic ocular lens is consistent with Marfan syndrome and are not features seen in Noonan syndrome.

  • Medscape - Noonan Syndrome

73

Q

During physical examination of a 3-kilogram full-term male newborn, you notice that the baby’s oxygen saturation decreases when he is laid on his back. The fall in oxygen saturation is sometimes associated with mild cyanosis. However, when the baby starts crying the oxygen saturation increases again. Which one of the following will be the next best step in management?

A. Rigid nasogastric tube.
B. Chest X-ray.
C. Surgery.
D. Intubation and ventilation.
E. Oxygen via nasal cannula.

A

A. Rigid nasogastric tube

The presentation is classic for choanal atresia. Choanal atresia is a congenital abnormality in which there is failure of canalization of the bucconasal membrane. This can be either unilateral or bilateral and is usually due to a combination of bone and soft tissue anomalies. It occurs in 1:5000 to 1:8000 births. Unilateral atresia is more common, with a predilection for females. CHARGE Syndrome (coloboma, heart defect, atresia choanae, retarded growth and development, genital abnormality and ear abnormality) or other congenital abnormalities are present in 50% of patients with bilateral choanal atresia.

Bilateral choanal atresia is a relatively rare anomaly of the upper airway. As neonates are obligatory nasal breathers, presentation may be with life-threatening respiratory distress, retractions and paradoxical cyanotic episodes which are relieved by crying as the infant begins to breathe via the mouth. Unilateral choanal atresia does not usually produce severe symptoms.

When choanal atresia is suspected the diagnosis can be confirmed with trying to pass a nasogastric tube. It is not possible to pass a nasogastric tube through the nares and choanae if there is choanal atresia.

In the presence of bilateral choanal atresia, as the history and physical suggest, oxygen through nasal cannula will be of no benefit.

Other measures to consider as parts of management plan include:

  • Oxygen
  • Oropharyngeal airway
  • Intubation and ventilation if there is life-threatening hypoxemia
  • CT scan
  • ENT consultation and referral for surgical treatment

Since the manifestation is typical of choanal atresia, chest X-ray is not indicated now, but it may be obtained to exclude other possible associated respiratory conditions or anomalies.

  • Western Australia Health Department - Choanal Atresia
  • Medscape - Choanal Atresia

74

Q

A 3-year-old boy is brought to the Emergency Department with complaints of fever and shortness of breath. His symptoms began 2 days ago with mild fever and rinorrhea. Today, he has a fever of 39°C, nasal flaring, tracheal tug and intercostal recession. He has a harsh cough as well. Which one of the following would be the next best step in management of this child?

A. Nebulized salbutamol.
B. Nebulized budesonide.
C. Intramuscular adrenaline.
D. Intravenous penicillin.
E. Oxygen by nasal prongs.

A

B. Nebulized budesonide

The clinical picture is highly suggestive of croup as the most likely diagnosis. Croup (acute laryngotracheobronchitis) presents with a coryzal prodrome, hoarseness (or husky voice in those old enough to speak), biphasic stridor, a harsh barking ‘brassy’ cough and variable airway obstruction due to inflammatory edema within the subglottis. Fever is often mild by can be as high as 39°C.

It is most common in 1- to 3-year-old children and is generally self-limiting, with a duration of 2 to 5 days. Parainfluenza viruses are the most common causes of croup.

Treatment of croup depends on the severity. In terms of severity, croup can be classified as mild, moderate, and severe based on the criteria outlined in the following table: (see photo)

  1. In mild cases (a croupy cough but no history of noisy breathing in a child older than 2 years), no treatment is required, but early review is essential.
  2. In moderate to severe cases (e.g., a history of noisy breathing or age less than 2 years, or presence of stridor on examination), a single dose of corticosteroid is indicated. Use either:
  • Dexamethasone 0.3 mg/kg orally (first-line), OR
  • Predniso(lo)ne 1mg/kg orally, OR
  • Budesonide 2mg by nebulizer

Further doses are usually not needed, but can be considered if response to therapy is suboptimal at 24 hours

  1. In more severe cases with significant airway obstruction or fatigue, treatment in hospital may be required with an initial dose of:
  • Adrenaline 1% (1:1000, 10 mg/ml) solution 0.05 ml/kg/dose up to 0.5 ml diluted with NaCl 0.9%, by nebulizer
    PLUS EITHER
  • Dexamethasone 0.6mg/kg orally (or IM if vomiting) or
  • Predniso(lo)ne 1mg/kg orally

Maintenance dose and its frequency vary with the severity of the condition and the response to treatment.

This child with nasal flaring and tracheal tug and stridor is classified as having moderate croup and should be treated with oral or IM dexamethasone, or oral predniso(lo)ne, or nebulized budesonide.

OPTION A : Nebulized salbutamol would be used if the child had an asthma attack.

OPTION C : Intramuscular adrenaline has no role in management of croup, but nebulized adrenaline is indicated in severe croup.

OPTION D : Antibiotics are not indicated in treatment of croup.

OPTION E : Supplemental oxygen is not often required, except in children with severely obstructed airway. Even so, nebulized adrenaline takes precedence to provide a patent airway for oxygen supplementation.

  • The Royal Children’s Hospital (RCH) - Croup (Laryngotracheobronchitis)
  • Therapeutic Guidelines – Antibiotic; available from: http://tg.org.au
Normal & Abnormal Growth and Development/Child Health/Paediatrics Flashcards by Lubi Mupwaya (9)

75

Q

A mother comes to your clinic because she is concerned about her 5-year-old son. He has started soiling his underwear every night since he entered school 3 months ago. His teacher also has asked her to come for a discussion about his behavior that seems inappropriate for his age at school. On examination, the boy seems small for his age. Which one of the following can be the most likely cause of the soiling?

A. Constipation.
B. Autism.
C. Cystic fibrosis.
D. Hirschsprung disease.
E. Anorectal malformations.

A

A. Constipation

The scenario represents a case of encopresis. Encopresis is defined as the voluntary or involuntary passage of formed, semiformed, or liquid stool into a place other than the toilet for more than one time per month for at least 3 months in a child older than 4 years.

Encopresis is either primary or secondary:
1. Primary encopresis: the child has never been continent
2. Secondary encopresis: fecal incontinence in a child who was previously continent

Encopresis is a common condition reported in approximately 1-4% of school-aged children.

Most fecal incontinence in children is functional, meaning there is no known organic defect. Approximately 80-90% of fecal incontinence results from chronic constipation, which leads to impaction and overflow soiling.

Constipation can be caused by the child repetitively attempting to avoid defecation or holding the stool because of pain, fear, or not liking defecation or using the toilet. This can lead to further constipation. It may also be associated with behavioral, emotional, or psychiatric issues and impacts the child’s physical and psychological development.

For many children with encopresis, there may be a history of an event that made having a bowel movement uncomfortable or frightening. This could range from fear of pain or toilet flushing to repeated sexual abuse.

Non-retentive encopresis occurs if the child refuses to defecate in an appropriate place, e.g., the toilet, but has no history of constipation. These children typically soil daily or regularly but their bowel movements are of normal size and consistency. While most children with encopresis have not experienced sexual abuse, if there is a history of early sexual abuse, encopresis may be more likely.

Non-functional encopresis (caused by medical or physical problems) accounts for about 5- 10% of cases. This is known as organic encopresis. Common organic causes of encopresis include:
* Anorectal malformations
* Spinal cord abnormalities
* Hirschsprung disease
* Certain medications.

This child is small for his age. Moreover, it is mentioned, by the school teacher, that his behavior is not appropriate for his age. With these in history, this child should be further evaluated for underlying problems, yet constipation remains the most likely cause of his problem.

NOTE - In the following situations the child should be referred for further assessment:

  • Soiling is prolonged and resistant to treatment
  • Associated significant behavioral problems
  • Soiling associated with day wetting
  • Soiling not associated with fecal retention and overflow

OPTION B : Gastrointestinal symptoms such as constipation may be seen with increased frequency in autistic children but this child has no features suggestive of autism in clinical exam or history. Such features include impaired language, stereotypic behavior, social inappropriateness, etc.

OPTION C: Cystic fibrosis can cause constipation and encopresis; however, cystic fibrosis would have presented earlier with recurrent chest infection if it was the underlying cause. Cystic fibrosis presents with meconium ileus (20% of patients), respiratory symptoms (45% of patients), and failure to thrive (28% of patients).

OPTION D : Hirschsprung disease is one of the non-functional (organic) causes of encopresis, but it presents much earlier in the neonatal and infantile period.

OPTION E : Anorectal anomalies can cause non-functional encopresis; however, it is less common than constipation as the underlying etiology.

  • Functional fecal incontinence in infants and children: Definition, clinical manifestations and evaluation

76

Q

A mother brings her 7-year-old son who has measles and wants to know if the child needs to be excluded from school. The diagnosis was made when the typical rash developed 6 days ago and he has not attended his class since then. The teacher has informed her that all children in the class has been immunized against measles. Which one of the following would be the most appropriate advice?

A. No more exclusion is required.
B. He should be excluded from school for a total period of 14 days after the rash.
C. He should be excluded for another 4 days.
D. He shoudl receive MMR vaccine before he can go back to school.
E. He can go to school when the rash completely resolves.

A

A. No more exclusion is required

All children with measles should be excluded from school for at least 4 days after the onset of the rash.

Immunized and immune contacts are not excluded. Non-immunized contacts of a case are to be excluded from child care until 14 days after the first day of appearance of rash in the last case, unless immunized within 72 hours of first contact during the infectious period with the first case. All immunocompromised children should be excluded until 14 days after the first day of appearance of rash in the last case.

Since more than 4 days has passed since the onset of the rash no more exclusion is required and he can go back to school.

Vaccination is not indicated anymore because this child will have life-long immunity to measles. However, MMR vaccine should still be given for protection against rubella and mumps at due time.

*https://www.nhmrc.gov.au/_files_nhmrc/publications

77

Q

The baby whose photograph is shown in the following photograph was born to an epileptic mother who had been on treatment with carbamazepine throughout the pregnancy. She is smoker but has cut down on it after the first trimester. She also had had one standard drink throughout the first trimester but stopped when she found out these might have detrimental effects on her baby. She was not supplemented with folic acid as recommended. Which one of the following could be the most likely cause of the birth defect of this child?

A. Smoking.
B. A sporadic or familial genetic condition.
C. Folic acid deficiency.
D. Carbamazepine.
E. Alcohol.

Normal & Abnormal Growth and Development/Child Health/Paediatrics Flashcards by Lubi Mupwaya (10)

A

B. A sporadic or familial genetic condition

The picture shows a cleft lip. Orofacial clefts are the outcome of interaction between genetic and environmental factors; however, genetic defects either sporadic or familial are the most important factors in developing orofacial clefts.

Enviromental factors can increase the risk of orofacial clefts in a genetically susceptible fetus. These factors include :

  • Medications - antiepileptic drugs (e.g., phenytoin, sodium valproate, and topiramate), methotrexate (folic acid antagonist). It should be noted that while the relative risk of orofacial abnormality is increased, the absolute risk of such defects after drug exposure remains small.
  • Smoking
  • Alcohol
  • Folic acid deficiency
  • Maternal obesity
  • Maternal diabetes

===

  • UpToDate - Etiology, prenatal diagnosis, obstetric management, and recurrence of cleft lip and/or palate
  • RCH - Cleft lip and cleft palate
  • Medscape - Pediatric Cleft Lip and Palate

78

Q

A 3-year-old Aboriginal boy is presented to you after he sustained a fit at home 20 minutes ago. According to his mother, the fit started with the boy falling on the ground and having jerky movements of the arms and legs. He was unresponsive during the attack. He never had such an episode before. On examination, the boy looks lethargic but oriented. His temperature is 37.2°C.The rest of examination is inconclusive. Of the following options, which one is most likely to reveal the cause of this seizure?

A. Vitamin D level.
B. EEG.
C. CT scan of the brain.
D. Chest X-ray.
E. Toxicology screen.

A

A. Vitamin D level

The following (VITAMINS) can be the causes of an episode of seizure:

  • Vascular (stroke, bleeding, arteriovenous malformations)
  • Infections (encephalitis, meningitis, abscess)
  • Trauma (especially penetrating)
  • Autoimmune (CNS vasculitis)
  • Metabolic (hypo/hyper natremia, hypocalcaemia, hypomagnesemia, hypoglycemia, hypoxia, drug overdose/withdrawal)
  • Idiopathic (epilepsy)
  • Neoplasms
  • pSychiatric (pseudoseizures)

It is very important to consider an episode of afebrile seizure as a symptom rather than a disease. The next best step in management of a seizure for the first time invariably starts with looking for an underlying cause after the patient is stabilized. Once no precipitating factor could be found for the seizure, the condition is termed as epilepsy.

The minimum investigations for a first-episode afebrile seizure include the following:

  • Serum calcium, magnesium, and the basic metabolic panel (sodium, potassium, blood sugar, chloride, bicarbonate, urea and creatinine)
  • Fasting blood glucose
  • ABG – to exclude hypoxia as a cause of the seizure
  • Toxicology screen
  • An MRI of the brain (first-choice) or CT scan

According to an article recently published in Medical Journal of Australia (MJA), vitamin deficiency is prevalent among adult Aboriginal Australian, particularly within autumn, spring and winter. The article, however, is not clear about the prevalence of vitamin D deficiency among Aboriginal children.

Other populations susceptible to vitamin D deficiency are those who live in refugee camps, people with limited sun exposure (residents in areas near Antarctica or North Pole or dark-skinned people).

Vitamin D insufficiency, leads to decreased absorption of dietary calcium and hypocalcemia. Hypocalcemia has several manifestations including seizure and, as mentioned earlier, serum calcium level should always be checked in the very first workup of any first-episode seizure.

The question does not ask about initial investigations. Measuring vitamin D level is not among initial assessments of a first-time seizure, but serum calcium level is. Once hypocalcemia is established, further investigation to determine the etiology of the hypocalcemia will include measurement of serum vitamin D level.

However, for the purpose of this question, if vitamin D was measured initially, a decreased level is likely to be present as the most likely cause of the seizure.

OPTION B : EEG is often considered if no secondary cause for the seizure is found (not among the very first initial steps).

OPTION C : A CT scan or preferably MRI of the brain, although should be considered as part of the investigation are not very likely to unveil an underlying brain pathology such as brain tumor or abscess, because there are no pointers towards space occupying lesion in this child on exam.

OPTION D : The boy is afebrile; therefore, studies such as chest X-ray or urinalysis, that are recommended as a part of work for identifying the source of the fever in febrile seizures are not indicated here.

OPTION E : Toxicology screen would be of great value in older patients such as adolescents or adults in whom intentional drug use is more common. A 3-year-old boy is unlikely to overdose on drugs deliberately.

  • MJA - Vitamin D insufficiency in Aboriginal Australians

79

Q

A 16-month-old child is brought to your attention by his parents because of their concerns about their child’s language delay. He was born at 36 weeks through normal vagin*l delivery with a birth weight of 2500 g. At 8 months of age, a ‘mother and child health screening test’ for hearing was performed that was completely normal. The parents say that he babbles but has not spoken any words so far. On physical examination, the child appears quite normal. Which one of the following will you recommend?

A. Reassure the parents that this is a normal variant.
B. Arrange for formal audiometry testing.
C. Reassess the child at 18 months of age.
D. Repeat the ‘mother-child health’ hearing test.
E. Check for dysmorphic features.

A

C. Reassess the child at 18 months of age

Parents are usually the first to be suspicious of their child having a hearing problem and they are almost always right. Concerns of parents about possible developmental delays should never be ignored and have to be addressed promptly. In this child, the parents are concerned about language delay.

Language delay is defined as either of the following:

  • Late to talk and first words do not appear by the age of 15-18 months.
  • The child gets their first words but then does not go on to develop new words quickly.
  • By two years of age, the child is saying less than 50 words and is not using any two word combinations (e.g. ‘more drink’, ‘Daddy gone’, ‘car go’).
  • The child has difficulty understanding what is being said to them and has difficulties following instructions.
  • The child’s language sounds immature for their age.
  • Difficulties attending at group time at kindergarten or school.
  • The very young child may have difficulties with eye contact, attending to activities and to speech and using sounds and gestures.
  • Difficulty answering questions.
  • Difficulty sequencing words together in sentences.
  • Difficulty reading and writing.
  • Difficulty getting their message across.
  • Uses incorrect grammar (e.g., ‘me want that red one’ instead of ‘I want the red one’).

By the age of 2 years a child should be able to:
1. Say first name
2. Say many words (mostly naming words)
3. Begin to use one- to two-word sentences such as ‘want milk’

Not saying even a word by the age of 15-18 months is definitely an indicator of language delay for which investigation should follow. This child is 16 months now; therefore, it is best to wait until 18 months of age to see if he can speak his first word. Sine hearing problems are the most common cause of language delay, a formal audiometry assessment (option B) would be the most appropriate option to consider if he fails to speak his first word by the age of 18 months.

OPTION A : Parents cannot be reassured unless their child can speak his first word by the age of 18 months.

OPTION D : As the name implies, UNNHS test is a screening test. Screening tests are to find asymptomatic cases. Since this child seems to have hearing problems (symptomatic) repeating a test would be useless and formal audiological assessment is needed as the most appropriate step in management. Several techniques are available for children who are still so young to engage in tests where speaking is necessary.

OPTION E : The hearing deficit as a possible cause of language delay may or may not be associated with dysmorphic features indicative of a congenital developmental problem or syndrome. This does not change the necessity for confirmation of hearing deficit by audiological assessments when indicated. This child still has until 18 months of age to say his word before language delay is even considered.

  • Kid Sence - Language Delay
  • Developmental milestones and the Early Years Learning Framework and the National Quality Standards
  • RACGP - Audiology

80

Q

A 3-year-old boy is brought to the Emergency Department with sever stridor and suprasternal notch and chest wall retraction. Based on the history and clinical findings, the diagnosis of croup is established. Which one of the following findings, if present, indicates severe respiratory distress?

A. Suprasternal notch and chest-wall retraction.
B. Use of accessory muscles.
C. Loudness of the stridor.
D. Restlessness and agitation.
E. Oxygen saturation by oxymetry.

A

D. Restlessness and agitation

Croup affects about 2% of preschool-aged children every year. Most children have mild croup and are managed at home often after review by a general practitioner. A minority of children develop moderate or severe croup. This group should be reviewed in an emergency department and may need hospital admission.

Determining the degree of airway obstruction (based primarily on the history) is the most important consideration when assessing children with croup. Since airway obstruction can deteriorate rapidly, repeated careful clinical assessment is crucial.

Based on clinical findings, airway obstruction can be classified as mild, moderate or severe:

  • Mild – A child with mild airway obstruction appears happy and is prepared to drink, eat, play and be interested in the surroundings. Most child presenting to general practitioners fall within this category. There might be mild chest-wall retraction and mild tachycardia, but there is no stridor at rest.
  • Moderate – Moderate airway obstruction is indicated by persisting stridor at rest, chest-wall retraction, use of accessory respiratory muscles and increased heart rate. The child can be consoled and remains interactive with people and the surroundings.
  • Severe – The child appears increasingly worried, tired and exhausted. There is marked persistent tachycardia unprortionate to fever. Restlessness, agitation, irrational behavior, decreased level of consciousness, hypotonia, cyanosis and marked pallor are late signs of life-threatening airway obstruction.

Of the given options, restlessness and agitation, especially if the child cannot be consoled and calmed down, indicates severe airway obstruction. Other indicators of severe airway obstruction include: confusion, lethargy, cyanosis, pallor, or confusion.

OPTION A and B : Suprasternal notch and chest wall retraction can be seen in moderate to severe or even mild airway obstruction. This is also true about use of accessory respiratory muscles; therefore, these findings cannot be reliably used to determine the severity of airway obstruction. Moreover, children with severe obstruction may have decreased chest-wall retraction and accessory respiratory muscle use as respiratory fatigue develops.

OPTION C : Loudness of the stridor cannot be used as a predictor because firstly, it could be underestimated or overestimated depending on the ambient noises, and secondly, the intensity of the stridor may decrease when respiratory fatigue and diminished air entry develops.

OPTION E : Oxymetry is not a reliable indicator because it may be close to the normal level in a child with severe croup, or, conversely, substantially decreased in a child with mild to moderate croup.

  • MJA - Croup: assessment and evidence-based management
    • RCH - Croup (Laryngotracheobronchitis)
Normal & Abnormal Growth and Development/Child Health/Paediatrics Flashcards by Lubi Mupwaya (11)

81

Q

A 6-month-old boy is brought to your office by his mother with persistence bouts of cough for the past 2 weeks. Every bout of cough is associated with cyanosis of the face and usually followed by vomiting. Which one of the following is the next step in management?

A. Nasopharyngeal aspirate and review in 24 hours.
B. Immunization at this visit with dTP and review in 24 hours.
C. Start him on clarythromycin and review in 24 hours.
D. Trial of salbutamol by mask.
E. Admission to the hospital.

A

E. Admission to the hospital

The clinical picture suggests respiratory infection with Bordetella pertussis.

Pertussis (whooping cough) is a bacterial respiratory infection caused by the organism, Bordetella pertussis (a Gram negative bacillus). It is highly infectious and is spread by respiratory droplets to 70–100% of susceptible household contacts and 50–80% of susceptible school contacts. Pertussis infection occurs worldwide, it affects all age groups, and is most serious in young, unprotected infants. It commonly occurs in teenagers and young adults who have not been immunized or with waning immunity. Patients are infectious just before, and for 21 days after, the onset of cough if left untreated.

If treated early with antibiotics, the period of infectivity usually lasts 5 days or less after the commencement of therapy. Maternal antibody does not give adequate protection against pertussis; therefore, babies can be infected before being immunized.

Pertussis presents with mild upper respiratory tract symptoms (catarrhal stage) and can progress to severe paroxysms of cough (paroxysmal stage), often with a characteristic inspiratory whoop, followed by vomiting. Fever is usually absent or minimal. Symptoms gradually resolve (convalescent stage). The paroxysmal cough and inspiratory whoop may be absent, particularly in infants less than 6 months of age and in adults. Mild illness is common, particularly in immunized individuals. The cough may persist for up to 3 months.

The indications for hospitalization/pediatric consultation include the following:

  • Infants less than 6 months of age
  • Any child who has had apnea, cyanosis, pneumonia, or encephalopathy as a complication of pertussis.

With cyanosis following the attacks of cough, this child needs to be admitted to the hospital as the most important next step in management.

OPTION A : Nasopharyngeal aspirate can be considered later, if indicated, after the child is admitted to the hospital.

OPTION B : Immunization, at a later date, should be encouraged even if the child has had pertussis, but this step is not a part of acute management plan.

OPTION C : Antibiotics should be considered if:

  • The patient is diagnosed in catarrhal or early paroxysmal phase (may reduce severity)
  • Cough for less than 14 days (may reduce spread - reduces school exclusion period)
  • Admitted to hospital
  • There are complications (pneumonia, cyanosis, apnea)

Antibiotics of choice are azithromycin or clarithromycin. They can be used after the child is admitted to the hospital.

OPTION D : Salbutamol or other bronchodilators has not been shown to be helpful in management of patients with pertussis.

  • RCH - Whooping cough (pertussis)
  • RACGP - Pertussis Presentation, investigation and management

82

Q

Paul, aged 4 months, is diagnosed with pertussis. The father is an engineer and the mother a housewife. They both have completed childhood vaccination against pertussis. Paul has a 5-year-old sister. Which one of the following is the most appropriate management plan to consider for Paul’s parents regarding pertussis?

A. Immediate booster immunization against pertussis.
B. A 10-day course of erythromycin.
C. Scheduling a 3-dose course of pertussis vaccination.
D. Nasopharyngeal swabs.
E. Immediate immunization with pertussis immunoglobulin.

A

A. Immediate booster immunization against pertussis

Current guidelines in Australia for the public health management of pertussis recommend Antibiotic treatment of the index case with exclusion from childcare, school, work, or other environment where high-risk contacts may be present, until they are non-infectious (that is, after 5 days of antibiotic treatment). Treatment must be commenced in the first 21 days of illness to be effective. It does not shorten the duration of the illness, but does limit duration of infectivity.

Chemoprophylaxis of contacts to prevent secondary transmission is not recommended in most situations because of the delayed presentation of the index case, and the cost and adverse effects of antibiotics. However, given the high risk of mortality and morbidity associated with infection of the newborn, particularly in the context of the rising incidence of pertussis in the community and the high transmission rate, chemoprophylaxis is recommended to limit transmission to those most at risk of the infection. Data to support this recommendation are limited.

Australian guidelines recommend post-exposure chemoprophylaxis for contacts to whom transmission is most likely, and when there is significant risk of morbidity or mortality or risk of transmission to other high-risk groups.

The following criteria must be met for commencement of prophylactic antibiotics:

  • Close contact with confirmed case of pertussis (index case) while index case is infectious, which is within the first 21 days of cough and < 5 days of antibiotics
    AND
  • First contact was within 14 days (or within 21 days for infants < 6 months) – beyond that time prophylaxis is not effective

If the above criteria are met, the following groups should receive prophylactic antibiotics:

  1. Children
    * Age < 6 months; OR
    * < 3 doses pertussis vaccine; OR
    * There is a household member aged < 6 months; OR
    Children who attend childcare in same room as infant < 6 months
  2. Adults (regardless of immunization status)
    * Expectant parents in last month of pregnancy; OR
    * First contact was within 14 days (or within 21 days for infants < 6 months) – beyond that time prophylaxis is not effective
    * Health care worker in maternity hospital or newborn nursery; OR Childcare worker in close contact with infants < 6 months; OR
    * Household member aged < 6 months

—-

In the given scenario, there is no other child less than 6 months in the house other than Paul who is the index case, and none of the parents are in close contacts with other children younger 6 than months, no prophylactic antibiotic is required; however, it is recommended that close contacts that are not up-to-date with their pertussis immunization should be given DTPa or dTpa as soon after exposure as possible. dTpa should be considered for adults who have not had pertussis-containing vaccine in the last 10 years.

OPTION B : Parents do not fulfill criteria to receive prophylactic antibiotics.

OPTION C : Parents have received their 3 doses in childhood and do not need to have it repeated again. All they need is a booster dose of pertussis vaccine.

OPTION D : Nasopharyngeal swab results do not affect the management of close contacts in terms of prophylaxis; therefore, not indicated.

OPTION E : Immunoglobulin (passive immunity) is not effective in reducing the chance of catching pertussis.

  • RCH - Whooping cough (pertussis)
  • CDNA National Guidelines for Public Health Units - Pertussis
  • RACGP - Pertussis Presentation, investigation and management

83

Q

An unvaccinated 3-year-old child has had severe paroxysms of cough with inspiratory whoop for the past 4 weeks. Bouts of cough have been followed by vomiting. Cough bouts occur more at night. Physical examination is normal. Laboratory investigations is significant for lymphocytosis. Which of the following is not true regarding this condition?

A. Antibiotics at this stage are not likely to reduce the communicability of the disease.
B. Antibiotics often do not improve the symptoms of the patient.
C. School exclusion for him is not necessary anymore.
D. Given the time of presentation, antibiotic prophylaxis for the close contacts is not likely to be effective.
E. Antibiotic prophylaxis for his household is required at this stage.

A

E. Antibiotic prophylaxis for his household is required at this stage

This child has signs and symptoms of whooping cough or pertussis. Pertussis is a respiratory infection caused by Bordetella pertussis. The clinical illness is divided into three stages- catarrhal, paroxysmal, and convalescent:

  • Catarrhal phase - Mild cough and coryza often lasting 1 to 2 weeks. Fever is uncommon, and if present, is usually low grade. Instead of improving, the cough gradually increases, unlike a typical upper respiratory infection.
  • Paroxysmal phase - In the paroxysmal phase, coughing persists and severity increases, occurring in paroxysmal attacks. The classic cough of pertussis is distinctive. The paroxysm is a long series of coughs, during which the child may develop gagging and cyanosis and appear to be struggling for breath. Whooping, which is characterized by the noise of the forced inspiratory effort, may be observed following a coughing attack during the paroxysmal phase. Post-tussive vomiting occurs frequently.
  • Convalescent phase - In convalescent phase, the cough begins to subside, and the child enters the convalescent stage. The cough continues to decrease gradually over several weeks to months.

Antibiotics are usually given to reduce the spread of the disease to others; however there is little evidence about improvement of the patient’s symptoms with antibiotics, unless it is given in the catarrhal phase (less than 14 days of symptoms) which is not the case most of the time. This child has had the symptoms for 4 weeks and is over the infectious period. Therefore, antibiotics at this stage are not likely to reduce the communicability (option A) of the disease; neither do they decrease the symptoms (option B).

Unimmunized (< 3 doses) household and close children contacts less than 7 years of age must be excluded from school or child care for 14 days from the last exposure to infection OR until they have taken 5 days of effective antibiotics. The index case should be excluded from school while in the infectious period or until he receives 5 days of antibiotics.This child is beyond the infectivity period is school exclusion for him is not necessary anymore (option C).

Chemoprophylaxis of contacts to prevent secondary transmission is not recommended in most situations because of the delayed presentation of the index case, and the cost and adverse effects of antibiotics. However, given the high risk of mortality and morbidity associated with infection of the newborn, particularly in the context of the rising incidence of pertussis in the community and the high transmission rate, chemoprophylaxis is recommended to limit transmission to those most at risk of the infection. Data to support this recommendation are limited.

Australian guidelines recommend post-exposure chemoprophylaxis for contacts to whom transmission is most likely, and when there is significant risk of morbidity or mortality or risk of transmission to other high-risk groups.

The following criteria must be met for commencement of prophylactic antibiotics:

  • Close contact with confirmed case of pertussis (index case) while index case is infectious, which is within the first 21 days of cough and < 5 days of antibiotics
    AND
  • First contact was within 14 days (or within 21 days for infants < 6 months) – beyond that time prophylaxis is not effective

If the above criteria are met, the following groups should receive prophylactic antibiotics:

  1. Children
    * Age < 6 months; OR
    * < 3 doses pertussis vaccine; OR
    * There is a household member aged < 6 months; OR
    Children who attend childcare in same room as infant < 6 months
  2. Adults (regardless of immunization status)
    * Expectant parents in last month of pregnancy; OR
    * First contact was within 14 days (or within 21 days for infants < 6 months) – beyond that time prophylaxis is not effective
    * Health care worker in maternity hospital or newborn nursery; OR Childcare worker in close contact with infants < 6 months; OR
    * Household member aged < 6 months

Since the above criteria are not met for the household in this scenario, no antibiotic prophylaxis for the index case’s household is required at this stage.

  • RCH - Whooping cough (pertussis)
  • CDNA National Guidelines for Public Health Units - Pertussis
  • RACGP - Pertussis Presentation, investigation and management

84

Q

A 6-month-old male infant was brought to your clinic due to characteristic cough of pertussis. Which one of the following is the best advice for the patient’s parents to reduce their risk of infection?

A. Commence a 3-dose pertussis revaccination.
B. Immediate booster immunizations against pertussis.
C. A 5-day course of azithromycin.
D. No action is required.
E. Immediate immunization with pertussis immunoglobulin.

A

B. Immediate booster immunizations against pertussis

Pertussis is a respiratory infection caused by Bordetella pertussis. The disease often has an incubation period of 7 to 10 days. In unvaccinated individuals, Bordetella pertussis is highly infectious; spreading by aerosols to 90% of susceptible household contacts.

Antibiotics, if given early during the paroxysmal and catarrhal stages, will cause amelioration of the symptoms. However, more patients do not present at this stage and come to medical attention when the been disease has established. At this point, the role of antibiotics is limited to preventing the disease from more spread to the patient’s contacts. It should be remembered that patients are rarely infectious after 3 weeks.

PatiePatientsld be advised to avoid contact with other people, until 5 days after antibiotics are started. The antibiotics used in pertussis for prevention of the disease spread for index case are:

  • (First –line) Azithromycin 500 mg (child 6 months or older: 10 mg/kg up to 500 mg) orally, for the first dose, then 250 mg (child 6 months or older: 5 mg/kg up to 250 mg) orally, daily for a further 4 days (neonate and child younger than 6 months: 10 mg/kg orally, daily for 5 days)
    OR
  • (First-line) Trimethoprim + sulfamethoxazole 160+800 mg (child 1 month or older: 4+20 mg/kg up to 160+800 mg) orally, 12-hourly for 7 days
    OR
  • (Second-line) Clarithromycin 500 mg (child: 7.5 mg/kg up to 500 mg) orally, 12-hourly for 7 days

NOTE - There is no clinical evidence to support roxithromycin for the treatment of pertussis

Regarding prophylactic measures, the following guidelines are currently in place:

Prophylaxis with antibiotics for household and close contacts:

Chemoprophylaxis to prevent secondary transmission is not recommended in most situations because of the delayed presentation of the index case and the cost and adverse effects of antibiotics. However, given the high risk of mortality and morbidity associated with infection of the newborn, particularly in the context of the rising incidence of pertussis in the community and the high transmission rate, chemoprophylaxis with antibiotics is recommended to limit transmission to those most at risk of the infection (young infants). Data to support this recommendation are limited.

Australian guidelines recommend post-exposure chemoprophylaxis with antibiotics for contacts to whom the transmission is most likely, and when there is a significant risk of morbidity or mortality or risk of transmission to other high-risk groups. These groups include:

  • all household contacts of an index case when the household includes children less than two years who have received less than three doses of vaccine (including newborn infants)
  • any woman in the last month of pregnancy
  • all adults and children in a childcare arrangement with an index case, if the group contains children less than two years who have received less than three doses of vaccine
  • healthcare workers in maternity and neonatal units
  • infants in maternity and neonatal units where a healthcare worker was the infected case.

Therapy must be started within 21 days of exposure to the index case to be effective.

Vaccination
Australian guidelines recommend a single dose of an acellular vaccine for contacts of pertussis older than eight years, and catch-up vaccination for unvaccinated or partially vaccinated (incomplete infant vaccination) contacts up to their eighth birthday.

This child’s parents do not require prophylaxis with antibiotics because they do not fulfill the mentioned criteria; however, based on current guidelines, they should receive a booster dose of the pertussis vaccine. Passive immunization with pertussis immunoglobulin is not effective in preventing disease spread. A full vaccination with 3 doses of pertussis-containing vaccines as a catch-up program is recommended for children up to 8 years old.

  • RCH - Whooping cough (pertussis)
  • CDNA National Guidelines for Public Health Units - Pertussis
  • RACGP - Pertussis Presentation, investigation and management
  • Australian Prescriber - Pertussis prophylaxis

85

Q

An 8-year-old boy is found to have persistent proteinuria of 2+ on dipstick urine. There is no hematuria. Physical examination including blood pressure is unremarkable. Which one of the following is the next best investigation to consider for this boy?

A. Renal biopsy.
B. 12-hour urine protein.
C. Urine examination.
D. C3 and C4 level measurement.
E. Ultrasonography.

A

C. Urine examination.

Proteinuria is defined as presence of 1+ protein or more on urine dipstick exam. To confirm the diagnosis of proteinuria, two consecutive urine dipsticks or urinalysis, 1 to 2 days apart must indicate proteinuria.

If proteinuria persists on dipstick, the next best will be a complete urine exam (urinalysis) on a midstream urine sample obtained in the office for a more accurate assessment of the urine. With urinalysis (urine examination), the urine sediment, as well as other signs of glomerular and/or parenchymal disease such as hematuria, red cell casts, pyuria and/or lipiduria are assessed. Particular findings on urine examination may suggest glomerulonephritis, nephrotic syndrome, or other renal parenchymal disorders.

Urinalysis may also suggest urinary tract infection with pyuria, bacteriuria, and positive nitrites or leukocyte esterase along with mild proteinuria.

Other initial laboratory evaluation includes renal function tests (blood urea nitrogen and creatinine), serum electrolytes, cholesterol, albumin, and total protein.

Another very important investigation to consider for this child is urine protein/creatinine ratio on a first morning void obtained at home as an estimation of the amount of protein excreted in 24 hours.

(Option A) The role of renal biopsy in children with asymptomatic persistent proteinuria is controversial and is not recommended routinely.

(Option B) If indicated, urine protein measurement is performed on a 24-hour urine collection (not 12 hours). This test is cumbersome and not necessary unless at times if the patient is found to have an abnormal urine protein/creatinine ratio.

(Options D and E) Other tests such as renal ultrasound, serum complement levels (C3 and C4), antinuclear antibody (ANA), streptozyme testing, hepatitis B and C serology, and human immunodeficiency virus (HIV) testing should be considered based on the clinical setting. A voiding cystourethrogram should be considered if there is an abnormal ultrasound with scarring or a history of febrile urinary tract infections.

86

Q

A 3-year-old child is brought to your office by his mother with complaints of diarrhea and colicky abdominal pains for the past three days. Which one of the following can be the most likely diagnosis?

A. Giardiasis.
B. Norovirus gastroenteritis.
C. Perforated appendicitis.
D. Staphylococcal gastroenteritis.
E. Shigellosis.

A

B. Norovirus gastroenteritis.

Of the options, gastroenteritis caused by Norovirus is the most likely diagnosis. Norovirus is a major human pathogen across the world. It is estimated to account for 95% of non-bacterial gastroenteritis in outbreaks in industrialized countries such as Australia and 50% of all gastroenteritis outbreak worldwide. All age groups are susceptible.

The most common route of transmission is via fecal-oral route. Consumption of contaminated food such as salads, ice, water, fresh fruit, bakery goods, and cold meats is a major cause of infection. In a minority of cases transmission occurs via aerosol/personal contact or fomites from toys, carpet, handles, surfaces etc.

The incubation period after digestion is 24-48 hours but it can occur as early as 12 hours.

Clinical manifestation includes:
Common

  • Nausea
  • Vomiting
  • Diarrhea
  • Abdominal cramps

Less common

  • Headache
  • Low-grade fever
  • Chills
  • Muscle aches
  • Malaise

Diarrhea of Norovirus gastroenteritis is non-inflammatory and is not bloody or mucoid. Except in outbreaks, no diagnostic test is necessary. PCR of the stool is the method of choice. Treatment is only symptomatic.

(Option A) Giardiasis should be thought of in hikers, campers, or those with history of drinking fresh water.
Symptoms of acute giardiasis include:

  • Diarrhea (sudden in onset; initially may be watery) - 90%
  • Malaise - 85%
  • Foul-smelling and fatty stools (steatorrhea) - 70%
  • Abdominal cramps and bloating - 70%
  • Flatulence - 75%
  • Nausea - 70%
  • Weight loss – 65%
  • Vomiting – 30%
  • Fever -10-15%

Symptoms usually develop after an incubation period of 7 to 14 days (range 1 to 45 days) and may last 2-4 weeks.

Significant weight loss can occur in about 50% of symptomatic patients.

With diarrhea and abdominal cramps, giardiasis can be a possibility; however, no history of using fresh water makes this diagnosis less likely.

(Option C) Perforated appendicitis presents with a completely different clinical picture including systemic toxicity and sign and symptoms of peritonitis.

(Option D) Staphylococcal gastroenteritis is a self-limiting condition caused by contaminated foods. Nausea and vomiting are more pronounced, and diarrhea is less common.

(Option E) Shigellosis is an infection of the lower gastrointestinal tract. Patients with Shigella gastroenteritis typically present with high fever, abdominal cramps, and bloody and mucoid diarrhea. The incubation period ranges from 1 to 7 days. The disease typically begins with constitutional symptoms such as fever, anorexia, and malaise. Initially, diarrhea is watery, but subsequently may contain blood and mucus. Tenesmus is a common complaint.

87

Q

A mother has brought her four-year-old son with complaint of nighttime bed wetting. According to her, he wets his bed 3-4 time a week at night despite achievement of daytime continence at the age of two years. He is otherwise asymptomatic. Physical examination is unremarkable. Which one of the following is the most appropriate management regarding this complaint?

A. Bed wetting alarms.
B. Nasal desmopressin.
C. Amitriptyline.
D. Waiting for spontaneous resolution.
E. Ultrasonography of the bladder, ureters and kidneys.

A

D. Waiting for spontaneous resolution.

According to the International Children’s Continence Society (ICCS), enuresis, also termed intermittent nocturnal incontinence, refers to discrete episodes of urinary incontinence during sleep in children of five years of age or older.

Monosymptomatic nocturnal enuresis refers to children with normal daytime voiding patterns and night time wetting only. Non-monosymptomatic enuresis refers to enuresis in children with daytime wetting and / or additional lower urinary tract symptoms (such as abnormal urine stream, hesitancy, urgency, dribbling or pain).

This child is only 4 years old, and does not fulfill the criteria for enuresis. He is very likely to grow out of the problem until 5 years age; therefore, no further action is required except waiting for spontaneous resolution. Approximately, 84% of children achieve full night-time bladder control by the age of 5 years.

(Option A) Bed wetting (enuresis) alarms are the most effective means of controlling nocturnal enuresis and preventing relapse. An arousal device, which is often an auditory alarm and/or a vibrating belt or pager, activates when a sensor placed in the underwear or on a bed pad, detects moisture. The alarms work through conditioning: the child learns to wake or inhibit bladder contraction in response to the physiologic conditions present before wetting.

Enuresis alarms are a first-line treatment for children whose bedwetting has not responded to advice about fluid intake, toileting, or an appropriate reward system. Enuresis alarms work best for well-motivated families and children with frequent enuresis (more than twice per week). Other treatment options should be used if:

  • Rapid or short-term improvement is the goal
  • The child or parents do not want to try the enuresis alarm
  • The child wets the bed only once or twice per week
  • The parents are having emotional difficulty coping with the burden of bedwetting
  • The parents are expressing anger, negativity, or blame toward the child

Bedwetting of this 4-year-old child is normal and does not require investigation or treatment for now.

(Option B) Desmopressin, a synthetic vasopressin analog, is the first-line treatment for enuresis in children older than 5 years, whose bedwetting has not responded to advice about fluid intake, toileting, or an appropriate reward system. It is an alternative to enuresis alarms for children and families who seek rapid or short-term improvement of enuresis, or have failed, refused, or are unlikely to adhere to enuresis alarm treatment; and for whom an enuresis alarm is unsuitable.

(Option C) Tricyclic antidepressants (TCAs) decrease the amount of time spent in REM sleep, stimulate vasopressin secretion, and relax the detrusor muscle. Given the efficacy and safety of enuresis alarms and desmopressin, tricyclic antidepressants (e.g., amitriptyline, despiramine, imipramine) are used as third-line treatment for monosymptomatic enuresis after failed alarm therapy and/or desmopressin).

(Option E) Ultrasonography of urinary system and urine examination are initial investigation to consider in children with enuresis. This child is only 4 years old and does not fulfill criteria for enuresis; therefore, no evaluation is indicated for him at this stage.

88

Q

A 6-yer-old girl is brought to your general practice by her mother, who is concerned about a neck mass in her daughter. The mass is shown in the following photograph. The child is otherwise healthy. On examination, there is a non-tender neck mass in the midline moving with swallowing and tongue protrusion. The rest of the physical examination is unremarkable. Which one of the following could be the most likely complication of this neck mass?

A. Infection.
B. Malignancy.
C. Fistula formation.
D. Overgrowth and compression of underlying structures.
E. Rupture.

Normal & Abnormal Growth and Development/Child Health/Paediatrics Flashcards by Lubi Mupwaya (12)

A

A. Infection.

The clinical presentation and the photograph are highly suggestive of thyroglossal duct cyst (TDC) as the most likely diagnosis. TDC is the most common type of developmental cyst encountered in the neck region. TDC occurs when the thyroglossal duct fails to obliterate and forms a bridge between the base of the tongue and the thyroid gland.

A TDC usually presents with an asymptomatic palpable midline neck mass usually at or below the level of the hyoid bone, above the thyroid cartilage. The mass is most often in the midline, although it can present slightly off the midline to one side or the other. A TDC may present in childhood (less than 50%) or later in life, usually as a young adult below the age of 20 years. Characteristically on examination, a TDC moves up when the tongue is protruded, and moves up and down upon swallowing reflecting the attachment of these cysts to the base of the tongue by the thyroglossal tract. Some patients have neck or throat pain, or dysphagia, but most patients are just concerned by the lump itself.

A TDC may become complicated. Complications of TDCs include:

  • Infection
  • Malignancy.
  • Overgrowth and pressure of the underlying structures.
  • Rupture and fistula formation.

Of these, infection is the most common complication. An infected TDC usually presents with redness and swelling (image below).

Malignancy (option B) is the second most common complication of TDCs. It can complicate the cyst in approximately 1% of patients. Papillary carcinoma is the most common malignancy arising from a TDC. Follicular papillary and Squamous cell carcinoma follow in order of commonality.

(Option D) Cyst enlargement and compression of the underlying structures is another complication of TDCs. Obstruction of the airway is of significance importance. However, compared to infection and malignancy, this complication is less common.

Rupture (option E) and fistula formation (option C) are other rare complications of a Thyroglossal duct cyst.

Normal & Abnormal Growth and Development/Child Health/Paediatrics Flashcards by Lubi Mupwaya (13)

89

Q

Concerned mother of a 6-year-old girl has brought her to your attention because she has frequently noticed blood-stained yellow discharge on her panties. She is otherwise healthy. There is no complaint of itching or urinary symptoms. The girl lives with her mother and birth father. On examination, the vitals are within normal ranges. On
inspection, there is mild erythema of the vulva and perianal area and offensive blood-stained yellow discharge in the introitus. Which one of the following could be the most likely diagnosis?

A. Monilial vulvovaginitis.
B. vagin*l foreign body.
C. Atrophic vulvovaginitis.
D. Chlamydia infection.
E. Threadworms.

A

B. vagin*l foreign body.

Blood-stained vagin*l discharge in female children should always be taken seriously and investigated promptly. One exception is vagin*l bleeding in the first week of birth in female neonates that is caused by withdrawal from maternal estrogen upon birth.

Causes of vagin*l bleeding in children include the following:

  • vagin*l foreign body
  • Severe vulvovaginitis
  • Trauma (including straddle injury and sexual abuse)
  • Excoriation associated with threadworms
  • Onset of first menstruation
  • Hematuria
  • Urethral prolapse (an inflamed “doughnut” of tissue is visible at the urethral meatus

Of the above, a vagin*l foreign body is the most common cause of bloody vagin*l discharge. The foreign body is often toilet papers. Foreign body in the vagin* causes offensive purulent vagin*l discharge that recurs frequently despite successful initial management with antibiotic unless the foreign body is removed. It can also cause vagin* bleeding. Children with suspected vagin*l foreign body should be referred to pediatrics specialist for removal of the foreign body.

(Option A) Monilial (candida) vulvovaginitis is almost never seen in prepubertal girls except as an association with nappy rash. It cannot be the cause of such presentation in a 6-year-old girl.

(Option C) Atrophic vulvovaginitis is a less common condition in prepubertal girls and if present should raise suspicion against lichen sclerosus. Even in case of atrophic vaginitis, itching and mucoid discharge would be the expected presentation. With a blood-stained yellow discharge, atrophic vaginitis is very unlikely.

(Option D) Infection with chlamydia, gonorrhea and trichom*onas can cause offensive bloody vagin*l discharge. These infections in children are highly suggestive of sexual abuse. For every child with vagin*l discharge associated with unusual features such as persistent and significant discharge or blood in the discharge, sexual abuse should be considered as a possibility and approached appropriately. Although sexual abuse should also be considered and thought of in such situation, the most likely cause of such presentation remains a vagin*l foreign body, unless
investigations establish the presence of chlamydia, gonorrhea or trichom*onas, in which case sexual abuse is almost always the cause.

(Option E) Infection with threadworm may vulvovaginitis, in which case, itching is the most prominent symptom. Excoriation can cause bleeding; however, absence of itching makes this diagnosis almost unlikely.

90

Q

Parents of a 12-year-old girl has brought her to a clinic after they noticed a rash on her back. She had sore throat 3 weeks ago which subsided gradually in a course of days. Last week she had a very painful left wrist pain that improved after 2 days before she develops pain in her right knee. Today, she has a painful right ankle. On physical examination, she looks ill and has a temperature of 38.1°C. The rest of her vital signs are within normal limits.

Examination of the musculoskeletal system is notable for a painful, slightly hot and swollen right ankle with decreased range of motion due to pain. There is a rash on her back which is shown in the following photograph.

Which one of the following is the most appropriate next step in management?

A. Echocardiography.
B. Blood culture.
C. ESR.
D. Throat swab.
E. Full blood examination (FBE).

Normal & Abnormal Growth and Development/Child Health/Paediatrics Flashcards by Lubi Mupwaya (14)

A

D. Throat swab.

The photograph shows well demarcated bright red and pink macules characteristic of erythema marginatum. The presence of the rash, migratory arthralgia/arthritis (involvement of one joint, improvement and then involvement of another joint), and more importantly, a history of sore throat 3 weeks before such presentation is highly suggestive of acute rheumatic fever as the most likely diagnosis.

Acute rheumatic fever (ARF) occurs predominantly in childhood and is secondary to infection with Group A streptococcus (GAS). It is an acute illness presenting with a cluster of signs and symptoms including carditis and polyarthralgia. Without preventive treatment, ARF may progress to chronic rheumatic heart disease (RHD) associated with damaged heart valves.

An accurate diagnosis of ARF is important because overdiagnosis results in unnecessary treatment over a long time, while underdiagnosis leads to further attacks of ARF, cardiac damage and premature death. Diagnosis is mostly clinical because there is no specific laboratory test.

The diagnosis of ARF is usually guided by the Jones criteria and the more recent World Health Organization (WHO) criteria. To increase sensitivity for ARF diagnosis in Australia’s unique high-risk population, the Jones and WHO criteria have been further modified to form the 2012 Australian criteria for the diagnosis. According to Australian guidelines, aseptic monoarthritis or polyarthralgia is a major criteria for populations at risk, including Aboriginal and Torres Strait Islanders communities living in rural or remote areas or in disadvantaged suburban areas, it is a minor criteria (in agreement with international guidelines) for other not-at-risk populations.

The following table summarizes the Australian ARF diagnostic criteria for high risk population and other populations: (See below)

To make a diagnosis of recurrent attack of ARF (previous ARF or established rheumatic heart disease [RHD]) in past medical history) there should be
Two major criteria OR one major and two minor criteria OR three minor criteria
PLUS
Evidence of preceding GAS infection

When signs and symptoms suggest ARF (based on the criteria), the most important step is to look for evidence of preceding group A streptococcus infection. This child fulfills clinical criteria for ARF; however, it should be determined that the cause of her sore throat has been GAS before a definite diagnosis of GAS is made. Of the options, a throat swab for microscopy and culture is the most important next step in management. Cultures for GAS are the gold standard; however, any of the following can be uses instead:

  • A positive rapid GAS carbohydrate antigen test in a child whose clinical presentation suggests a high pretest probability of streptococcal pharyngitis
  • Elevated or rising streptococcal antibody titer (a rise in titer is better evidence than a single titer result)
  • Antistreptococcal antibodies include the following:
  • Antistreptolysin O (ASO)
  • Antideoxyribonuclease B (anti-DNAse B)
  • Antistreptokinase
  • Antihyaluronidase
  • Anti-DNAse (anti-DNPase)

(Option A) Echocardiography with Doppler should be performed in all cases of confirmed or suspected ARF.
Echocardiography may show abnormalities if there is heart involvement. However, abnormal echocardiographic findings will not direct the management plan unless there is evidence of a preceding GAS and a definite diagnosis of ARF.

(Option B) Blood culture was indicated if the source of GAS infection is unknown. With the sore throat in history, which is possibly the source of the infection, cultures from throat swab (or other methods mentioned above) can provide sufficient clue regarding preceding GAS infection, and blood cultures are not necessary.

(Option C) an ESR≥30mm or CRP≥30mg/L is a minor criteria for ARF. This child already has adequate clinical criteria for ARF, and a positive ESR or CRP adds no diagnostic benefit at this stage. ESR or CRP could have been an option if further clinical or laboratory criteria were required to fulfill the diagnostic criteria.

(Option E) Although FBE is often routinely evaluated as a part of basic laboratory tests, it will not add any diagnostic benefit in this case. FBE in patients with ARF is often normal or inconclusive.

Normal & Abnormal Growth and Development/Child Health/Paediatrics Flashcards by Lubi Mupwaya (15)

91

Q

For which one of the following children with urinary tract infection (UTI) hospital admission is indicated?

A. An infant younger than 12 months of age.
B. A 6-year old male child with family history of urinary tract infection.
C. A 5-year old female child with family history of urinary tract infection.
D. An infant younger than 2 months of age.
E. A 2-year old child with allergy to penicillin.

A

D. An infant younger than 2 months of age.

According to the Royal Children Hospital (RCH) guidelines, the following children with UTI should be admitted to the hospital for intravenous antibiotics and close monitoring:

  • Most of children less than 3 months.
  • Children who are seriously unwell such as those with toxic appearance or dehydration.
  • Oral antibiotics cannot be tolerated.

NOTE - The cut-off age for hospitalization can be different according to other guidelines. For example, Medscape recommends admission of all children who have febrile UTI and are younger than 2 months of age.

92

Q

Which one of the following statements is correct about Perthes disease?

A. Hip mobility is reduced especially abduction and external rotation.
B. Ultrasound is the modality of choice to make a diagnosis.
C. Progress is evaluated with serial radiological examination.
D. Osteotomy is the treatment of choice.
E. The inflammatory markers such as WBC, ESR and CRP are usually raised.

A

C. Progress is evaluated with serial radiological examination.

Legg-Calvé-Perthes disease, also called Perthes disease is avascular necrosis (AVN) of the proximal femoral head resulting from compromise of blood supply to this area. Perthes disease usually occurs in children aged 2-12 years with a peak incidence in children between 4 and 8 years. It is more common in boys than girls. The disease has an insidious onset and may follow an injury to the hip. Perthes disease is unilateral in most cases (80% -90%), and in only 10%-20% of cases both hips are involved. Even in bilateral cases, the hip joints are involved successively, not simultaneously.

Affected children often have delayed bone age, disproportionate growth, and a mildly shortened stature (smaller than age). Perthes disease can be idiopathic, or it may result from a slipped capital femoral epiphysis, trauma, steroid use, sickle-cell crisis, toxic synovitis, or congenital dislocation of the hip.

The earliest sign of Perthes disease is an intermittent limp, especially after exertion, with mild or intermittent pain in the anterior part of the thigh.

NOTE- Perthes disease is the most common cause of a limp in the 4- to 10-year-old age group.

The classic presentation of Perthes disease has been described as a ‘painless limp’. The child may present with limited range of motion of the affected extremity. The most common symptom is persistent pain. Hip pain may develop and is a result of necrosis of the involved bone. This pain may be referred to the medial aspect of the ipsilateral knee or to the lateral thigh. The quadriceps muscles and adjacent thigh soft tissues may atrophy, and the hip may develop adduction flexion contracture. The patient may have an antalgic gait with limited hip motion, or a Trendelenburg gate (abductor lurch). Pain may be present with passive range of motion and limited hip movement, especially internal rotation and abduction.

Laboratory studies including a full blood count, ESR and CRP are usually normal and are often ordered to exclude other possible causes of hip pain and limp such as septic arthritis, transient synovitis, or juvenile rheumatoid arthritis.

In most cases X-rays are normal early in the course of the disease but are used serially to assess the progression of the disease. The following are radiographic stages of Perthes disease in time:
Stage 1 - Cessation of femoral epiphyseal growth
Stage 2 - Subchondral fracture
Stage 3 - Resorption
Stage 4 - Reossification
Stage 5 - Healed or residual stage

These stages can take 12-40 months. A bone scan can be used to evaluate the site for avascular necrosis (AVN).
Early radiographic findings in acute phase can be a nonspecific effusion of the joint evident by slight widening of the joint space. Other early findings may include decreased bone density around the joint and a bulging joint capsule.

The acute phase often lasts 1-2 weeks. Decreasing bone density in and around the joint is noted after a few weeks.

Once Perthes disease is suspected, the child should be urgently referred to orthopedic specialist for management and treatment. In the meanwhile, limited movement and reducing the weight from the hip joint should be encouraged to prevent or minimize further damage of the femoral head. For this purpose, crutches are prescribed and encouraged to use.

Treatment goals in Perthes disease are:

  • Eliminating hip irritability
  • Restoring and maintaining good range of motion in the hip
  • Preventing femoral epiphyseal collapse
  • Having a spherical femoral head when the hip heals

Initial therapy includes protecting the hip joint and making it non-weightbearing. This can be achieved by maintaining the femur abducted and internally rotated to keep it held well inside the rounded portion of the acetabulum. Most cases are treated conservatively using splinting (e.g. Scottish Rite brace). At occasions surgery (osteotomy) may be considered. Surgery does not speed femoral head healing, rather it results in a more spherical oreossification of the femoral head. Limb shortening is a potential complication of surgical treatment.

(See photo) Scottish Rite Brace

(Option A) Although Perthes disease is associated with hip pain and decreased range of motion, the two most severely affected movements are abduction and internal rotation, not external rotation.

(Option B) Perthes disease is mostly diagnosed clinically. The imaging modality of choice to help with diagnosis and also assessment of the disease progression is X-ray, not ultrasound.

(Option D) Osteotomy can be a treatment option depending on the child’s age and the disease severity; however, more conservative strategies such as bracing are applied as the treatment option of choice where feasible.

(Option E) Perthes disease is NOT an inflammatory condition; therefore, inflammatory markers such as WBC, ESR and CRP are usually negative. Positivity of such markers points toward an alternative diagnosis such as septic arthritis, rheumatoid arthritis, synovitis, etc.

Normal & Abnormal Growth and Development/Child Health/Paediatrics Flashcards by Lubi Mupwaya (16)

93

Q

The mother of a four-year-old female child has brought her to you after she noticed a mass in her neck while giving her a bath. The mas is 2x1.5 cm in size, located in the anterior triangle of the neck, and not tender. The child is otherwise healthy and has no other concerning feature on physical examination. Which one of the following would be the most appropriate next step in management?

A. Excision of the mass.
B. Fine needle aspiration cytology (FNAC).
C. Reassurance.
D. Ultrasonography of the neck.
E. Chest X-ray.

A

D. Ultrasonography of the neck.

Childhood cervical masses are a common presentation in general practice. Often, such masses cause significant parental anxiety and also a potential diagnostic dilemma. While these masses are commonly benign, there are other possibilities that should be taken into account. A good knowledge of relevant anatomy and a thorough and systematic history, examination, and investigations are crucial to ensure an appropriate and efficient approach.
Cervical masses in children are divided in the following broad categories:

  1. Congenital
  2. Inflammatory/ infective
  3. Neoplastic

While the majority of childhood neck lumps seen in general practice occur as a result of an inflammatory/infective process, other possibilities such as persistent congenital abnormalities and red flag of neoplastic diseases always must be considered.

In history, three main components should always be covered:
1. Timeline (since when and for how long has the mass been present?
2. Time course (how fast is the mass growing/has grown?)
3. Associated features/ red flags.

Cervical masses in the neonatal period and early infancy are often congenital. Examples are thyroglossal duct cyst, teratomas, sternocleidomastoid tumors of infancy and vascular or lymphatic malformations. Of note, vascular and lymphatic malformations typically grow with the growth of the child. Reactive lymphadenopathy is most common in infancy and early childhood, with 40–55% of young children found to have palpable cervical lymph nodes.

Congenital masses may present later in childhood/adolescence because of continuous growth or superimposed infection, while the likelihood of malignancy also rises in this age group.

Rapidly developed masses are typically inflammatory, including reactive lymphadenopathy, lymphadenitis, or secondary infection of underlying congenital or neoplastic masses. Inflammatory pathology typically resolves within four weeks. Cervical masses that persist past six weeks require further evaluation.

Rapidly growing masses should be immediately referred if there is a concern that they have the potential to affect the airway or have features suggestive of abscess formation. Masses that grow at a slower rate, for months to years, are suggestive of benign neoplasms or a slowly enlarging congenital malformation.

In terms of associated symptoms, a viral prodrome, fevers, and cervical tenderness are associated features suggestive of reactive lymphadenopathy. It is, however, important to consider suppurative lymphadenitis or infection of an underlying congenital or neoplastic mass, as this may present similarly.

On physical examination, location, size, consistency, and tenderness give important clues. Location-wise, midline masses are more likely congenital and are typically thyroglossal duct cysts or dermoid cysts. Thyroglossal duct cysts will elevate with tongue protrusion or swallowing, while dermoid cysts are tethered to the overlying skin.

Thyroid masses are potentially malignant and need further evaluation. Lymphadenopathy commonly arises as a lateral lump in the anterior or posterior triangle, and may present an inflammatory or neoplastic process.

Lymphadenopathy in the posterior triangle has a higher risk of malignancy, while supraclavicular lymphadenopathy is considered a red flag. Other masses of the lateral neck include lymphatic and vascular malformations and branchial cleft cysts.

Reactive lymphadenitis is characterized by a local collection of small, tender, mobile lumps. The possibility of suppurative lymphadenitis should be considered if there is palpable warmth, fluctuation, induration, or sever tenderness. If the mass is firm, irregular, or immobile, malignancy should be thought with high suspicion.

In terms of size, a palpable cervical lymph node less than 1 cm in size can be considered normal in children, while increasing size is associated with a significantly increased risk of malignancy. Lymph nodes greater than 1 cm in size that persist for longer than six weeks or despite antibiotic therapy should be evaluated with medical imaging and a possible tissue biopsy e.g., FNAC.

The following features are red flags for a neck lump in children, the presence of which warrants a more
aggressive approach:

  • Weight loss
  • Sustained fever
  • Night sweats
  • Generalized lymphadenopathy
  • Signs and symptoms of pancytopenia
  • Mass persisting> 6 weeks
  • Lymph node> 3 cm
  • Thyroid mass
  • Supraclavicular mass
  • Hard, irregular mass
  • Fixed mass

—-

This child has presented with a solitary neck mass that is incidentally found, and for that no exact duration can be determined. The size of the mass is over 1 cm, and there is no history of a preceding infection or any physical findings for that matter. Although there is no red flag in history or physical examination, an ultrasound scan of the lump would be the most appropriate way to go among other options.

Ultrasonography is a readily available, inexpensive and radiation-free imaging study that can provide important information about the location, size and consistency of a cervical mass. For this reason, ultrasound is the preferred initial imaging study for most children with a neck mass. Ultrasound can identify features typical of numerous congenital masses as well as benign and malignant lymphadenopathy.

FNAC (option B) or excision of the mass (option A) is considered if, based on the presence of red flags in history/physical examination or alarming features on ultrasound, malignancy is suspected. In most cases, ultrasound-guided FNAC is preferred over excisional biopsy.

(Option C) Watchful waiting when the diagnosis is in favor of benign diagnoses such as reactive lymphadenopathy is the approach of choice. This, however, does not mean that reassurance can be given, especially in this case where supporting findings for reactive lymphadenopathy such as a preceding viral illness, tenderness, etc. are lacking. If an option, watchful waiting and review could have been an acceptable option given the fact that there are no alarming features.

(Option E) Chest X-ray does not seem to provide additional diagnostic value. However, in the presence of red flags, it could be used for diagnoses such as TB or for further evaluation and/ or staging in malignancies.

TOPIC REVIEW
There is no proven algorithm for management of pediatric neck masses, and management depends on the use of sound clinical judgment and judicious surveillance for red flags.

However the following approaches are often recommended and practiced:
Watchful waiting for up to six for patients with suspected reactive lymphadenitis often evident by bilateral lymphadenopathy with no red flag features for malignancy or deep cervical abscess).

Although the use of empirical antibiotics is controversial, it is the wide accepted practice in patients thought to have suppurative lymphadenitis (e.g., lymphadenopathy with marked erythema and tenderness, asymmetric lymphadenopathy and systemic symptoms).

If symptoms have not improved with antibiotics or if the mass persists longer than four weeks, investigation with ultrasound and serological screens for atypical infections including mycobacteria is warranted. Targeted serological investigations for atypical infections may be indicated if the patient has associated risk factors or if a presumed reactive lymphadenitis has not improved with conservative management.

While there is no strong evidence for the value of a routine full blood count (FBC), it is recommended if
systemic disease is suspected or the diagnosis of infection is uncertain. An FBC and blood film may help
identify pancytopenia and atypical cells suggestive of hematological malignancy, while the differential may
suggest a viral or bacterial pathology. It is recommended that patients with suspected reactive lymphadenitis are referred to a head and neck surgeon if the patient’s condition deteriorates while on antibiotics, the mass persists longer than six weeks of observation or a collection requiring incision and drainage develops.

Timely referral to a head and neck surgeon for consideration of surgical management is advised for all suspected congenital neck masses. Medical imaging (typically ultrasound) should be considered prior to referral for determining the urgency of the referral process. Secondary infection of a congenital mass may require more urgent specialist review when the patient does not respond to oral antibiotics.

All neck masses suspicious for malignancy are best referred urgently to a head and neck surgeon for further evaluation including possible biopsy. Medical imaging, including ultrasound (most commonly), CT or MRI, may be considered as part of the referral.

94

Q

A 20-month-old male infant is brought to the Emergency Department, by his parents, with complaints of cough and shortness of breath. On examination, there is bilateral wheezing over lung fields on auscultation. Which one of the following could be the most likely diagnosis?

A. Bronchiolitis.
B. Croup.
C. Epiglottitis.
D. Asthma.
E. Foreign body aspiration.

A

D. Asthma.

Wheeze is a common presentation in young children. About 20% of infants have wheeze in infancy, and at least 40% of children < 6 years of age have at least one wheezing episode.

According to Australian pediatric guidelines, the most common cause for cough and wheeze in toddlers are foreign body aspiration and viral infections. While the wheeze caused by foreign body aspiration tends to be local and usually unilateral, viral infections manifest with a more widespread and bilateral wheeze. According to the algorithm by the Royal college of Australian general practice (RACGP), if there is no symptom of coryza, the first diagnosis in that age group would be foreign body, especially if the wheeze is localized or unilateral.

Although no coryza symptoms are mentioned in the scenario, bilaterality of the wheeze makes foreign body aspiration (option E) less likely, and either asthma or a viral infection should be considered as the most likely possibilities.

Bronchiolitis (option A) matches the scenario at first glance; however, the condition is more common within the first 12 months of life and is rarely seen beyond this age. This child is 20 months old and unlikely to have bronchiolitis. Of the options, only asthma could be an acceptable explanation for this scenario.

NOTE – Asthma is very rare in children younger than 12 months of age and not very common in children between 1 and 2 years age. In fact, according to RACGP guidelines, for wheezing between ages 2 and 5 years terms such as viral wheezing, pre-school wheeze, episodic viral wheeze and multiple trigger wheeze are preferred to a definite diagnosis of asthma. In this question, however, the only option that matches the scenario is asthma.

Croup (option B) is a common childhood disease presenting with sudden onset of a distinctive barking cough that is usually accompanied by stridor, hoarse voice, and respiratory distress resulting from upper airway obstruction. Since there is no lower respiratory tract involvement, wheeze is not a feature.

Epiglottitis (option C) is caused by infection (mostly bacterial) of the epiglottis. The condition presents with painful swallowing, hyperextended neck, drooling, and signs of upper respiratory tract obstruction such as difficulty breathing and stridor. The child is often toxic and miserable. Cough and wheezing in particular are not features of epiglottitis.

95

Q

Kim, 3 years old, is brought to the Emergency Department by her parents reporting that their daughter is not feeling well at all. According to them, she developed a fever this morning and started to refuse food and crying and throwing it out when she was fed. A short while later she became miserable and inconsolable, started drooling, and her breath became noisy. On examination, she has a pulse rate of 140 bpm, respiratory rate of 34 breaths per minute with expiratory stridor, and a fever of 39.1°C. She maintains her neck extended. Lungs are clear on auscultation. Which one of the following could be the most likely diagnosis?

A. Croup.
B. Acute bronchiolitis.
C. Retropharyngeal abscess.
D. Acute epiglottitis.
E. Bacterial pharyngitis.

A

D. Acute epiglottitis.

The given clinical picture is very suggestive of acute epiglottitis as the most likely diagnosis. Epiglottitis is an acute inflammation in the supraglottic region of the oropharynx with inflammation of the epiglottis, vallecula, arytenoids, and aryepiglottic folds. Acute epiglottitis affects all age groups. In children it is more common between ages 2 and 4 years; however, introduction of Hemophilus Influenza vaccine has resulted in a significant decline in incidence of the condition in children. The condition is not as common in children anymore.

Epiglottitis often has a short course of progression and presents with high fever, sore throat, dysphagia/odynophagia, and drooling due to inability to swallow the saliva because swallowing is painful and difficult. The child is miserable and often adopts a tripod position with a hyperextended neck to maintain the airway open. Muffled voice (hot potato voice), as if the patient is struggling with a mouthful of hot food, is another feature seen in epiglottitis. Lungs are clear on auscultation but later as the condition escalates, expiratory stridor develops which is an ominous sign.

See photos below:
Tripod position in a child with epiglottitis
How the tripod position maintains the airway patent

(Option A) Croup is the inflammation of larynx with or without involvement of the trachea and bronchi. The condition is caused by viruses with the RSV virus being the most common etiology. Children with croup present with a typical seal barking cough and inspiratory stridor. Fever is often minimal or absent. There is no drooling, dysphagia, odynophagia. There is usually a preceding viral prodrome including runny nose and sneezing. Lungs are clear on auscultation unless more distal airways are involved as well.

(Option B) Bronchiolitis, as the name implies, is the inflammation of bronchiole by viral respiratory infections. It is usually seen in infants younger than 12 months, and present with cough, tachypnea, chest retraction, and widespread wheeze and crackles on lung auscultation.

(Option C) Retropharyngeal abscess has a similar presentation to epiglottitis; however, the rapid onset of symptoms in the absence a preceding bacterial upper respiratory tract infection makes retropharyngeal abscess a far less likely diagnosis.

(Option E) Although bacterial pharyngitis can have a rapid course similar to epiglottitis, there are differences in presentation. Sore throat is a shared feature but acute pharyngitis does not cause the tripod position, hyperextension of the neck, and expiratory stridor. There might be refusal of food due to the sore throat but drooling is rarely seen in acute pharyngitis.

Normal & Abnormal Growth and Development/Child Health/Paediatrics Flashcards by Lubi Mupwaya (17)

96

Q

Caleb, 7 years old, is brought to your clinic by his mother with complaint of bilateral leg pain for the last couple of days. The pain is more prominent on the right side and is felt in the thighs behind the knee and in the calves. The pain occurs intermittently and more often in late evening and in the night and sometimes wakes him up. Caleb had an upper respiratory tract infection 10 days ago presenting with sore throat, mild fever and runny nose. He is
otherwise healthy and has had no serious medical problem to date. His vaccination is up to date. On physical examination, the only remarkable finding is mildly-tender cervical lymphadenopathy. Hip, knee, and ankle joints of the right leg are normal with no swelling, warmth, or tenderness, and have preserved range of motion. He walks normally without limp. Which one of the following could be the most likely diagnosis?

A. Perthes disease.
B. Slipped capital femoral epiphysis (SCFE).
C. Growing pains.
D. Transient tenosynovitis.
E. Septic arthritis of the hip.

A

C. Growing pains.

The scenario is a typical presentation of growing pain.

Growing pains refers to pain in the lower extremities of growing active children. The condition is the cause of leg pain in 20-30% of children between 2 and 12 years with the peak incidence in preschool children. Unlike what the name implies, growing pains are not caused by growth, rather they are hypothesized to be related to excessive physical activity and exercise.

Although accepted as an independent entity, growing pain is a diagnosis of exclusion, and all other potential diagnoses need to be excluded before such diagnosis is made. To make a diagnosis of growing pain, there must be no associated constitutional symptoms such as fever, malaise or change in appetite, no signs of joint involvement (swelling, tenderness, warmth or redness) and no related significant clue in history or physical examination pointing toward another etiology.

The pain is mostly felt in the thighs, calves and behind the knee. The pain is intermittent and in mostly present in the afternoon and evening after activity during the day. Pain may also wake up the child at night. The pain responds to conservative measures such as heat, massage and over-the-counter analgesics, and resolves by the morning.

Growing pains are bilateral in 80%. Although in 20% cases growing pain may be unilateral, it is recommended that to remain on the safe side by considering an alternative diagnosis rather than growing pain is considered and promptly investigated until proven otherwise.

(Option A) Perthes disease is avascular necrosis of the femoral head. The condition is seen mostly in children between 4 and 10 years of age and is associated with constant hip pain and often a limp. Affected hip often has limited range of motion.

(Option B) SCFE is mostly seen in obese boys between ages 10 and 14. Limp and limited range of motion of the affected hip is usually features seen in the condition.

(Option D) Transient tenosynovitis (irritable hip) is the most common cause of limp in the preschool age group usually occurring in children between 3 and 8 years old. There is often a history of preceding viral illness 1 to 2 weeks before. Children with transient tenosynovitis are able to walk but with pain. Affected children are otherwise well and have no fever. The range of motion, especially internal rotation, is mildly decreased. Although the scenario also resembles transient tenosynovitis, the normal range of motion of the hip in Caleb as well as the intermittent nature of the pain and also absence of pain on walking make such diagnosis less likely than growing pains.

(Option E) In septic arthritis of the hip, the child is often ill and febrile. The range of motion of the hip is decreased and joint motion causes pain. The child would refuse to put weight on the affected leg.

97

Q

A 5-year-old boy is presented to your practice with headache and vomiting for the past 6 weeks. On examination, he has a temperature of 37.8°C. A non-tender lymph node is palpated in the posterior triangle of the neck. Which one of the following is the investigation to consider at this stage?

A. Blood culture.
B. CT scan of the head.
C. Urine culture.
D. Lumbar puncture.
E. Fine needle aspiration of the lymph node.

A

E. Fine needle aspiration of the lymph node.

Lymphadenopathy in children, or neck masses in this age group in general, is a common presenting symptom in general practice causing remarkable parental concern as well as a diagnostic dilemma for the doctor.

While lymphadenopathy in children is often a result of a benign inflammatory/ infectious process such as a viral upper respiratory tract infection, more serious diagnoses such as systemic infections, atypical infections and malignancies must also be considered. Examples are: TB, Infectious mononucleosis, cytomegalovirus infection, cat scratch disease, hematological and non-hematological malignancies, and Kawasaki disease.

As always, a thorough history and physical examination is the main component of approaching children with lymphadenopathy or neck mass. Three important areas to inquire are (1) the timeline of the lump (when and for how long?), (2) time course (how fast has t grown/ is it growing), and (3) associated symptoms and potential red flags.

On physical examination, location, size, consistency, and tenderness give important clues.

Lymphadenopathy commonly arises as a lateral lump in the anterior or posterior triangle, and may present an inflammatory or neoplastic process. Lymphadenopathy in the posterior triangle has higher risks of malignancy. Supraclavicular lymphadenopathy is very concerning and considered a red flag.

A very important and crucial part of approach to neck lumps in children is to look for the presence of alarming signs and symptoms in physical exam and history – the red flags.

Red flags include the following:

  • Weight loss
  • Sustained fever
  • Night sweats
  • Generalized lymphadenopathy
  • Signs and symptoms of pancytopenia
  • Mass persisting > 6 weeks
  • Lymph node > 3 cm
  • Thyroid mass
  • Supraclavicular mass
  • Hard, irregular mass
  • Fixed mass

While watchful waiting for 6 weeks is recommended for lymphadenopathy with high likelihood of being reactive (e.g., history of a recent viral infection, node tenderness, etc.), prompt and more invasive approach should be taken in the presence of red flag(s).

This child has a mild fever on examination but it is not clear if it has been sustained. There is also a lymph node found but it is not clear if it has been present for more than 6 weeks. Presence of headache and vomiting are not red flags either but these all together indicate an unwell child, who has painless lymphadenopathy in the posterior triangle. Lymphadenopathies in the posterior triangle have higher chances of association with malignancies. For these reasons, an FNA probably guided by ultrasound is the most appropriate option to consider first among others.

While FNA can be traumatic and carries a risk of inadequate tissue sampling, it is considered a highly sensitive and specific investigation (both reported up to >90%) and is less invasive than open biopsy in all patients carrying red flags.

The presence of fever and lymphadenopathy, may also suggest an infectious process. While there is no strong evidence for the value of a routine full blood count (FBC), it is recommended if systemic disease is suspected or if the diagnosis of infection is uncertain. An FBC and blood film may help identify pancytopenia and atypical cells suggestive of haematological malignancy, while the differential may suggest a viral or bacterial pathology. Although a blood culture (option A) may be required later on bases on the FBC or other findings, it does not seem to take precedence over FNA at this stage.

This child has headache, vomiting and a mild fever. This constellation of symptoms can be seen in children with meningitis. In children, classical signs of meningeal irritation (e.g., Kernig’s and Brudzinski’s signs) might be absent. However, the duration of headache and vomiting for 6 weeks, make meningitis a less likely diagnosis; therefore, an LP (option D) is not necessary for now.

Ultrasonography (USS) is a readily accessible, inexpensive and radiation-free imaging modality that can provide crucial information about the location, size and consistency of a cervical mass. For this reason, ultrasound is the preferred initial imaging study for most children with a neck mass. USS can identify features typical of numerous congenital masses as well as benign and malignant lymphadenopathy, and can provide guidance for fine needle aspiration (FNA) if deemed necessary. USS is the initial imaging modality of choice for thyroid gland pathology including malignancy and thyroglossal duct cysts. CT scan (option B) involves radiation exposure and should be avoided in children unless a malignancy or deep cervical abscess is suspected.

Magnetic resonance imaging (MRI) is an increasingly available and popular alternative to CT and is considered the gold standard imaging study for vascular malformations. While MRI does not involve radiation exposure, the frequent requirement for sedation in the pediatric population does decrease its utility. Histopathology may be required to confirm diagnoses and guide treatment in certain cases.

For this child, urine culture (option C) is not indicated unless investigations exclude other more likely causes of such presentation. This boy is old enough to have more specific symptoms of urinary tract infection such as dysuria or frequency or, at least, abdominal pain. It is unlikely that urinary infections present with painless cervical lymphadenopathy and headache.

TOPIC REVIEW
There is no proven algorithm for management of pediatric neck masses, and management depends on the use of sound clinical judgment and judicious surveillance for red flags.

However the following approaches are often recommended and practiced:
Watchful waiting for up to six for patients with suspected reactive lymphadenitis often evident by bilateral lymphadenopathy with no red flag features for malignancy or deep cervical abscess).

Although the use of empirical antibiotics is controversial, it is the wide accepted practice in patients thought to have suppurative lymphadenitis (e.g., lymphadenopathy with marked erythema and tenderness, asymmetric lymphadenopathy and systemic symptoms).

If symptoms have not improved with antibiotics or if the mass persists longer than four weeks, investigation with ultrasound and serological screens for atypical infections including mycobacteria is warranted. Targeted serological investigations for atypical infections may be indicated if the patient has associated risk factors or if a presumed reactive lymphadenitis has not improved with conservative management.

While there is no strong evidence for the value of a routine full blood count (FBC), it is recommended if
systemic disease is suspected or the diagnosis of infection is uncertain. An FBC and blood film may help
identify pancytopenia and atypical cells suggestive of hematological malignancy, while the differential may
suggest a viral or bacterial pathology.

98

Q

Tom, 10 years old, is brought to your clinic by his mother who is concerned about his height and weight. She says Tom has always been the biggest boy in his classes since he started school. On examination, his weight and height are on 96 and 97 percentile, respectively. Which one of the following would be the most important initial evaluation to consider?

A. TSH.
B. Urine cortisol level.
C. Bone age.
D. Blood sugar.
E. MRI of the head.

A

D. Blood sugar.

For children and adolescents aged 2–18 years, growth is monitored based on age, height, and weight, using sexspecific Body Mass Index (BMI) percentile charts. BMI is not a fixed measure in this age group but varies with normal growth, stage of puberty, and sex. Either the United States Centers for Disease Prevention and Control (US-CDC) or WHO BMI percentile charts may be used, with the same chart used over time to allow for consistent monitoring of growth.

The US-CDC categorizes overweight as between the 85th and 95th percentile and obesity as above the 95th percentile. The WHO categorizes overweight as between the 85th and 97th BMI percentiles and obesity as above the 97th percentile. These categories are not diagnostic but contribute to the overall clinical impression of the child or adolescent being measured.

Tom, based on either chart, is obese. The likelihood that childhood overweightness and obesity will persist into adulthood increases with the age of the child and with the presence of parental obesity. One of the strongest predictors of a child’s weight is the weight status of his/her parents. For overweight or obese children, initial assessment by history and clinical assessment should determine current health problems and risks for future disease.

History taking includes developmental history, physical and mental health (including family history of obesity), and current health behaviors.

Clinical assessment includes pubertal stage, possible causes for overweightness or obesity (e.g., hypothyroidism), and indicators of comorbidities (e.g., metabolic syndrome, raised blood pressure, joint pain, gastrointestinal symptoms, insulin resistance, intertrigo, dental health etc.)

Of the options, measuring blood glucose is the most important investigation to consider at this stage as well as a lipid profile and liver function tests. Obese children are at increased risk of metabolic syndromes and the condition should be identified and addressed early in the course of assessment and treatment.

It is also very important that secondary causes for obesity are considered and excluded. Hypothyroidism can be a potential cause of obesity; however, the prevalence of hypothyroidism in overweight and obese children and adolescents is not greater than in normal-weight ones. Given these, measuring TSH (option A) would be an appropriate option if the clinical picture is suggestive of hypothyroidism or if no apparent cause for overweightness or obesity can be identified. It is however important to note that the prevalence of hypothyroidism is not greater than in normal-weight children and adolescents.

(Option B) Urine cortisol level would have been indicated if Tom had clinical features suggestive of Cushing syndrome such as moon face, central obesity, etc.

(Option C) Bone age is often used as the initial assessment for short stature that is not the problem here.

(Option E) This child does not have any alarming features necessitating brain imaging using MRI or CT scan.

99

Q

A 6-year-old Indigenous boy presents to your practice in a rural area presents with purulent left ear discharge for the past 2 months. History and physical examination establish a diagnosis of chronic suppurative otitis media. Culture of the ear discharge is positive for pseudomonas aeruginosa. After ear toileting, which one of the following is the most appropriate treatment option to consider for this child?

A. Oral Amoxicillin.
B. Oral Augmentin.
C. Ciprofloxacin ear drop.
D. Paramycetin with steroid ear drops.
E. Gentamycin ear drops.

A

C. Ciprofloxacin ear drop.

Chronic suppurative otitis media (CSOM) is the infection of the middle ear with a perforated tympanic membrane and discharge of at least 6 weeks duration. CSOM can cause hearing impairment. Occasionally and in less developed countries and Aboriginal and Torres Strait Islander people, serious complications such as intracranial infections and mastoiditis can occur.

Treatment of CSOM includes ear toileting and topical antibiotics. Ear toileting is a very important step and should be performed before instillation of antibiotic drops. Ear toileting can be performed by a healthcare professional with measures such as mechanical suction, or cotton wool on a probe under direct visualization. It can be also done by the patient or cared using dry mopping the ear with rolled tissue spears or similar, 6-hourly.

Quinolones such as ciprofloxacin 3% ear drop is the antibiotic of choice after ear toileting achieves a dry and clean ear canal. It is given as 5 drops into the affected ear, 12-hourly, until the middle ear has been free of discharge for at least 3 days.

Oral antibiotics such as amoxicillin (option A) or Augmentin (option B) are not appropriate options because the most common organism in CSOM is pseudomonas aeruginosa which is a gram-negative pathogen. Since topical treatment eradicates the infection, systemic antibiotic therapy is unnecessary.

Paramycetin (chloramphenicol) (option D) do not adequately cover pseudomonas aeruginosa and is not an appropriate treatment option. Also, there is inadequate evidence to support use of topical steroids in combination with topical antibiotics for treatment of CSOM.

Aminoglycoside-based ear drops such as gentamycin (option E) have been previously used for CSOM; however, due to concerns of safety, especially the risk of auditory and vestibular toxicity are not preferred options now.

100

Q

An 8-year-old boy is brought to the Emergency Department with right ear pain and purulent discharge since yesterday and a red swelling behind the right ear since this morning. He has the history of chronic bilateral otitis media for which he underwent bilateral grormmet procedure. Which one of the following is the most appropriate next step in management?

A. Take samples for culture.
B. Hearing assessment.
C. Antibiotics.
D. CT scanning of the temporal bone.
E. Involvement of an ENT consultant.

A

A. Take samples for culture.

The scenario is a typical presentation of acute mastoiditis (AM). Acute mastoiditis is rare yet the most common suppurative complication of acute otitis media (AOM) and may be associated with intracranial complications. The diagnosis of AM is almost always made clinically based on the presence a current or preceding history of AOM plus the following findings:

  • Post auricular inflammatory signs such as erythema, edema, tenderness, or fluctuance,
    and/ or
  • A protruding auricle and/or external auditory canal edema

The mainstay of initial treatment is intravenous antibiotics (flucloxacillin + a third-generation cephalosporin) (option C). Before commencement of antibiotics, however, a sample from the ear discharge should be taken for microscopy, culture, and sensitivity because antibiotics may alter the result. If the tympanic membrane is intact, tympanocentesis or myringotomy should be obtained to both relief the middle ear pressure and obtain a sample. This should be
performed by an ENT consultant. Some cases may require surgical treatment; therefore, an ENT consultant must be involved as well (option E).

CT scanning of the temporal bone (option D) is the standard for evaluation of the extend and complications of mastoiditis and may be considered for further management planning. The results do not affect the initial management; therefore, not the best appropriate next step. It may later be considered for further management, often by the ENT specialist.

A hearing assessment (option B) using audiometry must be obtained after the convalescence
from the acute phase mostly due to medicolegal issues. Audiometry is rarely appropriate or useful during the acute phase of AM.

NOTE - In general, mastoiditis includes all inflammatory processes of the mastoid air cells of the temporal bone.

Since the mastoid is contiguous to and an extension of the middle ear cleft in the temporal bone, by definition and virtually, every patient with acute otitis media or chronic middle ear inflammatory disease has mastoiditis.

However, in practice, acute mastoiditis refers to instances where middle ear infection spreads beyond the mucosa of the middle ear cleft, and there is involvement of mastoid air-cell system or periosteitis of the middle ear cleft. While acute mastoiditis is associated with acute otitis media (AOM), chronic mastoiditis is most associated with chronic suppurative otitis media (CSOM) and particularly with cholesteatoma formation.

101

Q

Raymond, 9 years of age, is brought to your GP clinic with right sided hearing impairment. A Rinne test shows that air conduction is better than bone conduction in both ears. Weber test indicates localization to the left ear. If the following options, which one justifies these findings?

A. Acute otitis media.
B. Acute otitis externa.
C. Glue ear.
D. Otosclerosis.
E. Acoustic neuroma.

A

E. Acoustic neuroma.

Hearing loss is a common presentation to clinical practice. In such cases Rinne and Weber tests are performed to differentiate between a conductive (middle and outer ear causes) and a sensorineural deafness (caused by damage to the cochlea or to the 8th nerve – or its central connections). These tests are always done together. The Rinne test is performed first.

The function of the external ear is to collect sounds vibrations form the air and focus these onto the tympanic membrane. These vibrations are then transmitted through the middle ear cavity by the ossicular chain (Malleus, Incus and Stapes). The stapes transmits these vibrations to the cochlea through the oval window (fenestra ovalis).

Sound can also be transmitted through the bones of the skull to the cochlea. The hair cells in the cochlea convert the physical vibrations into action potentials that are transmitted via the nerves in the vestibulo-cochlear (auditory) nerve to the brainstem for further processing.

The Rinne test is performed for evaluation of hearing loss in one ear (unilateral hearing loss) and compares
perception of sound transmitted by air conduction to those transmitted by bone conduction through the mastoid.

For the Rinne test, a vibrating tuning fork (typically 512 Hz) is placed initially on the mastoid process behind each ear until sound is no longer heard, signaled by the patient. The fork is then immediately positioned just outside the ear with the patient asked to report when the sound caused by the vibration is no longer heard. A normal or positive Rinne test is when the patient can still hear the vibration while the fork is in front of the ear (air conduction or AC)
after he/she could not hear it while the tuning fork was in the mastoid bone (bone conduction or BC) [AC>BC]. In conductive hearing loss, bone conduction is better than air or BC > AC, a negative Rinne.

In the Weber test, a vibrating tuning fork (often 256Hz or 512Hz) is placed in the middle of the forehead, or above the upper lip under the nose over the teeth, or on top of the head in an equal distant from the patient’s ears. The patient is then asked to report in which ear the sound is louder. A Weber test result is normal (positive) if the patient reports the sound equally on both sides. Lateralization (hearing the sound better in one ear) is due to either a sensorineural or conductive problem. In a patient with sensorineural hearing loss, the normal ear hears the sound better than the defective one. In a patient with conductive hearing loss, the sound is lateralized to the affected side.

In this child, air conduction is greater than bone conduction in both ears (bilateral positive Rinne). This means that the cause of hearing loss is not conductive. In other words, acute otitis media, (option A), acute otitis externa (option B), Glue ear (otitis media with effusion) (option C), or otosclerosis (option D) [affecting the ossicular chain the middle hear, and impairs sound transmission] cannot be the underlying cause of the hearing impairment in this child.

Weber test, on the other hand, shows lateralization the left side. Given the normal Rinne test results, there is right-sided sensorineural hearing impairment that only can be justified by acoustic neuroma among options.

Acoustic neuromas are intracranial, extra-axial tumors arising from the Schwann cell sheath investing either the vestibular or cochlear nerve. As acoustic neuromas increase in size, they eventually occupy a large portion of the cerebellopontine angle. Acoustic neuromas account for approximately 80% of tumors found within the cerebellopontine angle. The remaining 20% are principally meningiomas.

Unilateral hearing loss is overwhelmingly the most common presenting symptom. Any unilateral sensorineural hearing loss should be assumed to have been caused by an acoustic neuroma until proven otherwise. The tumor can produce hearing loss through at least two mechanisms, direct injury to the cochlear nerve or interruption of cochlear blood supply. Progressive injury to cochlear fibers probably accounts for slow progressive neurosensory hearing loss observed in a significant number of patients with acoustic neuromas. Sudden and fluctuating hearing losses are likely to be caused by disruption of cochlear blood supply.

Headache and imbalance are other symptoms that may develop later in the course of the disease but are less common as presenting symptoms compared to hearing loss.

Acoustic neuroma is rare in children, yet it is the only explanation for this case scenario among other options.

102

Q

A 5-month-old child is brought to your practice with a 10-day history of cough paroxysms followed by inspiratory
whoop and vomiting. There is no cyanosis associated with bouts of cough, and the child is well between the attacks.
Which one of the following is the most appropriate action now?

A. Reassure.
B. Prescribe a 5-day course of antibiotics and discharge the child home.
C. Admit the child to the hospital for observation.
D. Chemoprophylaxis of the household.
E. Vaccinate the household.

A

C. Admit the child to the hospital for observation.

The clinical picture is consistent with whooping cough (pertussis) caused by Bordetella pertussis.

Pertussis is a prolonged coughing illness with varying clinical manifestations depending on age and immunization status. An initial catarrhal phase is characterized by insidious onset of runny nose, sneezing absent or low-grade fever, and a mild occasional cough. The cough gradually becomes paroxysmal usually after 1-2 weeks, and may end in vomiting, cyanosis, and/or a characteristic high-pitched inspiratory whoop. Paroxysms can recur with subsequent respiratory illnesses for many months after the onset of pertussis. Fever is generally minimal or absent throughout the course of the disease. Infants are less likely to have the inspiratory whoop and a significant catarrhal phase and are more likely to present with gagging, gasping, cyanosis, apnea or non-specific signs such as poor feeding or seizures.

Types of patients to be admitted & managed in the hospital:
1. Infants < 6 months
2. Any child with complications (ie. apnea, cyanosis, pneumonia, encephalopathy)

This child is below 6 months, thus indicates admission to hospital.

103

Q

A 7-month-old child is brought to your practice with paroxysmal cough and vomiting for 2 weeks and relative wellness and health in between paroxysms. You suspect he may have whooping cough (pertussis). He has already been started on azithromycin. He lives at home with his mother, father and older brothers, aged 3 and 5 years.

Neither of his brothers have been
immunized against pertussis. Which one of the following options is the most appropriate next action to take?

A. Arrange to have the child admitted to hospital, and isolated immediately.
B. Report the family to the child protection agency in your state for failing to immunize their children.
C. Immediately vaccinate his brothers.
D. Prescribe azithromycin for the whole family.
E. Take no action.

A

Correct answer is C.

This child is older than 6 months and does not have any serious complication of Bordetella pertussis infections such as cyanosis, apnea or pneumonia; therefore, admission to hospital is not required.

The decision as to whether a child is immunized is made by the parents. Parents can decide to not immunize their children and it is not an act subject to notification to child protection authorities.
Parents can be talked into vaccinating their children for their benefit but they cannot be forced to because this is their right to refuse vaccination of their children. Moreover, vaccination at this point will not help the current situation.

Prophylaxis against pertussis in an attempt to decrease spread to the community is indicated in the following situations:

The close contact with confirmed case of pertussis (index case) while index case is infectious, which is within the first 21 days of cough and <5 days of antibiotics
AND
First contact was within 14 days (or within 21 days for infants <6 months) – beyond that time prophylaxis isnot effective

If the above criteria are met, the following groups should receive prophylactic antibiotics:

[CHILDREN]
- Age <6 months; OR
- <3 doses pertussis vaccine; OR
- There is a household member aged <6 months; OR
- Children who attend childcare in same room as infant <6 months

[ADULTS (regardless of immunization status)]
- Expectant parents in last month of pregnancy; OR
- Healthcare workers in maternity hospital or newborn nursery; OR
- Childcare worker in close contact with infants <6 months; OR
- Household members aged <6 months

Prophylaxis is indicated for the 3- and 5-year-old children, but not for parents. Therefore, the option suggesting that all family members should receive prophylactic antibiotics is incorrect. It is recommended that close contacts that are not up-to-date with their pertussis immunization should be given DTPa or dTpa as soon after exposure as possible. In the absence of an option suggesting prophylaxis for 3- and 5-year-old children, immediate vaccination of these two children would be the most appropriate alternative option. Vaccinated adults, who have not had pertussis containing vaccine in the last 10 years, should receive a booster dose by dTpa.

104

Q

Which one of the following statements is correct about pertussis?

A. Febrile convulsions are not a contraindication for vaccination against pertussis.
B. DTPa cannot be given with another live virus-containing vaccine.
C. Pertussis vaccine should not be given to a child with infantile convulsions.
D. Erythromycin reduces the severity of the signs and symptoms.
E. Prophylaxis is always indicated for the entire household.

A

A. Febrile convulsions are not a contraindication for vaccination against pertussis.

Febrile convulsions are very rarely reported following DTPa-containing vaccines within 48 hours of vaccination. The risk is even lower in infants who complete their primary course at 6 months of age because febrile convulsions are uncommon in children <6 months of age. Children who experience a febrile convulsion after a dose of DTPa-containing vaccine have a slightly increased risk of further febrile convulsions following a subsequent dose of DTPa-containing vaccine. The risk can be minimized by appropriate measures to prevent fever, so vaccination is still recommended.

(Option B) Simultaneous administration (that is administration on the same day) of the most widely used live and inactivated vaccines does not result in decreased antibody responses or increased rates of adverse reactions. Simultaneous administration of all vaccines for which a child is eligible is very important in childhood vaccination programs because it increases the probability that a child will be fully immunized at an appropriate age; therefore, DTPa can be given with other live-virus vaccines.

(Option C) Pertussis-containing vaccines do not cause infantile spasms or epilepsy. Infants and children known to have active or progressive neurological disease can be safely vaccinated with DTPa-containing vaccines.

(Option D) Antibiotics are not likely to reduce the severity of signs and symptoms; however, they can reduce the infectious period, and decrease the communicability of the disease.

(Option E) Chemoprophylaxis to prevent secondary transmission is NOT recommended in most situations because of the delayed presentation of the index case and the cost and adverse effects of antibiotics. However, given the high risk of mortality and morbidity associated with infection of the newborn, particularly in the context of the rising incidence of pertussis in the community and the high transmission rate, chemoprophylaxis is recommended to limit transmission to those most at risk of the infection (young infants). Data to support this recommendation are limited. Australian guidelines recommend post-exposure chemoprophylaxis for contacts to whom transmission is most likely, and when there is significant risk of morbidity or mortality or risk of transmission to other high-risk groups. Routine chemoprophylaxis of all household is not indicated unless certain criteria are met.

105

Q

Which one of the following is the most common cause of persistent cough in children?

A. Post-viral cough.
B. Asthma.
C. Post-pertussis.
D. Gastro-esophageal reflux disease (GERD).
E. Passive smoking.

A

B. Asthma.

By duration, cough is defined as acute (<3 weeks), subacute (3-8 weeks) and chronic or persistent (>8 weeks).

Asthma is the most common cause of persistent (chronic) cough in children and the second most common cause in adults (after chronic bronchitis).

Post-viral cough is the most common cause of acute and subacute (not chronic) cough in children.

Other causes of persistent cough in children are as follows:

  • Persistent bacterial bronchitis
  • Chronic suppurative lung disease and bronchiectasis
  • Foreign body
  • Gastro-esophageal reflux disease (GERD)
  • Upper respiratory cough syndrome (formerly known as postnasal drip syndrome)
  • Post-pertussis cough
  • Increased cough receptor sensitivity
  • Habit (psychogenic) cough
  • Otogenic cough

106

Q

A mother brings her 3-week-old male baby to the Emergency Department because of his loose bloody bowel motions. The mother says the diarrhea has started 2 days ago after she started her son on formula milk. The mother is lactose-intolerant and does not use dairy products.

On examination, the child is mildly dehydrated and has an eczematous rash on his right cheek. He is not febrile. The rest of the exam is inconclusive. Family history is remarkable for atopy including asthma and eczema in his father and older sister. Mother is concerned that his son may have the same allergy to milk and dairy product she has. Which one of the following is the most likely diagnosis?

A. Lactose intolerance.
B. Gastroenteritis.
C. Galactosemia.
D. Fructose intolerance.
E. Cow’s milk protein intolerance.

A

E. Cow’s milk protein intolerance.

The onset of bloody diarrhea shortly after introduction of formula, the presence of eczema, and family history of atopy makes cow’s milk protein allergy (intolerance) (CMPA) the most likely diagnosis with high certainty.

CMPA results from an immunological reaction to one or more milk proteins. This immunological basis distinguishes CMPA from other adverse reactions to cow’s milk protein such as lactose intolerance.
CMPA may be immunoglobulin E (IgE) or non-IgE associated. In IgE-associated cases, CMPA may be a manifestation of the atopic diathesis. These reactions may occur as short as minutes after ingestion of cow’s milk or cow’s milkbased formula. These early reactions usually manifest as urticaria, angioedema, vomiting or an acute flare of atopic dermatitis. The remaining 42% show a later reaction, typically atopic dermatitis or gastrointestinal disturbances.

Even small amounts of cow’s milk in breast milk of mother’s who take dairy products may trigger the condition.

Other options do not justify the clinical presentation.

107

Q

A 2-year-old boy is brought to the Emergency Department, at 11 pm, by his parents with complaints of a harsh barking cough. On examination, he is febrile with a temperature of 39°C and has a respiratory rate of 32 breaths per minute. There is inspiratory stridor as well as scattered crackles over both lung fields. He is easily irritated but can be consoled by the parents. Which one of the following is most important to give him first?

A. Intravenous prednisolone.
B. Nebulized adrenaline.
C. Nebulized salbutamol.
D. Oral steroids.
E. Antibiotics.

A

D. Oral steroids.

The age of the child, the fever, the harsh cough and more importantly the stridor is suggestive of croup until proven otherwise.

In mild cases (a croupy cough but no history of noisy breathing in a child older than 2 years), no treatment is required, but early review is essential.

In moderate to severe cases (e.g., a history of noisy breathing or age less than 2 years, or presence of stridor on examination), a single dose of corticosteroid is indicated.

Use either:
Dexamethasone 0.3 mg/kg orally (first-line); or
Predniso(lo)ne 1mg/kg orally, or
Budesonide 2mg by nebulizer

Further doses are usually not needed, but can be considered if response to therapy is suboptimal at 24 hours.

In more severe cases with significant airway obstruction or fatigue, treatment in hospital may be required with an initial dose of:

  • Adrenaline 1% (1:100, 10mg/ml) solution 0.05ml/kg/dose up to 0.5ml diluted with NaCl 0.9%, by nebulizer
    PLUS EITHER
  • Dexamethasone 0.6mg/kg orally (or IM if vomiting), or
  • Predniso(lo)ne 1mg/kg orally

Maintenance dose and frequency vary with the severity of the condition and the response to treatment.

This child has stridor and is irritable but consolable; therefore, he is most likely having moderate croup. He should be given a single dose of oral dexamethasone or predniso(lo)ne or nebulized budesonide.

(Option A) Intravenous administration of steroids is not indicated in patients with croup.

(Option B) Nebulized adrenaline should be reserved for cases with severe croup.

(Option C) Bronchodilators (e.g., salbutamol) in any form have no role in treatment of croup.

(Option E) Antibiotics would be indicated if the underlying cause is found to be bacterial tracheitis, but airway always takes precedence.

108

Q

A 4-year-old girl is presented with the proptosis of her left eye and a white cornea which was noticed yesterday by her parents. Based on the exam findings, you suspect retinoblastoma. Which one of the following is the most appropriate initial diagnostic test?

A. MRI.
B. Ultrasound.
C. CT scan.
D. X-ray.
E. Biopsy.

A

** B. Ultrasound.**

This child has suspected diagnosis of retinoblastoma.

Retinoblastoma is the most common intraocular tumor in children. Leukocoria (white cornea or Cat’s eye) is the most common presenting sign in developed countries, followed by strabismus as the second most common feature.

Other findings include decreased vision, ocular inflammation, vitreous hemorrhage, hyphema, orbital cellulitis, proptosis, glaucoma, eye pain, and fever. A family history is present in 5 -10% of cases.
The diagnosis is usually made by dilated indirect ophthalmoscopic examination under anesthesia. The characteristic finding is a chalky, white-grey retinal mass with soft and friable consistency.

Ultrasonography is cheap and readily available and is used in conjunction with physical exam to establish the diagnosis.

Characteristic finding on ultrasonography is intraocular calcification.

(Option A) Together with ultrasonography, high-resolution MRI has emerged as an important imaging modality for pretreatment assessment and diagnostic confirmation, detection of local tumor extension, involvement of the optic nerve, detection of associated developmental malformations of the brain, and detection of associated intracranial primitive neuroectodermal tumors (trilateral retinoblastoma). However, MRI is not the initial investigation and is considered after physical exam, funduscopy and ultrasound established the diagnosis.

(Options C and D) In the past, CT was used to determine tumor size, retro-orbital spread and intracranial growth; however, because of its superior soft-tissue contrast, MRI is more sensitive and specific than CT in detection of tumor extent and metastatic risk factors. Furthermore, CT and x-rays scan poses children with hereditary retinoblastoma at increased risk of other malignancies given the fact that hereditary retinoblastoma is caused by mutation in RB1 gene which is a tumor suppressor. Routine use of CT scan as a diagnostic modality is discouraged; X-ray carries the same risk.

(Option E) Biopsy has no role in diagnosis of retinoblastoma.

109

Q

A 42 year-old woman presents to your office complaining of marked lack of energy, breast tenderness and insomnia. These symptoms start a week before the onset of menses. She is severely depressed as she has been unable to concentrate at work during these times. Her symptoms improve after the onset of menses. She neither smokes nor drinks alcohol. She has no history of any mental illness in the past. Which one of the following would be the most appropriate treatment?

A. Evening prim rose oil.
B. Fluoxetine.
C. Oral contraceptive pills.
D. Non-steroidal anti-inflammatory drugs.
E. Bromocriptine.

A

B. Fluoxetine

With severe depression, lack of energy, difficulty concentrating, insomnia, and breast tenderness this patient has 5 out of 11 symptoms of premenstrual dysphoric disorder (PMDD). The symptoms are affecting her work (lack of concentration at work), are related to menstruation, and have been present for more than 2 cycles. Furthermore, these symptoms cannot be attributed to any other underlying psychiatric conditions as the history suggests. With these, this patient has PMDD and selective serotonin reuptake inhibitors (SSRIs) are the most effective (not first line) medications for treatment. This does not mean that SSRIs should be start immediately because every pharmacological approach should be considered if conservative measures such as CBT, regular exercise, life style modification and supplementation with vitamin E, B6, calcium and magnesium proved ineffective after at least 2-3 menstrual cycles.

Systematic review of SSRIs for PMDD concluded that these drugs are effective. Fluoxetine 20mg/day has been shown to decrease the symptoms of PMS and PMDD with an overall response of 60-75%. Sertraline (50-150mg/day), paroxetine (20- 30mg/day) and citalopram (20-30mg/day) can be used with aboout the same efficacy. Adverse effects of SSRIs include, but not limited to, headache, anxiety, nausea and impaired sexual function. Sexual dysfunction including anorg*smia and decreased libido may be the most problematic adverse effects.

Some women who do not respond to one SSRI may respond to a second SSRI and it is worth attempting this switch in non-responders. Additionally, women who do not respond completely to intermittent therapy may benefit from daily therapy and some women who undergo daily treatment may do better with intermittent treatment.

GnRH antagonists and danazol would be considered as next step if SSRIs are ineffective. The rationale behind their use is the fact that ovarian steroids have been proved to be the underlying cause of PMS and PMDD through a rather unknown mechanism.

GnRH antagonists and danazol, by suppression of ovarian steroid production, decrease the symptoms. Their adverse effect profile, however, makes them a last-resort therapy for PMS and PMDD. Acne and hirsutism are the most common complications.

Vitamins such as B6, D, and E have been studied with controversial results. Despite lack of solid evidence these agents can be used, as well as regular exercise and healthy life style, as the first –line treatment for mild to moderate PMS. PMDD, however, demands more prompt and aggressive treatment with SSRIs as the first-line treatment option.

OPTION C : Oral contraceptives may have an effect on reducing the symptoms of PMS and are the simplest method of inducing anovulation. Older combined oral contraceptives were not more effective than placebo in several studies; however, new meta- analysis suggests that an oral contraceptive containing the progestin, drospirenone, and use of a 4 rather than 7 day pill-free interval, is more effective than placebo for symptom relief in women with PMDD. Oral contraceptives may be used as first-line treatment if only physical symptoms of PMS are of great concern and significance.

OPTION D : NSAIDs are ineffective in treatment of PMDD, but they occasionally might be considered for aches and pain associated with PMS.

OPTION A : Evening prim rose oil has no effect over placebo in PMS overall, but studies has shown that it can be effective in management of cyclic mastalgia.

TOPIC REVIEW

Premenstrual dysphoric disorder (PMDD)

PMDD is severe form of premenstrual syndrome and is characterized by severe feeling of sadness, emotional labiality with frequent tearfulness, loss of interest in daily activities, decreased concentration, fatigue, insomnia and feeling of being overwhelmed or out of control. Symptoms are more related to affections rather than being somatic. In other words, affective impairment is the dominating feature.

The Diagnostic and Statistical Manual of Mental Disorders, Fifth Edition (DSM-5), established 4 research criteria (A through D) for the diagnosis of PMDD.

Criterion A - in most menstrual cycles during the past year, at least 5 of the following 11 symptoms (including at least 1 of the first 4 listed) were present:

  1. Markedly depressed mood, feelings of hopelessness, or self-deprecating thoughts
  2. Marked anxiety, tension, feelings of being ‘keyed up’ or ‘on edge’
  3. Marked affective lability (e.g. feeling suddenly sad or tearful or experiencing increased sensitivity to rejection) 4. Persistent and marked anger or irritability or increased interpersonal conflicts
  4. Decreased interest in usual activities (e.g. work, school, friends, and hobbies)
    6 . Subjective sense of difficulty in concentrating
  5. Lethargy, easy fatigability, or marked lack of energy
    8 . Marked change in appetite, overeating, or specific food cravings
  6. Hypersomnia or insomnia
  7. A subjective sense of being overwhelmed or out of control
  8. Other physical symptoms, such as breast tenderness or swelling, headaches, joint or muscle pain, a sensation of bloating, or weight gain

The symptoms must have been present for most of the time during the last week of the luteal phase, must have begun to remit within a few days of the onset of menstrual flow, and must be absent in the week after menses.

Criterion B - the symptoms must be severe enough to interfere significantly with social, occupational, sexual, or scholastic functioning. For example, the patient may avoid social activities or exhibit decreased productivity and efficiency at work or school.

Criterion C - the symptoms must be discretely related to the menstrual cycle and must not merely represent an exacerbation of the symptoms of another disorder, such as major depressive disorder, panic disorder, dysthymic disorder, or a personality disorder (although the symptoms may be superimposed on those of any of these disorders).

Criterion D - criteria A, B, and C must be confirmed by prospective daily ratings during at least 2 consecutive symptomatic menstrual cycles. The diagnosis may be made provisionally before this confirmation.

Of the 11 symptoms listed in DSM-5, 10 are emotional and behavioral in nature; only 1 includes multiple common physical symptoms. Thus, PMDD defines a narrow group of women with the most severe premenstrual emotional symptoms, with functional impairment, and without a concurrent Axis I or Axis II disorder that is exacerbated premenstrually.

Normal & Abnormal Growth and Development/Child Health/Paediatrics Flashcards by Lubi Mupwaya (2024)
Top Articles
Lexus RX (1998-2023) firing order — what is it? | REREV
Lexus GS (1997-2020) firing order — diagram & guide | REREV
12 Rue Gotlib 21St Arrondissem*nt
Zavvi Discount Code → 55% Off in September 2024
Petco Clinic Hours
Miller Motte College Student Portal
Use Caution: Herds of wild horses escaping Davis Fire spotted evacuating up Geiger Grade
Dyi Urban Dictionary
I Feel Pretty (2018) | Rotten Tomatoes
2014 Can-Am Spyder ST-S
Walmart Front Door Wreaths
An Honest Review of Accor Live Limitless (ALL) Loyalty Program
Sinai Web Scheduler
Hangar 67
Seafood Restaurants Open Late Near Me
Lubbock Avalanche Journal Newspaper Obituaries
Inloggen bij AH Sam - E-Overheid
Voy Pageant Discussion
'Kendall Jenner of Bodybuilding' Vladislava Galagan Shares Her Best Fitness Advice For Women – Fitness Volt
Dow Futures Pre Market Cnn
My Time Banner Health
Clarksville.craigslist
Coleman Funeral Home Olive Branch Ms Obituaries
Elm Nychhc Org
G122 Pink Pill
Oh The Pawsibilities Salon & Stay Plano
Craigslist Vt Heavy Equipment - Craigslist Near You
Jvid Rina Sauce
Tamiblasters.in
When Is Meg Macnamara Due
Jill Vasil Sell Obituary
Walgreens On 37Th And Woodlawn
Ansos Umm
Erste Schritte für deine Flipboard Magazine — Ein Blogger-Guide -
Deshaun Watson Stats, News and Video - QB | NFL.com
Shirley Arica Unlock
Issue November 5, 1949 - The Hockey News
Dimmitt Range Rover
Mychart University Of Iowa Hospital
Sierra Vista Jail Mugshots
My Perspectives Grade 10 Volume 1 Answer Key Pdf
Famous Church Sermons
Open The Excel Workbook Revenue.xls From The Default Directory
Barbie: A Touch of Magic
Ms Trigger Happy Twitter
Play Jelly Collapse Game: Free Online Colorful Tile Matching Breaker Video Game for Kids & Adults
Magnifeye Alcon
Siôn Parry: The Welshman in the red of Canada
Morse Road Bmv Hours
Toldeo Craigslist
Synergy Grand Rapids Public Schools
Good Number To Shoot For
Latest Posts
Article information

Author: Arline Emard IV

Last Updated:

Views: 6169

Rating: 4.1 / 5 (72 voted)

Reviews: 87% of readers found this page helpful

Author information

Name: Arline Emard IV

Birthday: 1996-07-10

Address: 8912 Hintz Shore, West Louie, AZ 69363-0747

Phone: +13454700762376

Job: Administration Technician

Hobby: Paintball, Horseback riding, Cycling, Running, Macrame, Playing musical instruments, Soapmaking

Introduction: My name is Arline Emard IV, I am a cheerful, gorgeous, colorful, joyous, excited, super, inquisitive person who loves writing and wants to share my knowledge and understanding with you.